Download NEET PG 2020 Question Paper with Answers

Download NEET PG (The National Eligibility cum Entrance Test Postgraduate) 2020 Question Paper with Answers

1.Upwardmovementofthethyroidglandis
preventeddueto?
a)Berryligament
b)Pretrachealfascia
c)Sternothyroidmuscle
d)Thyrohyoidmembrane
CorrectAnswer-B
Ans:B.Pretrachealfascia
Thethyroidglandiscoveredbyathinfibrouscapsule,whichhasan
innerandanouterlayer.Theinnerlayerextrudesintotheglandand
formstheseptumthatdividesthethyroidtissueintomicroscopic
lobules.
Theouterlayeriscontinuouswiththepretrachealfascia,attaching
theglandtothecricoidandthyroidcartilagesviaathickeningofthe
fasciatoformtheposteriorsuspensoryligamentofthethyroidgland
alsoknownasBerry'sligament.Thiscausesthethyroidtomoveup
anddownwiththemovementofthesecartilageswhenswallowing
occurs.
Gray'sAnatomy:TheAnatomicalBasisofClinicalPractice,41e
,Pageno470


2.Thereasonforthelongleftrecurrent
laryngealnerveisduetothepersistenceof
whicharchartery?

a)3rdarch
b)4tharch
c)5tharch
d)2ndarch
CorrectAnswer-B
Ans:B.4tharch
LeftRLNwindsaroundthearchofaorta
Archofaortaisderivedfromthe4tharch
LangmansMedicalEmbryology13thedition(Pageno88,239)

3.Ligationofthehepaticarterywillimpair
bloodsupplyin
a)RightgastricandRightgastroepiploicartery
b)RightgastricandLeftgastricartery
c)Rightgastroepiploicandshortgastricvessels
d)Rightgastricandshortgastricvessels
CorrectAnswer-A
Ans:A.RightgastricandRightgastroepiploicartery
Therightgastricarteryisabranchofthecommonhepaticartery
Therightgastroepiploicarteryisabranchofthegastroduodenal
arterywhichisabranchofthecommonhepaticartery
Theleftgastricarteryisabranchoftheceliactrunk
Shortgastricvesselsarisefromthesplenicartery
Gray'sAnatomy:TheAnatomicalBasisofClinicalPractice,41st
Edition(Pagenos1116and1117)


4.Wolffianductremnantinfemaleis
a)PouchofDouglas
b)Uterovesicalpouch
c)Gartner'scyst
d)Broadligament
CorrectAnswer-C
Ans:C.Gartner'scyst
Gartner'scysts,sometimesincorrectlyreferredtoasvaginal
inclusioncysts,arethemostcommonbenigncysticlesionsofthe
vagina.
Theyrepresentembryologicalremnantsofthecaudalendofthe
mesonephric(Wolffian)duct.
Gartner'sductsarefoundinabout25%ofadultwomen.Almostone
percentoftheseductsevolveintoGartner'sductcyst.
Ref:AcaseofGartner'scystofvagina,J.AnesthCritcare
OpenAccess,2017,00259.


5.Nervesupplyoftheextraocularmusclesis
constitutedbyallexcept
a)Ophthalmicnerve
b)Oculomotornerve
c)Trochlearnerve
d)Abducentnerve
CorrectAnswer-A
Ans:A.Ophthalmicnerve
Lateralrectusissuppliedby6thcranialnerve(abducentnerve)
Superiorrectusissuppliedby4thcranialnerve(trochlearnerve)
Alloftheremainingextraocularmusclesaresuppliedbythe
oculomotornerve.Theophthalmicnerveisabranchofthe
trigeminalnerveandispurelysensoryinnature.
BDC7thedition,volume3,pgno.215.

6.Claudicationduetopoplitealfemoral
incompetenceisprimarilyseenin
a)Thigh
b)Calf
c)Buttocks
d)Feet
CorrectAnswer-B
Ans:B.Calf
Calf
AortaandCommonIliac-Buttocks
FemoralArtery-Thigh
Superficialfemoralartery-Calfandpoplitealartery
PosteriortibialArtery-Feet
BDC7thedition,volume2,pageno137.

7.Whichmuscleisparalyzedifthereis
hyperextensionofmetacarpophalangeal
jointandflexionoftheinterphalangeal
joint?

a)Extensordigitorum
b)Interosseiandlumbricals
c)Adductorpollicis
d)Pronatorquadratusmuscle
CorrectAnswer-B
Ans:B.Interosseiandlumbricals
Hyperextensionofmetacarpophalangealjointandflexionofthe
interphalangealjointisduetopalsyoflumbricalsandinterossei
muscles.
TheactionofLumbricals:FlexionofMCP,ExtensionofIPjoint
TheactionofPalmarinterossei:Adductionoffingers
TheactionofDorsalinterossei:Abductionoffingers
Ref:BDC,7thedition,volume1,pgno.163.

8.Tumouroftheuncinateprocessofthe
pancreaswillcompresswhichartery
a)Portalvein
b)Superiormesentericartery
c)Inferiormesentericartery
d)Commonhepaticartery
CorrectAnswer-B
Ans:B.Superiormesentericartery
Thesuperiormesentericarterypassesanteriortotheuncinate
process
Posteriorly,theuncinateprocessisrelatedtoaorta.
Ref:BDC,7thedition,vol2,page-328.

9.Aboymetwithamotorbikeaccident.CT
brainshowsinjurytotheposteriorendof
thesuperiortemporalgyrus.Heislikelyto
sufferfrom

a)Fluentaphasia
b)Non-fluentaphasia
c)Conductionaphasia
d)Noneoftheabove
CorrectAnswer-A
Ans:A.Fluentaphasia
Fluentaphasia
LesionsintheposteriorportionoftheleftSTGwereassociatedwith
thelossoftheabilitytocomprehendandproducespokenwords
whicharecalledas"fluentaphasia"
BDC7thedition,volume4,pageno129.

10.A65-year-oldladypresentswitha
vascularinjurytotheinferiorfrontal
gyrus.Whichfunctionalareawould
mostlybeaffected

a)Visual
b)Auditoryss
c)Wernicke
d)Motorspeech
CorrectAnswer-D
Ans:D.Motorspeech
Motorspeechdefectisalsocalledapraxiaofspeech.
InjurytotheBroca'sarea/leftinferiorfrontalgyruscausesmotor
speechdefect.
BDC7thedition,volume4,pageno129.

11.Whereisthehighestoxygen
concentrationpresentsinfetalcirculation
a)SVC
b)IVC
c)Rightventricle
d)Aorta
CorrectAnswer-B
Ans:B.IVC
Highlyoxygenatedblood
fromtheplacentaiscarriedtothe
fetusbytheumbilicalvein,whichisshuntedtotheinferiorvena
cava.
NelsonTextbookofPediatrics20thEdition(Pageno2161)

12.WhenthevalueofV/qisinfinity,it
means?
a)NoO2goesfromalveolitobloodandnoco2goesfromblood
toalveoli
b)Deadspace
c)ThePO2ofalveolarairis159mmHgandPCO2is40mmHg
d)PartialpressureofO2andCO2areequal
CorrectAnswer-B
Ans:B.Deadspace
Deadspace
Anareawithventilationbutnoperfusion.
*V/Qundefined,thoughapproachinginfinity.
InterpretationsofV/Qratiovalues:
*Valueis0.8
-Ventilation-Perfusionmatching
*IfV/Qis>0.8?
-Thismeansmoreventilationthanperfusion.
*IfV/Qis<0.8?
-V/Qmismatch
Ref:ArvindArorareviewbookofphysiology(p.240-241)

13.Proteoglycanpresentintheglomerular
basementmembraneis?
a)Keratansulphate1
b)Keratansulphate2
c)Heparansulphate
d)Chondroitinsulphate
CorrectAnswer-C
Ans:c.Heparansulphate
Theglomerularmembrane(orthefiltrationbarrier)isthefiltration
surfacethroughwhichthefluidisfilteredoutfromtheblood.
Theglomerularmembrane(filtrationbarrier)comprises:
*Theglomerularcapillaryendothelium.
*Thebasementmembrane(basallamina).
*TheBowman'svisceralepithelium(podocytes).
*Importantconstituentsofglomerularmembraneinclude,
-Laminin
-Type4collagen
-Nidogen
-Proteoglycans-(Heparansulphate)
Ref:ArvindArorareviewbookofphysiology(p.371-372)

14.35yroldfemalewaswatchingTVforlong
hourswithhandsunderherhead.She
complainsoftinglingsensationoverher
arm.Whichtypeofnervefibersismost
likelytobeaffected?

a)Fibers
b)B-fibers
c)C-Fibers
d)Sympatheticnervefibers
CorrectAnswer-A
Ans:A.Fibers
A-
Features:
Functions:
High
delta Diameter-
Pain-
susceptibleto
1-5
-"Fast/Epicritic/First"pain. Pressure
Myelination-
-Sincefibersarerelatively
Somemyelination fast
Conduction-5-30 Temperature
Pressure
Touch
Ref:ArvindArorareviewbookofphysiology(p.58-59)

15.Thereflexinwhichthereisinhibitionof
gastricemptyingwhenthereisacidand
hypertonicsolutionintheduodenum?

a)Enterogastric
b)Gastroileal
c)Gastrocolic
d)Myenteric
CorrectAnswer-A
Ans:A.Enterogastric
Theenterogastricreflexisoneofthethreeextrinsicreflexesofthe
gastrointestinaltract,theothertwobeingthegastroilealreflexand
thegastrocolicreflex.
Theenterogastricreflexisstimulatedbyduodenaldistension.Itcan
alsobestimulatedbyapHof3-4intheduodenumandbyapHof
1.5inthestomach.
Themainneuralinfluenceingastricemptyingisthoughttobe
mediatedviaaninhibitorymechanismreferredtoasthe
enterogastricreflex.Fats,fattyacids,soaps,proteindigestive
products,acids,andhypertonicsolutionsinthesmallintestineinhibit
motility.
Ref:https://www.gastrojournal.org/article/S0016-5085(75)80295-
2/pdf


16.Inmultiplesclerosis,slowconductionof
motorandthesensorypathwaysisdue
to?

a)DefectinthenodeofRanvier
b)Lossofmyelinsheath
c)Leakingofsodiumchannels
d)Leakingcalciumchannels
CorrectAnswer-B
Ans:B.Lossofmyelinsheath
MultipleSclerosis(MS)isadisablingprogressiveneurological
disorder
ThepathophysiologyofMSresultsindisruptionorlossofaxonal
myelininthecentralnervoussystem(CNS),leadingtotheformation
ofscartissue(sclerosis).
Demyelinationproducesalterationsinsaltatoryconduction,slowed
conductionvelocity,andapredispositiontoconductionblock
Ref:https://www.physiology.org/doi/pdf/10.1152/japplphysiol.00460.2010

17.Whichofthefollowingclottingfactorinapatienton
Warfarintherapy,wouldhavedecreasedgamma
carboxyglutamateresidue?

a)Factor2
b)Factor11
c)Tissuefactor
d)Factor5
CorrectAnswer-A
Ans:A.Factor2
VitaminKisacofactorfortheenzymaticconversionofglutamicacid
(Glu)residuestogamma-carboxyglutamicacid(GLA)invitaminK-
dependentproteins,viatheendoplasmicreticulumresidentvitamin
K-dependentgamma-glutamylcarboxylase.
Thiscarboxylaseactivityisfoundinessentiallyallmammalian
tissues,anditsreactionproduct,Gla,hasbeenobservedinboth
vertebratesandinvertebrates;bothplayanimportantbiologicalrole
inproteinfunction.
VitaminK?dependentclottingfactordeficiency(VKCFD)isarare
autosomalrecessivebleedingdisorderthatoftenpresentswith
severehemorrhageduringinfancy.
Arareinheritedformofdefective-carboxylationresultinginthe
earlyonsetofbleedingwasfirstdescribedbyMcMillanandRoberts
in1966andsubsequentlyhasbeentermedvitaminK?dependent
clottingfactordeficiency
(VKCFD).
CombineddeficiencyofvitaminK?dependentclottingfactorsII,VII,
IX,andX(andproteinsC,S,andZ)isusuallyanacquiredclinical
problem,oftenresultingfromliverdisease,malabsorption,or
warfarinoverdose.

Patientsplasmashowedlessthan3%activityoffactorsII,VII,IX,
andX.

18.Bloodstoredincitrate-phosphate-dextroseisbetterfor
hypoxicpatientsthanacidic-citrate-dextrosebecause?
a)IthaslessP50
b)Itislessacidic
c)Thefallin2,3DPGisless
d)Noneoftheabove
CorrectAnswer-C
Ans:C.Thefallin2,3DPGisless
TheinfusionofACDbloodcausedP50and2,3-DPGconcentration
todecreasesignificantly.
Theinfusionofbloodstoredincitratephosphatedextrose(CPD)did
notsignificantlyincreasetheoxygenaffinity.
Tocompensatefortheincreasedoxygenaffinity,theremustbea
riseincardiacoutputormorelikelyadecreaseinvenousPO2.
ThetransfusionofCPDblood,therefore,ismorefavorableinterms
ofoxygensupply,particularlyinpatientswhohavehadcardiac
surgery.

19.Whichofthefollowingisreferredtoasthe"Windowofthe
limbicsystem"?
a)Hypothalamus
b)Amygdala
c)Hippocampus
d)Thalamus
CorrectAnswer-B
Ans.Bi.e.Amygdala.
*Theamygdalahasbeencalledthe"window"throughwhichthe
limbicsystemseestheplaceofthepersonintheworld.
*Theamygdalareceivesneuronalsignalsfromallportionsofthe
limbiccortex,aswellasfromtheneocortexofthetemporal,parietal,
andoccipitallobes,especiallyfromtheauditoryandvisual
associationareas.

20.Cerebralbloodflowisregulatedbyall
except:
a)Bloodpressure
b)ArterialPCO2
c)Potassiumions
d)A&C
CorrectAnswer-D
Ans.A&C.i.e.Bloodpressure&Potassiumions
Thetotalcerebralbloodflowisheldconstantinfaceofconsiderable
changesinthesystemicbloodpressure(60-150mmHg)".-
Principlesofmedicalphysiology.
"Cerebralbloodflowisnotaffectedoverafairlywidevariationin
arterialbloodpressure".-RKMarya.
Regulationofcerebralbloodflow:
l)Autoregulation:-Duetotheinherentpropertyofbloodvessels,
explainedbymyogenictheory.
2)Metabolicregulation:CBFvariesdirectlywithmetabolicactivity.
FactorsaffectingregulationarePCO2,PO2,K+,andadenosine.
AmongthisPCO2isthemostimportantone.
3)Sympatheticinnervation:-Importantonlyincaseswithvery
severeelevationofBBe.g.inverystrenuousexercises.
4)Intracranialpressure:-ByCushingreflex.
5)Others:Bloodviscosity.

21.Hepcidininhibits?
a)Absorptionofcobalamine
b)Transferofironintoenterocytes
c)Folicacidsynthesis
d)Respiratoryoxidase
CorrectAnswer-B
Ans.is'b'i.e.,Transferofironintoenterocytes
Hepcidinisanironmetabolismregulatoryhormonethatinhibitsiron
absorption(transferofironintoenterocytes).
Ascorbicacid(vitaminC)formssolublecomplexeswithiron
andreducesironfromtheferrictotheferrousstate,
therebyenhancingironabsorption.
Tannins,presentintea,forminsolublecomplexeswithiron
andloweritsabsorption.


22.Whichofthefollowingtechniqueisused
tostudycurrentflowacrossasingleion
channel?

a)Patchclamp
b)Voltageclamp
c)lontophoresis
d)Galvanometry
CorrectAnswer-A
Ans.a.Patch-clamp
Patch-clampisatechniquetorecordioncurrentflowthrough
asingleproteinchannel.
Thepatch-clamptechniqueisalaboratorytechniquein
electrophysiology,thatallowsthestudyofsingleormultiple
ionschannelsincells.
Thetechniquecanbeappliedtoawidevarietyofcellsbutis
especiallyusefulinthestudyofexcitablecellssuchasneurons,
cardiomyocytes,musclefibers,andpancreaticbetacells.
Itcanalsobeappliedtothestudyofbacterialionchannels
especiallypreparedgiantspheroplasts.
Inthepatch-clampexperiment,asmallpipetteiscarefully
maneuveredtosealoffaportionofacellmembrane.Thepipette
hasanelectrodebathedinanappropriatesolutionthatallowsforthe
recordingofelectricalchangesthroughanyporeinthemembrane.

23.A4-year-oldboyofafirst-degree
consanguineouscouplewasnotedbythe
parentstohavedarkeningoftheurineto
analmostblackcolorwhenitwasleft
standing.Hehasanormalsibling,and
therearenoothermedicalproblems.
Growthanddevelopmenttodateare
normal.Whichofthefollowingismost
likelytobeelevatedinthispatient?

a)Methylmalonate
b)Homogentisate
c)Phenylpyruvate
d)-Ketoisovalerate
CorrectAnswer-B
Ans:B.Homogentisate.
*Alkaptonuriaisararemetabolicdiseaseinvolvingadeficiencyin
homogentisicacidoxidase,andthesubsequentaccumulationof
homogentisicacidintheurine,whichturnsdarkuponstanding.
*Theelevationof
methylmalonate(duetomethylmalonylCoAmutasedeficiency),
Phenylpyruvate(duetophenylalaninehydroxylasedeficiency),
-ketoisovalerate(duetobranched-chain-ketoacid
dehydrogenasedeficiency),
Homocysteine(duetocystathionine-synthasedeficiency)
*Alloftheseareinconsistentwithahealthychildwithadarkeningof

theurine.
*Reflippincott'sIllustratedreviews,5thedition,AminoAcid
DegradationandSynthesis,Pg276.

24.Whichofthefollowingistrueabout
differentstructuresofprotein?
a)Secondarystructureisthethree-dimensionalstructureof
protein
b)Secondarystructureisstabilizedbydisulfidebonds
c)Primary,secondaryandtertiarystructuresdestroyedduring
denaturation
d)Secondaryandtertiarystructuredependsonthesequenceof
aminoacids
CorrectAnswer-D
Ans:D.Secondaryandtertiarystructuredependsonthe
sequenceofaminoacids
Explanation
Proteinsarearrangedinanyofthefollowingfourstructures
viz:
*Primary
Thesequenceofaminoacidsinaproteiniscalledtheprimary
structureoftheprotein.chain
*Secondary
-Thepolypeptidebackbonedoesnotassumearandomthree-
dimensionalstructure,butinsteadgenerallyformsregular
arrangementsofaminoacidsthatarelocatedneartoeachotherin
thelinearsequence.The-helix,-sheet,and-bend(-turn)are
examplesofsecondarystructuresfrequentlyencounteredin
proteins.
*Tertiary
-Itreferstothethree-dimensionalarrangementofapolypeptide
chainthathasassumeditssecondarystructure.Disulfidebonds

betweencysteineresiduesmaystabilizethetertiarystructure.
Proteindenaturationresultsintheunfoldinganddisorganizationof
theprotein'ssecondaryandtertiarystructures,whicharenot
accompaniedbyhydrolysisofpeptidebonds
*Quaternary
-Thequaternarystructurerequiresmorethanonepolypeptide
chain.Thesechainsassociatethroughnoncovalentinteractions.
Reflippincott'sIllustratedreviews,5thedition,Structureof
Protein,Pg13


25.Theinsulinglucagonratiodecreased.The
enzymeisactiveatthistime?
a)Glucokinase
b)Hexokinase
c)Phosphofructokinase
d)Glucose6phosphatase
CorrectAnswer-D
Ans.D.Glucose6phosphatase
Explanation
Lowinsulin:glucagonratio(IGR)stimulatesmobilizationofstored
nutrients,increasesglycogenolysisandgluconeogenesis,and
promotesthebreakdownofadiposetissueintofreefattyacidsand
glycerol.
DecreasesInsulin/glucagonratioshowsthefastingstage.
Glucokinase,Hexokinase,andphosphofructokinaseareglycolytic.
Onlygluconeogenicisglucose6phosphatase.
Ref-https://www.ncbi.nlm.nih.gov/pmc/articles/PMC4801814

26.Ochronosisisduetotheaccumulation
of?
a)Homogentisicacid
b)Homogentisicacid
c)Xanthurenate
d)Glyoxylate
CorrectAnswer-A
Ans:A.Homogentisicacid
Alkaptonuriaisanautosomalrecessiveconditionduetothe
deficiencyofhomogentisate1,2dioxidase.Thisresultsinthe
excretionofhomogentisicacidinurine.iscompatiblewithafairly
normallife.
Theonlyabnormalityistheblackeningofurineonstanding.
Homogentisicacidisoxidizedbypolyphenoloxidaseto
benzoquinoneacetate.Itisthenpolymerizedtoblackcolored
alkaptonbodies.
Bythe3rdor4thdecadeoflife,thepatientmaydevelopochronosis
(depositionofalkaptonbodiesinintervertebraldiscs,cartilagesof
nose,pinnaoftheear).Blackpigmentsaredepositedoverthe
connectivetissuesincludingjointcavitiestoproducearthritis.
Nospecifictreatmentisrequired.Butminimalproteinintakewith
phenylalaninelessthan500mg/dayisrecommended.
Ref-DMVasudevan-Textbookofbiochemistryformedicalstudents,
6thedn,AromaticAminoAcids,andAminoAcidurias,pg208.

27.Bilirubininserumcanbemeasuredby
a)VandenBerghreaction
b)Ehrlich'sReaction
c)Schlesinger'sReaction
d)Fouchet'sReaction
CorrectAnswer-A
Ans:A.VandenBerghreaction
BilirubinismostcommonlydeterminedbythevandenBergh
reaction,inwhichdiazotizedsulfanilicacidreactswithbilirubinto
formredazodipyrroles
Thesearemeasuredcolorimetrically.Inaqueoussolution,thewater-
soluble,conjugatedbilirubinreactsrapidlywiththereagent(within
oneminute),andissaidtobe"direct-reacting."Theunconjugated
bilirubin,whichismuchlesssolubleinaqueoussolution,reacts
moreslowly.
However,whenthereactioniscarriedoutinmethanol,both
conjugatedandunconjugatedbilirubinaresolubleandreactwiththe
reagent,providingthetotalbilirubinvalue.The"indirect-reacting"
bilirubin,whichcorrespondstotheunconjugatedbilirubin,is
obtainedbysubtractingthedirect-reactingbilirubinfromthetotal
bilirubin
Note:Innormalplasma,onlyabout4%ofthetotalbilirubinis
conjugatedordirect-reacting,becausemostaresecretedintobile.
Ref-Lippincott'sIllustratedreviews,5thedition,Conversionofamino
acidsinspecializedproducts,Pg285.

28.IfasampleofDNAifadenineis28%what
willbetheamountofCytosinepresent
a)23%
b)25%
c)46%
d)22%
CorrectAnswer-D
Ans:D.22%
ThebasesofonestrandofDNAarepairedwiththebasesofthe
secondstrandsothatadenineisalwayspairedwiththymineand
cytosineisalwayspairedwithguanine.Therefore,one
polynucleotidechainoftheDNAdoublehelixisalwaysthe
complementoftheother.Giventhesequenceofbasesonone
chain,thesequenceofbasesonthecomplementarychaincanbe
determined.
Note:ThespecificbasepairinginDNAleadstotheChargaffRule:
InanysampleofdsDNA,theamountofadenineequalstheamount
ofthymine,theamountofguanineequalstheamountofcytosine,
andthetotalamountofpurinesequalsthetotalamountof
pyrimidines.
Thebasepairsareheldtogetherbyhydrogenbonds:twobetweenA
andTandthreebetweenGandC(Figure29.5).Thesehydrogen
bonds,plusthehydrophobicinteractionsbetweenthestacked
bases,stabilizethestructureofthedoublehelix.
Ref-lippincott'sIllustratedreviews,5thedition,thestructureofDNA,
pg397

29.Whichofthefollowingvitaminathigher
dosescausescystoidmacularedema-
a)VitA
b)VitD
c)VitE
d)Niacin
CorrectAnswer-D
Ans:D.Niacin
Cystoidmacularedema(CME)isaconditionthatinvolvesthe
macula,causingpainlessvisionloss.
Niacin(nicotinicacid,vitaminB3,vitaminPP),onecomponentofthe
dietarysupplementtakenbythepatient,isavitaminpreparation
usuallyusedforthetreatmentoflipiddisorders
Fraunfelderetal.reportedthat3gormoreperdayofnicotinicacid
couldcausemanyocularsideeffectssuchasblurredvision,eyelid
edema,toxicamblyopia,proptosis,lossofeyelashesoreyebrow,
superficialpunctatekeratitis,andcystoidmacularedema,which
representsthemostseriousocularcomplications.Alltheseadverse
effectsarereversiblewithdiscontinuationofniacintherapy
Ref-CaseReportsinOphthalmologicalMedicine,Volume2013,
ArticleID713061,5page

30.TruestatementregardingHuntington's
choreais
a)Thereisalossoffunctiontypeofmutation
b)Itisanautosomalrecessive
c)Itisatrinucleotiderepeatexpansiontypeofdisorder
d)IncreasednumberofCAArepeats
CorrectAnswer-C
Ans:C.Itisatrinucleotiderepeatexpansiontypeofdisorder
Huntington'schoreaisanautosomaldominantdisorderwithan
increasednumberofCAGrepeats.Clinicallymanifestedas
involuntaryjerkymovements,mooddisturbancesandfinallysevere
dementia.
Ref-Langebiochemistryandgeneticsflashcards,Huntington's
disease,pg161

31.AdditionofwhichAminoAcidwill
increaseUVabsorption
a)Tryptophan
b)Leucine
c)Proline
d)Arginine
CorrectAnswer-A
Ans:A.Tryptophan
AminoAcidAbsorbUVLight

AminoAcidsthatabsorb250-290nm(Maximumat280nm)UVlight
aretryptophan,phenylalanine,tyrosine.
ThemaximumabsorptionofUVlightisbytryptophan.
Asitabsorbsultravioletlightabouttentimesmoreefficientlythan
phenylalanineortyrosine,tryptophanmakesthemajorcontribution
totheabilityofmostproteinstoabsorblightintheregionof280nm.
TryptophanandtyrosineabsorbUVatapproximately280nrn.
UVabsorptionspectroscopyisutilizedtomeasureprotein
concentration.TheabsorptionspectrumofaproteinintheUV
wavelengthrangeistheresultofabsorptionoflightbythearomatic
aminoacids(250-320nm),thedisulfidebonds(250-300nm)andthe
carbonylgroupofthepeptidebond(190-210).
Remember
AromaticAminoAcidsabsorbUVLights
Aminoacidsarecolourlessbecausetheydonotabsorbvisible
light.
Ref-Rebeccajames-selfassessmentandreviewofbiochemistry,
3rdedn,Chemistry,andMetabolismofAminoAcids,Pg7

32.V-RichestsourceofvitaminB12?
a)Meat
b)Greenleafyvegetables
c)Cornoil
d)Sunfloweroil
CorrectAnswer-A
Ans:A.Meat
*VitaminB12isnaturallyfoundinanimalproducts,includingfish,
meat,poultry,eggs,milk,andmilkproducts.
*Synonymsarecobalamin,extrinsicfactor(EF)ofCastleandanti
perniciousanemiafactor.VitaminB12iswater-soluble,heatstable
andredincolor.Itissatisfiedbyanyofthefollowinggroups:
cyanide,hydroxyl,adenosylormethyl.
*Cyanocobalamin-Oralpreparationsareinthisform.
*Hydroxycobalamin-Injectablepreparationsareinthisform.
*Adenosylcobalamin-Thisisthemajorstorageform,seeninthe
liver.
*Methylcobalamin-Thisisthemajorformseeninbloodcirculation
aswellasinthecytoplasmofcells.
*VitaminB12isgenerallynotpresentinplantfoods,butfortified
breakfastcerealsareareadilyavailablesourceofvitaminB12with
highbioavailabilityforvegetarians.
*RichsourceofvitaminB12
-Beef,liver,andchicken.
-Fishandshellfishsuchastrout,salmon,tunafish,andclams.
-Fortifiedbreakfastcereal.
-Low-fatmilk,yogurt,andcheese.
-Eggs.

Ref-DMVasudevan-Textbookofbiochemistryformedical
students,6thedn,Nutrition,pg404.

33.Whichaminoacidisusedtosynthesize
Nitricoxide?
a)Glycine
b)Arginine
c)Tyrosine
d)Threonine
CorrectAnswer-B
Ans:B.Arginine
Nitricoxideisformedfromargininebytheenzymenitricoxide
synthase(NOS).Itcontainsheme,FAD,FMN,NADPH,and
tetrahydrobiopterin.
Nitricoxidehasaveryshorthalf-life(3-4seconds).NOcombines
withoxygentoformNO2.Thesenitritesareexcretedthroughurine.
Reactingwithhemoglobin,NOisconvertedtoNO3;andnitratesare
alsoexcretedintheurine.TheverylowquantityofNOisexpelled
throughthelung.
Arginineisahighlybasic,semi-essentialaminoacid.Itisglucogenic
innature.

34.TrueAboutNoncompetitiveantagonist-
a)Kmremainssame,Vmaxdecreases
b)Kmremainssame,Vmaxdecreases
c)Kmdecreases,Vmaxincreases
d)Kmincreases,Vmaxincreases
CorrectAnswer-A
Ans:A.Kmremainssame,Vmaxdecreases
Noncompetitiveinhibitionoccurswhentheinhibitorandsubstrate
bindatdifferentsitesontheenzyme.Thenoncompetitiveinhibitor
canbindeitherthefreeenzymeortheEScomplex,thereby
preventingthereactionfromoccurring.
EffectonVmax:Noncompetitiveinhibitioncannotbeovercomeby
increasingtheconcentrationofthesubstrate.Thus,noncompetitive
inhibitorsdecreasetheapparentVmaxofthereaction.
EffectonKm:Noncompetitiveinhibitorsdonotinterferewiththe
bindingofsubstratetoenzyme.Thus,theenzymeshowsthesame
Kminthepresenceorabsenceofthenon-competitiveinhibitor.
EffectonLineweaver-Burkplot:Noncompetitiveinhibitionisreadily
differentiatedfromcompetitiveinhibitionbyplotting1/voversus1/[S]
andnotingthattheapparentVmaxdecreasesinthepresenceofa
non-competitiveinhibitor,whereasKmisunchanged
Ref-lippincott'sIllustratedreviews,5thedition,InhibitionofEnzyme
Activity,pg61.

35.Wernersyndromeassociatedwith
prematureagingiscausedduetoadefect
inwhichofthefollowing?

a)Telomerase
b)Caspase
c)DNAtopoisomerase
d)DNAhelicase
CorrectAnswer-D
Ans:D.DNAhelicase
Wernersyndromeisahumanautosomalrecessivedisorderthat
displayssymptomsofprematureaging,includingearlygrayingand
thinningofhair,wrinklingandulcerationofskin,atherosclerosis,
osteoporosis,andcataracts.
Inaddition,Wernersyndromepatientsexhibitanincreased
incidenceofdiabetesmellitustype2,hypertension,andarehighly
disposedtotheemergenceofbenignandmalignantneoplasms.
WernersyndromecausedbymutationoftheWRNgene,amember
oftheRecQDNAhelicasefamily.
Ref-https://www.ncbi.nlm.nih.gov/pmc/articles/PMC3237395/

36.Whichofthefollowingdietaryfiberis
insolubleinwater?
a)Pectin
b)Lignin
c)Hemicellulose
d)Cellulose
CorrectAnswer-B
Ans:B.Lignin
Theunavailableorindigestiblecarbohydratesinthedietarecalled
dietaryfiber.
Dietaryfiberisnecessarytomaintainthenormalmotilityofthe
gastrointestinaltract.Theyarechiefly-
Cellulose-Retainswaterinfeces,promotesperistalsis,increases
bowelaction
Lignin-Antioxidant,increasesbileacidexcretion,
hypocholesterolemic.
Pectins-PartiallyAbsorbwater,slowsesterifiedgastricemptying,
bindsbileacids,increasestheirexcretion
Hemi-cellulose-Retainswaterinfeces,cellulose,increasesbileacid
anduronicacidexcretion.
Ref-DMVasudevan-Textbookofbiochemistryformedical
students,6thedn,EnergyMetabolismandNutrition.Pg435.

37.AccordingtoNCEP-ATPIII,whichamong
thefollowinghavenotbeenincludedin
metabolicsyndrome?

a)HighLDL
b)Hypertriglyceridemia
c)CentralObesity
d)Hypertension
CorrectAnswer-A
Answer-A.HighLDL
Metabolicsyndromereferstotheco-occurrenceofseveralknown
cardiovascularriskfactors,includinginsulinresistance,obesity,
atherogenicdyslipidemia,andhypertension.Theseconditionsare
interrelatedandshareunderlyingmediators,mechanisms,and
pathways.
CriteriafortheDiagnosisofMetabolicSyndrome
Elevatedwaistcircumference:(Formen>90cmandforwomen,>80
cm).
Elevatedtriglycerides:>150mg/dL
ReducedHDL("good")cholesterol:Formen,<40mg/dL;forwomen,
<50mg/dL
Elevatedbloodpressure:>130/85mmHg
Elevatedfastingglucose:>100mg/dL
Insulinresistance(hyperinsulinemia)
Additionalparametersinclude:coagulationabnormalities,
hyperuricemia,microalbuminurianon-alcoholicsteatohepatitis
(NASH)andincreasedCRP.
Diagnosisismade,ifany3outofthe5criteriagivenabove.

Ref-DMVasudevan-Textbookofbiochemistryformedical
students,6thedn,Clinical,andAppliedBiochemistry,pg286

38.Whichofthefollowingisthebasisforthe
intestine-specificexpressionof
apoproteinB-48?

a)DNArearrangementandloss
b)DNArearrangementandloss
c)RNAalternativesplicing
d)RNAediting
CorrectAnswer-D
Ans:D.RNAediting.
TheproductionofapoB-48intheintestineandapoB-100inliveris
theresultofRNAeditingintheintestine,whereasensecodonis
changedtoanonsensecodonbypost-transcriptionaldeaminationof
CtoU.DNArearrangementandtransposition,aswellasRNA
interferenceandalternatesplicing,doaltergeneexpression,butare
notthebasisofapoB-48tissue-specificproduction.
Reference-Lippincott'sIllustratedReviews:Biochemistry,5th
edition,RegulationofGeneExpression,Pg464.

39.A30-year-oldmalecamewithcomplaints
ofswellingaroundthekneejoint.
Histopathologicalexaminationofthe
swellingdemonstratedmanygiantcells
interspersedwithmononuclearcells.
Whatistheprobablediagnosis

a)Osteosarcoma
b)Ewing'ssarcoma
c)Giantcelltumour
d)Chondrosarcoma
CorrectAnswer-C
Correctans:C
GCTsarelargeandred-brownandoftenshowcysticdegeneration.
Theyarecomposedofuniformovalmononuclearcellsandscattered
osteoclast-typegiantcellscontaining100ormorenuclei.
Mitoticfiguresaretypicallyfrequent.
Necrosis,hemorrhage,andreactiveboneformationalsoare
commonlypresent.
RefRobbin'sbasicsofpathology9theditionpageno.781

40.30yearsoldcamewithcomplaintsofeasy
fatigability,exertionaldyspnea,and
weightloss.Shealsocomplainsof
frequentfalls.physicalexamination
revealedtherewasabilateraldecreasein
vibrationsense.Herhemoglobinlevels
were8.2g%.Shewastreatedwithfolate.
Heranemiaimprovedbutneurological
symptomsworsened.Whichofthe
followingisthemostprobablereasonfor
hercondition?

a)Folatenotabsorbed
b)Unmaskedpyridoxinedeficiency
c)DeficiencyoffolatereductaseinCNS
d)FolatetherapycausedrapiduseofVitB12storesaggravating
symptoms
CorrectAnswer-D
Ans:D.FolatetherapycausedrapiduseofVitB12stores
aggravatingsymptoms
VitaminB12isrequiredfortherecyclingoftetrahydrofolate,theform
offolatethatisneededforDNAsynthesis.
Inkeepingwiththisrelationship,theanemiaofvitaminB12
deficiencyisreversedwiththeadministrationoffolate.Bycontrast,
folateadministrationdoesnotpreventandmay,infact,worsenthe

neurologicsymptoms


41.A45-year-oldmanwhoisachronic
smokercametotheclinicwitha
complaintofcough.Thephysician
examinesthepatientandtakesabiopsy,
thepictureinthebiopsywasasthe
descriptionbelow.Whichofthefollowing
cellularchangeshashappenedtothis
patient?

a)Hyperplasia
b)Dysplasia
c)Metaplasia
d)CIN3
CorrectAnswer-B
Ans:C.Metaplasia
Metaplasiaisareversiblechangeinwhichoneadultcelltype
(epithelialormesenchymal)isreplacedbyanotheradultcelltype.

(epithelialormesenchymal)isreplacedbyanotheradultcelltype.
squamousmetaplasiaoftherespiratoryepitheliumoftencoexists
withlungcancerscomposedofmalignantsquamouscells.
cigarettesmokinginitiallycausessquamousmetaplasia,
Ref:Robbin'sbasicsofpathology9theditionpageno.5

42.A30-year-oldwomanpresentswith
thyroidswelling.Oninvestigations,her
TSHlevelsarefoundtobeelevated.
Postoperativereportsshowed
lymphocyticinfiltrationandhurthlecells.
Amostprobablediagnosisis?

a)Gravesdisease
b)Hashimoto'sthyroiditis
c)Follicularcarcinoma
d)Medullarycarcinomathyroid
CorrectAnswer-B
Ans:B.Hashimoto'sthyroiditis
Hashimotothyroiditisisthemostcommoncauseofhypothyroidism
whichischaracterizedbygradualthyroidfailuresecondarytothe
autoimmunedestructionofthethyroidgland.
Itismostprevalentbetweentheagesof45and65yearsandis
morecommoninwomenthaninmen,ultimatelyleadingtoavarying
degreeoffibrosisandthyroidenlargement.
ThethyroidusuallyisdiffuselyandsymmetricallyenlargedThecut
surfaceispaleandgray-taninappearance,andthetissueisfirm
andsomewhatfriable.
Microscopicexaminationrevealsthewidespreadinfiltrationofthe
parenchymabyamononuclearinflammatoryinfiltratecontaining
smalllymphocytes,plasmacells,andwell-developedgerminal
centers
.
Thethyroidfolliclesareatrophicandarelinedinmanyareasby

epithelialcellsdistinguishedbythepresenceofabundant
eosinophilic,granularcytoplasm,termedHurthle,oroxyphilcells.

43.Whichofthefollowingwillbeseenon
cardiacbiopsyofapatientwhohadapost
MIreperfusioninjury?

a)Wavinessoffibres
b)Eosinophiliccontractionbands
c)Neutrophilsincardiaccells
d)Swellingofcells
CorrectAnswer-B
Ans:B.Eosinophiliccontractionbands
Microscopically,irreversiblydamagedmyocytessubjectto
reperfusionshowcontractionbandnecrosis;
Inthispathologicprocess,intenseeosinophilicbandsof
hypercontractedsarcomeresarecreatedbyaninfluxofcalcium
acrossplasmamembranesthatenhancesactin-myosin
interactions.

44.Lossoffootprocessisclassicalincase
of?
a)Membranousglomerulitis
b)Segmentalglomerulosclerosis
c)IgAnephropathy
d)Diabeticnephropathy
CorrectAnswer-B
Ans:B.Segmentalglomerulosclerosis
Focalsegmentalglomerulosclerosis(FSGS)ischaracterizedby
sclerosis,hyalinosis,adhesions/synechiaeformation,resultingin
segmentalobliterationofglomerularcapillaries.OnEM,footprocess
effacementisthepredominantfindingwithoutsignificantbasement
membraneabnormalities.Immunofluorescenceshowsstainingfor
IgMandC3inscleroticareas.Juxtamedullarynephronsareaffected
firstandhenceinadequatesamplingmaymissfocallesions.
https://www.ncbi.nlm.nih.gov/books/NBK532272/

45.Whichofthefollowingfactorsplaya
majorroleintheinitiationofthrombus
formation?

a)Vasoconstriction
b)Coagulationcascadeactivation
c)Plateletsactivation
d)Endothelialinjury
CorrectAnswer-D
Ans:D.Endothelialinjury
Thethreeprimaryabnormalitiesthatleadtothrombusformation
(calledVirchow'striad)are
(1)endothelialinjury,
(2)stasisorturbulentbloodflow,and
(3)hypercoagulabilityoftheblood.
Ref:
robbins9theditionchapter4

46.A33-year-oldmanpresentswitha5-week
historyofcalfpain,swelling,andlow-
gradefever.Serumlevelsofcreatinine
kinaseareelevated.Amusclebiopsy
revealsnumerouseosinophilsalsoshe
hadperipheralbloodeosinophilia.Which
ofthefollowinginterleukinsisprimarily
responsiblefortheincreasein
eosinophilsinthispatient?

a)IL2
b)IL4
c)IL1
d)IL6
CorrectAnswer-B
Ans:B.IL4

47.WhichofthefollowingistrueaboutPAN?
a)Microscopyshowsfibrinoidnecrosisinlargearteries
b)ANCAispositive
c)30%ofpeoplehaveHbsAgpositive
d)Patienthavehypogammaglobulinemia
CorrectAnswer-C
Ans:C.30%ofpeoplehaveHbsAgpositive
Polyarteritisnodosa(PAN)isanecrotizingvasculitisinvolvingsmall
andmedium-sizedmusculararteriesofmultipleorgansandtissues.
Thediseaseoccursmorecommonlyinadultmalesthanfemales.
Mostcommonlyaffectedorgans,indescendingorderoffrequencyof
involvement,arethekidneys,heart,gastrointestinaltract,kidneys,
andmuscle.
Theconditionresultsfromimmunologicresponsetoanidentifiable
antigenthatmaybebacteria(e.g.streptococci,staphylococci,
mycobacteria),viruses(e.g.hepatitisBvirus,influenzavirus,CMV),
malarialparasite,certaindrugs,andchemicals.
ThereisnoassociationwithANCA,[7]butabout30%ofpeoplewith
PANhavechronichepatitisBanddepositscontainingHBsAg-
HBsAbcomplexesinaffectedbloodvessels,indicatinganimmune
complex-mediatedcauseinthatsubset.InfectionwiththeHepatitis
CvirusandHIVareoccasionallydiscoveredinpeopleaffectedby
PAN.
Leukocytoclasticvasculitis,characterizedbyfibrinoidnecrosiswith
neutrophilicinfiltrateinthevesselwall.Manyoftheneutrophilsare
fragmented.Thisformisfoundinvasculitiscausedbydepositsof
immunecomplexes.

48.A23-year-oldmalepresentedwitha
historyoffatigueandtiredness.On
investigation,hewasfoundtohaveHb
valuesof9gm%,MCVof101FL.
peripheralsmearexaminationshowed
microcyticRBCandhypersegmented
neutrophils.Whichismostprobable
etiology

a)Leadpoisoning
b)Irondeficiencyanemia
c)Chronicalcoholism
d)Hemolyticanemia
CorrectAnswer-A
Ans:A.Leadpoisoning
Findings
MicrocyticAnemia-IronDeficiency
HypersegmentedNeutrophils-B12andFolicAcidDeficiency
Young&NoH/oOfAlcoholConsumption[Patientcamewitha
historyofFatigueandTiredness]-ChronicAlcoholismcanberuled
out
HemolyticAnemia-maydemonstrateredbloodcellfragments
calledschistocytes,redbloodcellsthatlooklikespheres
(spherocytes),and/orredbloodcellsmissingsmallpieces(bite
cells).Anincreasednumberofnewlymaderedbloodcells
(reticulocytes)mayalsobeasignofbonemarrowcompensationfor
anemia.

anemia.
SoMostProbableAnswerwouldbeLeadPoisoning[Tiredness&
Fatigue]

49.Whatisthemainfeatureofchemotaxisas
observedinwhitebloodcells?
a)Increasedrandommovementofneutrophils
b)Increasedadhesivenesstointima
c)Increasedphagocytosis
d)Unidirectionallocomotionofneutrophils
CorrectAnswer-D
Ans:D.Unidirectionallocomotionofneutrophils
Chemotaxisisdefinedasaunidirectionalmovementofleukocytes
uptoaconcentrationgradientofchemotacticmolecules.All
granulocytes,monocytesandtoalesserextentlymphocytes
respondtochemotacticstimuliwithvaryingratesofspeed.
Chemoattractantsareexogenousorendogenous.Exogenous
agentsarebacterialproducts.Endogenousmediatorsare:
ComponentsofcomplementpathwayC5a
ProductsoflipoxygenasepathwayLTB4
Cytokinesparticularlythoseofchemokinefamilies.

50.Whichofthefollowinganticancerdrugs
arecompetitiveinhibitorsoftyrosine
kinase?

a)Imatinibandsunitinib
b)Letrozole
c)Bicalutamide
d)Fulvestrant
CorrectAnswer-A
Ans.is'a'i.e.,Imatinibandsunitinib
Moleculartargetedagents
Tyrosinekinaseinhibitors
CompetitiveinhibitorsImatinib,Nilotinib,Sunitinib,Dasatinib,
Erlotinib,Gefitinib,Lapatinib,Sorafenib(Rememberallendwith
'nib').
MonoclonalantibodiesCetuximab,panitumumab.
HER2/neu(ERBB2)inhibitorsMonoclonalantibody-Trastuzumab.
TargetedantibodyGemtuzumab(antiCD-33),Rituximab(anti-
CD20),Alemtuzumab(antiCD-52).
Vascularendothelialgrowthfactor(VEGF)inhibitorMonoclonal
antibody-Bevacizumab.
ProteosomeinhibitorsBortezomib.
HistonedeacetylaseinhibitorVorinostat
DNA-methyltransferaseinhibitor5-azacytidine,2-deoxy-5
azacytidine.
All-trans-retinoicacid.
Biologicalresponsemodifier-RecombinantIL-2(aldesleukin,
denileukin).


51.Whichisnotaprokineticagent?
a)Dopamineantagonist
b)5HT4agonist
c)Macrolides
d)Diphenylmethane
CorrectAnswer-D
Ans:D.Diphenylmethane
Prokineticdrugs
:
Thesearedrugsthatpromotegastrointestinaltransitandspeed
gastricemptyingbyenhancingcoordinatedpropulsivemotility.
Thisexcludestraditionalcholinomimeticsandanti-ChEswhich
producetonicandlargelyuncoordinatedcontraction.
Drugsincluded:
Metoclopramide,
Domperidone,
Cisapride,
Mosapride,
Itopride
Ref:K.D.Tripathi7thEdition.Page663-665

52.Whichofthefollowingdrugsactby
inhibitingDNAreplication?
a)6Mercaptopurine
b)ActinomycinD
c)MitomycinC
d)Asparaginase
CorrectAnswer-A
Ans:A.6Mercaptopurine
6-MercaptopurineactsbyinhibitingDNAreplication.
Antimetabolites:
TheseareanaloguesrelatedtothenormalcomponentsofDNAorof
coenzymesinvolvedinthenucleicacidsynthesis.
Theycompetitivelyinhibittheutilizationofthenormalsubstrateor
getthemselvesincorporatedformingdysfunctionalmacromolecules.
Includes:
Folateantagonist-Methotrexate(Mtx)
Purineantagonist-6-Mercaptopurine(6-MP)
Pyrimidineantagonist-5-Fluorouracil(5-FU)
Purineantagonists
Mercaptopurine(6-MP)andthioguanine(6-TG).
Thesearehighlyeffectiveantineoplasticdrugs.
Aftersynthesisinthebodytothecorrespondingmono
ribonucleotides,theyinhibittheconversionofinosine
monophosphatetoadenineandguaninenucleotidesthatarethe
buildingblocksforRNAandDNA.
Thereisalsofeedbackinhibitionofdenovopurinesynthesis.
TheyalsogetincorporatedintoRNAandDNAwhichare
dysfunctional.

Ref:K.D.Tripathi7thEdition.Page.858,862?863

53.DOCforOnychomycosis?
a)Terbinafine
b)Fluconazole
c)Nystatin
d)Itraconazole
CorrectAnswer-A
Ans:A.Terbinafine
Onychomycosis(afungalinfectionofthenail,usuallycausedbya
dermatophyte).
Onychomycosisismoredifficulttotreatthanmostdermatophytosis
becauseoftheinherentslowgrowthofthenail.
Olderantifungalagents(ketoconazoleandgriseofulvin)are
unsuitableforonychomycosisbecauseoftheirrelativelypoor
efficacyandpotentialadverseeffects.
Threerecentlydevelopedantimycoticagents(fluconazole,
itraconazole,andterbinafine)offerhighcureratesandgoodsafety
profiles.
Ref:https://www.ncbi.nlm.nih.gov/pmc/articles/PMC88888/

54.Theophyllinebywhatmechanismcauses
diuresis?
a)PDE3inhibition
b)PDE4inhibition
c)Beta2agonistaction
d)AdenosineA1receptorantagonism
CorrectAnswer-D
Ans:D.AdenosineA1receptorantagonism
Themildlydiureticactionsofbothmethylxanthinesaremainlythe
resultofinhibitionoftubularfluidreabsorptionalongtherenal
proximaltubule.
Basedupontheuseofspecificadenosinereceptorantagonistsand
theobservationofacompletelossofdiuresisinmicewithtargeted
deletionoftheA1ARgene,transportinhibitionbymethylxanthinesis
mediatedmainlybyantagonismofadenosineA1receptors(A1AR)
intheproximaltubule.
Methylxanthinesareweakrenalvasodilators,andtheyactas
competitiveantagonistsagainstadenosine-inducedpreglomerular
vasoconstriction.
Caffeineandtheophyllinestimulatethesecretionofreninby
inhibitionofadenosinereceptorsandremovalofthegeneral
inhibitorybrakefunctionofendogenousadenosine.
Sinceenhancedintrarenaladenosinelevelsleadtoreduced
glomerularfiltrationrateinseveralpathologicalconditions
theophyllinehasbeentestedforitstherapeuticpotentialintherenal
impairmentfollowingtheadministrationofnephrotoxicsubstances
suchasradiocontrastmedia,cisplatin,calcineurininhibitorsor
followingischemia-reperfusioninjury.

Ref:https://www.ncbi.nlm.nih.gov/pubmed/20859805

55.Whichofthefollowingantimicrobials
shouldnotbegiventoachronic
asthmaticpatientmanagedon
theophyllinetherapy?

a)Erythromycin
b)Amoxicillin
c)Cefotaxime
d)Cotrimoxazole
CorrectAnswer-A
Ans:A.Erythromycin
Drugswhichinhibittheophyllinemetabolismandincreaseitsplasma
levelareErythromycin,Ciprofloxacin,Cimetidine,Oral
contraceptives,Allopurinol;
Thedoseshouldbereducedto2/3.
Ref:K.D.Tripathi7thEdition.Page.226-227

56.DOCofprophylaxisformotionsickness?
a)Promethazine
b)Prochlorperazine
c)Metoclopramide
d)Itopride
CorrectAnswer-A
Ans:A.Promethazine
Motionsickness:

Antiemeticswiththeanticholinergic-antihistaminepropertyarethe
firstchoicedrugsformotionsickness.
Antidopaminergicandanti-HT3drugsarelesseffective.
Allanti-motionsicknessdrugsactbetterwhentaken??1hour
beforecommencingthejourney.
Oncesicknesshasstarted,itismoredifficulttocontrol;higher
doses/parenteraladministrationmaybeneeded.
DrugsIncluded:
Promethazine,diphenhydramine,dimenhydrinate:
Thesedrugsaffordprotectionofmotionsicknessfor4?6hours.
Promethazinetheoclate:
Thissaltofpromethazinehasbeenspeciallypromotedasan
antiemetic,buttheactiondoesnotappeartobesignificantly
differentfrompromethazineHCl.

57.Whichofthefollowingantihypertensive
drugisavoidedinpatientswithhigh
serumuricacidlevels?

a)Hydrochlorothiazide
b)Enalapril
c)Prazosin
d)Atenolol
CorrectAnswer-A
Ans:A.Hydrochlorothiazide
Long-termuseofhigherdosethiazidesinhypertensionhascaused
ariseinblooduratelevels.
Ref:K.D.Tripathi7thEdition.Page.585

58.Mechanismofresistancetopenicillinsvia
beta-lactamaseis
a)Alteredpenicillin-bindingproteins
b)Drugefflux
c)Breaksdrugstructure
d)Alterationin50Sribosomestructure
CorrectAnswer-C
Ans:C.Breaksdrugstructure
-lactamasesareproducedbystaphylococci,Haemophilus,
gonococci,etc.whichinactivatepenicillinG.
The-lactamasesmaybepresentinlowquantitybutstrategically
locatedperiplasmically(asingram-negativebacteria)sothatthe
drugisinactivatedsoonafterentry,ormaybeelaboratedinlarge
quantities(bygram-positivebacteria)todiffuseintothemediumand
destroythedrugbeforeentry.
Beta-lactamantibioticssharethestructuralfeatureofabeta-lactam
ring.
Thisfeatureisresponsiblefortheinhibitionofbacterialcellwall
synthesis.
Thetargetmoleculesarepeptidoglycancross-linkingenzymes(e.g.
transpeptidasesandcarboxypeptidases)whichcanbindbeta-lactam
antibiotics(penicillin-bindingproteins,PBP).
Bacterialcelldeathisinitiatedbybeta-lactamantibiotic-triggered
releaseofautolyticenzymes.
Incontrasttogram-positivebacteria(absenceofanouter
membrane),theantibiotichastopenetratethroughporinsofthe
outermembraneofgram-negativebacteriabeforetouchingPBP's.
Bacterialresistancetobeta-lactamantibioticsincludesmodification

ofporins(permeabilitybarrier)andoftargets(lowaffinityofPBP's
forthedrug),productionofinactivatingenzymes(beta-lactamases)
andinhibitionofreleaseofautolyticenzymes.
Ref:K.D.Tripathi7thEdition.Page.717?720
ncbi.nlm.nih.gov/pubmed/8314292


59.Apatientonlithiumtherapydeveloped
hypertension.HewasstartedonThiazide
forhypertension.Afterafewdays,he
developedcoarsetremorsandother
symptomssuggestiveoflithiumtoxicity.
Whatistheprobablemechanismof
interaction?

a)Thiazideincreasesthetubularreabsorptionoflithium
b)Thiazideinhibitsthemetabolismoflithium
c)Thiazidesactasanaddonthedrugtolithium
d)Alloftheabove
CorrectAnswer-A
Ans:A.Thiazideincreasesthetubularabsorptionoflithium
DrugInteraction:
Diuretics+Lithium=Decreasedexcretion--riseinLi+level-
toxicity;
Diuretics(thiazide,furosemide)bycausingNa+losstopromote
proximaltubularreabsorptionofNa+aswellasLi+plasma
levelsoflithiumrise.Potassium-sparingdiureticscausemilderLi+
retention.
Management:Reducedoseoflithiumandmonitorlevel.
Ref:K.D.Tripathi7thEdition.Page.449,932

60.Whichdrugactsviathetyrosinekinase
receptor?
a)Insulin
b)TSH
c)LH
d)MSH
CorrectAnswer-A
Ans:A.Insulin
Insulinactsonspecificreceptorslocatedonthecellmembraneof
practicallyeverycell,buttheirdensitydependsonthecelltype:liver
andfatcellsareveryrich.
Theinsulinreceptorisareceptortyrosinekinase(RTK)whichisa
heterotetramericglycoproteinconsistingof2extracellularand2
transmembranesubunitslinkedtogetherbydisulfidebonds.
Ref:K.D.Tripathi7thEdition.Page.261


61.Pegloticaseisusedforthetreatmentofan
Ankylosingspondylosis
a)ReactiveArthritis
b)CPPD
c)Chronictophaceousgout
d)RefractoryRheumatoidarthritis
CorrectAnswer-C
Ans:C.Chronictophaceousgout
Pegloticase:
Pegloticaseisamedicationforthetreatmentofthesevere,
treatmentofrefractorychronicgout.
Itisathird-linetreatmentinthoseinwhomothertreatmentsarenot
tolerated.
RefGoutandothercrystalarthropathiesbyRobertTerkettaud]

62.Fluoroquinolonecontraindicatedinliver
diseaseis
a)Levofloxacin
b)Pefloxacin
c)Ofloxacin
d)Lomefloxacin
CorrectAnswer-B
Ans:B.Pefloxacin
Pefloxacinhaslongert?:accumulatesonrepeateddosing
achievingplasmaconcentrationstwiceashighasafterasingle
dose.
Becauseofthis,itiseffectiveinmanysystemicinfectionsaswell.
Thedoseofpefloxacinneedstobereducedinliverdisease,butnot
inrenalinsufficiency.
Ref:K.D.Tripathi7thEdition.Page.713

63.Atahighaltitudeof3000m,aperson
complainsofbreathlessness.Allofthe
followingcanbeusedforthe
managementofthispersonexcept?

a)Intravenousdigoxin
b)Oxygensupplementation
c)Immediatedescent
d)Acetazolamide
CorrectAnswer-A
Ans:A.Intravenousdigoxin
ManagementofAMSfollowsthreeaxioms:a)furtherascentshould
beavoideduntilthesymptomshaveresolved,b)patientswithno
responsetomedicaltreatmentshoulddescendtoaloweraltitude
andc)ifandwhenHACOissuspected,patientsshouldurgently
descendtoaloweraltitude.
Descentandsupplementaryoxygenarethetreatmentsofchoice
andforsevereillness,thecombinationprovidesoptimaltherapy.
Remarkably,adescentofonly500to1000musuallyleadsto
resolutionofacutemountainsicknesswhilehigh-altitudecerebral
edema(HACO)mayrequirefurtherdescent.Simulateddescentwith
portablehyperbaricchambers,nowcommonlyavailableinremote
locations,arealsoeffective.
Medicaltherapybecomescrucialwhenthedescentisnot
immediatelypossible.
Ref:https://www.ncbi.nlm.nih.gov/pmc/articles/PMC4923381/#!po=31.2500

64.ApatientwithdiabetesandCOPD
developedpostoperativeurinary
retention.Whichofthefollowingdrugs
canbeusedforshorttermtreatmentto
relievethesymptomsofthisperson?

a)Bethanechol
b)Methacholine
c)Terazosin
d)Tamsulosin
CorrectAnswer-A
Ans:A.Bethanechol
Bethanechol
isapreferreddruginthetreatmentofpostpartum
andpostoperativenonobstructiveurinaryretention,anditalso
cancounteractbladderdysfunctionoftenseenwithphenothiazines
andtricyclicantidepressants.
Itcanaffordsymptomaticreliefincongenitalmegacolonand
gastroesophagealrefluxbutisrarelyusedforthese.
Ref:https://www.pdr.net/drug-summary/Urecholine-
bethanechol-chloride-801,
K.D.Tripathi7thEdition.Page.104


65.Drugofchoiceforinvasiveaspergillosis
is
a)Posaconazole
b)Voriconazole
c)LiposomalAMB
d)Caspofungin
CorrectAnswer-B
Ans:B.Voriconazole
ThepreferredtreatmentofprimaryIAisvoriconazole,whichhas
beenfoundtobesuperiortoamphotericinB.
Azolesinterferewiththesynthesisofergosterolfoundinthefungal
cellmembrane,whereaspolyenes--suchasamphotericinB--
interferewithergosterolfunction.
Anechinocandinthatdisruptsfungalcellwallsynthesis?
caspofungin--anditraconazolehavebeenapprovedforsalvage
therapyofIA.
Ref:https://www.ncbi.nlm.nih.gov/pmc/articles/PMC4200583/

66.Whichofthefollowingdrugsactsby
inhibitingthetranscriptionofDNAto
RNA?

a)Rifampicin
b)Nitrofurantoin
c)Ciprofloxacin
d)Novobiocin
CorrectAnswer-A
Ans:A.Rifampicin
OverallinhibitionofRNAsynthesisbyrifampiciniscausedbya
destabilizingeffectonthebindingoftheintermediate
oligonucleotidestotheactiveenzyme-DNAcomplex.
RifampicinitselfcanonlyinteractspecificallywithRNApolymerase
iftheenzymeisfreeorinabinarycomplexwithDNA.
Ref:https://www.ncbi.nlm.nih.gov/pmc/articles/PMC327649/

67.Apatientwasrecentlystartedon
Fluphenazine.Afewweekslater,he
developedtremors,rigidity,bradykinesia,
andexcessivesalivation.Thefirstlineof
managementforthispatientis

a)Selegiline
b)Trihexyphenidyl
c)Pramipexole
d)Amantadine
CorrectAnswer-B
Ans:B.Trihexyphenidyl
*Fluphenazinemayblocktheeffectsofagentsusedtotreat
Parkinson'sdiseasesuchaslevodopa/carbidopa.
*Trihexyphenidylisanantispasmodicdrugthatexertsadirect
inhibitoryeffectontheparasympatheticnervoussystem.
-Italsohasarelaxingeffectonsmoothmuscle.
-ItisindicatedinallformsofParkinsonism.
*Trihexyphenidylworksasananticholinergicandisusedforthe
treatmentoftremors,spasms,stiffness,andweakmusclecontrol
seeninpatientswithParkinson'sdisease.
-Itcanalsobeusedforthepreventionortreatmentofsimilar
muscularconditionswhicharecausedbycertaincentralnervous
system(CNS)drugssuchasfluphenazine,haloperidol,
chlorpromazine.
Ref:https://www.ncbi.nlm.nih.gov/books/NBK519488/,https://www.ncbi.nlm.nih.gov/books/NBK519488/

68.Apersonwasgivenamusclerelaxantthat
competitivelyblocksnicotinicreceptors.
Whichofthefollowingdrugsisusedfor
reversalofmusclerelaxationafter
surgery?

a)Neostigmine
b)Carbachol
c)Succinylcholine
d)Physostigmine
CorrectAnswer-A
Ans:A.Neostigmine
Neostigmine
hasbeentraditionallyusedastheagentofchoice
toreverseNeuromuscularBlockade(NMB)aftermuscleparalysis
duringgeneralanesthesia.
Anticholinesterases(neostigmine)aregenerallyusedtoreversethe
effectsofneuromuscularblockingagents.
Ref:https://www.hindawi.com/journals/cria/2017/8197035/

69.Zero-orderkineticsisotherwiseknownassaturationkinetics.Itisindependent
of:
a)Plasmaconcentration
b)Clearance
c)Volumeofdistribution
d)Halflife
CorrectAnswer-A
Ans.A.Plasmaconcentration
Inzero-orderkineticsorsaturationkinetics,eliminationmechanisms
becomesaturatedandunabletoprocessmoredrugswhendrug
concentrationsrise.
Consequently,fordrugsthatareeliminatedbyzero-order
kinetics,aconstantamountofdrugiseliminatedperunittime
regardlessofdrugplasmaconcentration.


70.Whichofthefollowinganticancerdrugs
arecompetitiveinhibitorsoftyrosine
kinase?

a)Imatinibandsunitinib
b)Letrozole
c)Bicalutamide
d)Fulvestrant
CorrectAnswer-A
Ans.is'a'i.e.,Imatinibandsunitinib
Moleculartargetedagents
Tyrosinekinaseinhibitors
CompetitiveinhibitorsImatinib,Nilotinib,Sunitinib,Dasatinib,
Erlotinib,Gefitinib,Lapatinib,Sorafenib(Rememberallendwith
'nib').
MonoclonalantibodiesCetuximab,panitumumab.
HER2/neu(ERBB2)inhibitorsMonoclonalantibody-Trastuzumab.
TargetedantibodyGemtuzumab(antiCD-33),Rituximab(anti-
CD20),Alemtuzumab(antiCD-52).
Vascularendothelialgrowthfactor(VEGF)inhibitorMonoclonal
antibody-Bevacizumab.
ProteosomeinhibitorsBortezomib.
HistonedeacetylaseinhibitorVorinostat
DNA-methyltransferaseinhibitor5-azacytidine,2-deoxy-5
azacytidine
All-trans-retinoicacid.
Biologicalresponsemodifier-RecombinantIL-2(aldesleukin,
denileukin)


71.Whichofthefollowingisthelawonchild
sexualabuseinIndia?
a)ChildSexualAbusePreventionAct
b)ProtectionOfChildrenfromSexualOffencesAct(POCSO)
c)ChildWelfareAct
d)SexualOffencesAct
CorrectAnswer-B
Ans:B.POSCOAct
TheProtectionofChildrenfromSexualOffences(POSCO)Act,
2012,2019
ItisapplicabletothewholeofIndia.
Itdefinesachildasanypersonbelowtheageof18yearsand
providesprotectiontoallchildren
OffencescoveredundertheAct:
PenetrativeSexualAssault
AggravatedPenetrativeSexualAssault
SexualAssault
AggravatedSexualAssault
SexualHarassmentoftheChild
UseofChildforPornographicPurposes

72.PunishmentofperjurycomesunderIPC
section?
a)IPC191
b)IPC192
c)IPC193
d)IPC197
CorrectAnswer-C
Ans:C.IPC193
Definespunishmentforfabricatingfalseevidence
Thewitnessisliabletobeprosecutedforperjury,andthe
imprisonmentmayextenduptosevenyears.
193IPC:punishmentforfalseevidence,(punishmentforperjury):
imprisonmentupto7yearsandalsoliableforfine
I91IPC:Givingfalseevidence(perjury)underoath.
I92IPC:Fabricatingfalseevidence(perjury)
THEESSENTIALSOFFORENSICMEDICINEANDTOXICOLOGY
-Dr.K.S.NARAYANREDDY33rdedition-pg-13

73.Whencivilnegligenceisbroughtup
againstadoctor,theonusofprooflies
upon-

a)Judicialfirst-degreemagistrate
b)Policenotbelowthelevelofsub-inspector
c)Doctor
d)Patient
CorrectAnswer-D
Ans:D.Patient
"Generallyspeaking,itisforthepatientinanactionfornegligenceto
establishtheguiltofthedoctor,whoseinnocenceisotherwise
assumed."-Parikh
However,incaseswheretheruleofresipsaloquiturisapplied,the
patientneednotprovenegligence.Resipsoloquiturmeansthat"the
thingorfactspeaksforitself."Thepatienthastomerelystatewhat
accordingtohimwastheactofnegligence.
Civilnegligence
Whenapatient(orhisrelative,incaseofpatient'sdeath)filessuitin
acivilcourttogetcompensationfromhisdoctorfortheinjuryor
deathofthepatientduetodoctor'snegligence.
Adoctorfilesacivilsuittogetfeesfromhispatient,whorefusesto
payallegingnegligence.
Notethat?
Evenifadoctorisnegligent,apatientisnotentitledtoany
compensationifnodamagehasoccurred.
Anerrorindiagnosisortreatmentisnotnegligenceprovidedproper
careandskillhasbeenexercised.

Contributorynegligenceisnotadefenseincriminal
negligence.(Contributorynegligenceisanynegligenceonthepart
ofthepatientorhisattendant,whichalongwiththedoctor's
negligence,contributedtotheinjury)
CriminalnegligencecasesaredealtwithunderI.P.C.section304A.

74.Relativesofapatienttoldduring
postmortemexaminationthattheperson
hadatattoo-whichwasnowinvisible.
Howtoidentify?

a)ExaminetheLymphnode
b)Spectrophotometer
c)Ordinarylight
d)X-ray
CorrectAnswer-A
Answer-A-ExaminetheLymphnode
Tattoomarksarepermanentwhendyepenetratesthedermis.
Infraredphotographymakesoldtattoosreadilyvisible.
Afadedtattoomarkbecomesvisiblebytheuseofultravioletlamp4
orrubbingthepartandexaminingwithamagnifyinglens.
Marksarerecognizedevenindecomposedbodies(Aret)whenthe
epidermisisremoved.Lymphnodesnearthetattoomarkshow
pigmentdeposition.

75.TrueabouthymeninChildrapeis-
a)Hymeneasilytearsbecauseitisthin
b)Hymeneasilytearsbecauseitisinthefront
c)Hymenhardlytearsbecauseitishighlyelastic
d)Hymenhardlytearsbecauseitissituateddeep
CorrectAnswer-D
Answer-D-Hymenhardlytearsbecauseitissituateddeep
Thehymenmaynotruptureafterrapeif:
Penetrationwasnotfull
Thehymenistough,fleshyandelastic
Inayoungchild,fullpenetrationmaynotoccur
GautamBiswasReviewofforensicmedicine&Toxicology2/e;pg-
332
https://www.who.int/violence_injury_prevention/resources/publications/en/guidelines_chap7.pdf

76.Magnan'ssymptomsarerelatedto
a)Cocainepoisoning
b)Cannabis
c)Cannabis
d)Alcohol
CorrectAnswer-A
Answer:A.Cocainepoisoning
Magnan'ssignorSymptomisaclinicalsigninwhichpeople
withcocaineaddictionexperienceparaesthesiawhichfeelslikea
constantlymovingforeignbodies,(cocainebugs)suchasfinesand
orpowder,undertheskin.Itisthemosttactilehallucination.
Degenerationofthecentralnervoussystemoccurs,andthepatient
maysufferfromhallucinations,convulsions,delirium,andinsanity.
Magnan'ssymptomorcocainebugsischaracteristic,inwhichthere
isafeelingasifgrainsofsandarelyingundertheskinorsome
smallinsectsarecreepingontheskingivingrisetoitchingsensation
(formication,tactilehallucination)withresultantexcoriation,leading
toirregularscratchesandulcers.
THEESSENTIALSOFFORENSICMEDICINEANDTOXICOLOGY
ByDr.K.S.NARAYANREDDY33/e-pg-603

77.ParentsoftheChildcomplainsof
assaultedbyoneoftheirrelativesand
analpaininachild,Oninvestigationtest
showsyellowiodinecrystalswithpicric
acid-whatisthenameofthetest?

a)Florencetest
b)Barberio'stest
c)Acidphosphatasetest
d)CreatinePhosphokinase
CorrectAnswer-B
Answer:B.Barberio'stest
Itdetectsspermin(secretedbytheprostate)andusesanaqueous
oralcoholicsolutionofpicricacid.
Asaturatedaqueousoralcoholicsolutionofpicricacid,whenadded
tothespermaticfluid,producesyellowneedle-shapedrhombic
crystalsofsperminepicrate.Thereactionprobablydependsonthe
presenceofprostaticsecretion.
TheAcidPhosphataseTest:Theprostaticsecretionelementof
seminalfluidcontains500to1000timesgreateracidphosphatase
thananyotherbodyfluid.Humanredcells,semenofhigherapes,
andjuiceofcauliflowerhaveacidphosphataselevelsimilartothatof
humansemen
THEESSENTIALSOFFORENSICMEDICINEANDTOXICOLOGY
ByDr.K.S.NARAYANREDDY33/e-pg-434


78.TheMagistratecandetainthemaximum
numberofdaysforamentallyillperson
aspermentalhealthcareis

a)30days
b)90days
c)50days
d)100days
CorrectAnswer-A
Ans:A.30days
*IftheMagistrateissatisfiedthatitisnecessarytodetainthe
allegedmentallyillpersoninapsychiatrichospital,hepassesa
ReceptionOrder(orderforadmissionanddetention),whichisvalid
for30days.Ifheisnotsatisfied,hemayrefusetheapplication,
givinghisreasonsinwriting,acopyofwhichissuppliedtothe
applicant.
*AdmissionUnderSpecialCircumstance
-Amentallyillpersonmaynotbeabletoexpresshiswillingness
foradmissionasavoluntarypatient,Suchpersoncanbeadmittedin
apsychiatrichospital(psychiatricnursinghome)foraperiodof90
daysifanapplicationismadebyarelativesandfriends

79.Vectorforzikavirusis-
a)Aedes
b)Culex
c)Anopheles
d)Noneofthese
CorrectAnswer-A
Ans:A.Aedes
Zikavirusdisease
iscausedbytheZikavirus,whichisspreadto
peopleprimarilythroughthebiteofaninfectedmosquito(Aedes
aegyptiandAedesalbopictus).Theviruscanalso
betransmittedthroughsex.
Thesearethesamemosquitoesthatspreaddengueand
chikungunyaviruses.
Amosquitogetsinfectedwithaviruswhenitbitesaninfected
personduringtheperiodoftimewhentheviruscanbefoundinthe
person'sblood,typicallyonlythroughthefirstweekofinfection.
MedicalMicrobiologyandImmunology14/e-pg;395

80.AllaretrueregardingJapanese
encephalitisexcept?
a)Causedbyflavivirus
b)Humansaredead-endhosts
c)Transmittedbyculex
d)Cattlesareamplifierhosts
CorrectAnswer-D
Ans.is'd'i.e.,Cattlesareamplifierhosts
Japaneseencephalitis
CausedbyagroupBarbovirus(flavivirus)
ItisaZoonoticdiseaseieinfectingmainlyanimalsandincidentally
man.
Insouth,epidemicshaveoccuredinKarnataka,Andhrapredesh,
TamilNadu,andKerala.
Human,cattle,andhorsesaredead-endhostsasthedisease
manifestsasfatalencephalitis.
Pigsactasanamplifyinghostandhaveaveryimportantroleinthe
epidemiologyofthedisease.
Infectioninswineisasymptomatic,exceptinpregnantsows,when
abortionandfetalabnormalitiesarecommonsequelae.
ThemostimportantvectorisCulextritaeniorhynchus,whichfeeds
oncattleinpreferencetohumans.
ThenaturalhostsoftheJapaneseencephalitisvirusarebirds,not
humans.
InNovember2011,theJapaneseencephalitisviruswasreported
inCulexbitaeniorhynchusinSouthKorea

81.Allofthefollowingis/arehaving
superantigenPropertyExcept
a)Vibriocholera
b)Streptococcalpyrogenic
c)Staphylococcalenterotoxins
d)Noneofthese
CorrectAnswer-A
Answer-A-VibrioCholera
SAgsareproducedbysomepathogenicvirusesandbacteriamost
likelyasadefensemechanismagainsttheimmunesystem.
Toxicshocksyndrometoxin(TSST-1),epidermolytictoxinandother
staphylococcalenterotoxinaresuperantigens.
ThesesuperantigenscanbindMHCmoleculesoutsidethepeptide-
bindingcleft.
Consequently,superantigencanactivateupto10%ofT-cellsina
nonspecificmannerwhichinturnleadstothereleaseoflarge
quantitiesofcytokines.
Jawetz27/e-pg-135

82.TypeAbioterrorismAgent-
a)Chikungunya
b)Anthrax
c)Hendra
d)Influenza
CorrectAnswer-B
Answer-B-Anthrax
CategoryAagents-consistsoftheagentsthatareconsideredthe
highestrisk
IncludedamongCategoryBagentsisonethatcouldconceivably
threatenwaterandfoodsafety.
CategoryCincludespathogensthatareconsideredemerging
infectiousdiseasethreatsandwhichcouldbeengineeredformass
dissemination.
CategoryA
CategoryB
CategoryC
Anthrax
Caliciviruses
AntimicrobialResistance
Botulism
Chikungunya
Hendra
Dengue
Cholera
Influenza(highlypathogenicstrains)
Ebola
E.coliO157:H7 MERS
Hantavirus HepatitisA
Nipah
Lassa
Ricintoxin
Prions
Marburg
Salmonella
Rabies
Plague
Typhusfever
SARS
Smallpox Yellowfever
Tick-Borneencephalitis
Tularemia Zika
Tuberculosis
Jawetz27/e-pg-417


83.CulturemediausedforE.coli0157:H7is
a)SMAC
b)WilsonandBlairmedium
c)PotassiumtelluriteinMcleod'smedium
d)Deoxycholatecitrateagar(DCA)
CorrectAnswer-A
Answer-A-SMAC
Anenterohemorrhagicbacterialstrain,E.coliO157:H7infectsthe
alimentarytractandinducesabdominalcrampswithhemorrhagic
diarrhea.TransmissionofE.coliO157:H7occursviathefecal-oral
routeafterconsumptionofcontaminated,undercookedliquidsand
foods.
MacConkey-SorbitolChromoSelectAgar(SMAC)isrecommended
forselectiveisolationofEscherichiacoli0157:H7fromfoodand
animalfeedingstuffs.
MacConkeySorbitolAgarisbasedontheformulationdescribedby
RappaportandHenigh.Themediumcontainssorbitolinsteadof
lactoseanditisrecommendedforthedetectionofenteropathogenic
strainsofE.coli0157:H7whichfermentslactosebutdoesnot
fermentsorbitolandhenceproducecolorlesscolonies.
SorbitolfermentingstrainsofE.coliproducepink-redcolonies.The
redcolorisduetotheproductionofacidfromsorbitol,absorptionof
neutralredandasubsequentcolourchangeofthedyewhenthepH
ofthemediumfallsbelow6.8.
https://www.sigmaaldrich.com/analytical-
chromatography/microbiology/microbiology-products.html?
TablePage=18297647

84.DonovanosisisCausedBy-
a)H.ducreyi
b)Leishmaniadonovani
c)K.granulomatis
d)Treponemapallidum
CorrectAnswer-C
Answer-C-K.Granulomatis
Donovanosisisasexuallytransmittedgenitalulcerdisease.

Thebacteriumthatcausesdonovanosis(Klebsiella
granulomatis)infectstheskinaroundthegenitals,groinoranal
areaandcausesulcersanddestructionoftheskin.


85.FungalInfectionwhichisacquiredby
traumaticinoculationis?
a)Sporothrix
b)Blastomycosis
c)Coccidioides
d)Paracoccidioides
CorrectAnswer-A
Answer-A-Sporothrix
Sporothrixschenckiiisathermallydimorphicfungusthatliveson
vegetation.Itisassociatedwithavarietyofplants,grasses,trees,
sphagnummoss,rosebushes,andotherhorticulturalplants.
TheconidiaorhyphalfragmentsofS.schenckiiareintroducedinto
theskinbytrauma.
Followingatraumaticintroductionintotheskin,S.schenckiicauses
sporotrichosis,achronicgranulomatousinfection.
Thismanifestationmimicschroniccavitarytuberculosisandtendsto
occurinpatientswithimpairedcell-mediatedimmunity.
Jawetz27/e-pg-670

86.Whichofthefollowingisnotinvolvedin
urethritis
a)Trichomonas
b)H.ducreyi
c)Chlamydia
d)Gonococcus
CorrectAnswer-B
Answer-B-H.ducreyi
Bacteriathatcommonlycauseurethritistoinclude:
E.coliandotherbacteriapresentinthestool
Gonococcus,whichissexuallytransmittedandcausesgonorrhea.
Chlamydiatrachomatis,whichissexuallytransmittedand
causeschlamydia.
Theherpessimplexvirus(HSV-1andHSV-2)canalsocause
urethritis.Trichomonasisanothercauseofurethritis
-https://www.webmd.com/a-to-z-guides/urethritis-symptoms-
causes-treatments#1;Jawetz27/e-pg-746


87.Apatientcomplainsaboutnausea,
vomitingandstomachcrampsafter
attendingasocialgatheringparty,which
causativeorganismislikelyresponsible
forFoodPoisoningwithin3hours.

a)Staphylococcusaureus
b)Salmonella
c)Clostridiumbotulinum
d)Clostridiumperfringens
CorrectAnswer-A
Answer-A-Staphylococcusaureus
Staphfoodpoisoningischaracterizedbyasuddenstartofnausea,
vomiting,andstomachcramps.Mostpeoplealsohavediarrhea.
Symptomsusuallydevelopwithin30minutesto6hoursaftereating
ordrinkinganitemcontainingStaphtoxinandlastnolongerthan1
day.Severeillnessisrare.
Theillnesscannotbepassedfromonepersontoanother.
https://www.cdc.gov/foodsafety/diseases/staphylococcal.html

88.A46-year-oldwomanwithHIVcomplains
severepersistentdiarrhea,Histological
Investigationwasperformed,Identifythe
organismcausingdiarrheainHIVPatient.

a)Cryptosporidium
b)Staphylococcusaureus
c)Salmonella
d)Clostridiumbotulinum
CorrectAnswer-A
Answer-A-Cryptosporidium
*TheoriginofinfectiousdiarrheainpatientswithAIDScanbe
dividedinto2generalcategories:thatcausedbycommon
pathogensandthatcausedbyopportunisticpathogens.
*ThemostcommoninfectiousorganismscausingAIDS-related
diarrheainclude
-Cytomegalovirus(CMV)
-TheparasitesCryptosporidium,
-Microsporidia
-Giardialamblia
*AndthebacteriumMycobacteriumavium-intracellulare(MAC).
*Otherbacteriaandparasitesthatcausediarrhealsymptomsin
otherwisehealthypeoplemaycausemoresevere,prolongedor
recurrentdiarrheainpeoplewithHIVorAIDS
https://aidsinfo.nih.gov/news/175/aids-related-diarrhea

89.OrganismCausingLGV
a)Chlamydiatrachomatis
b)Neisseriagonorrhoeae
c)Treponemapallidum
d)Haemophilusducreyi
CorrectAnswer-A
Answer-A-Chlamydiatrachomatis
Lymphogranulomavenereum(LGV)isalong-term(chronic)
infectionofthelymphaticsystem.
Itiscausedbyanyofthe3differenttypes(serovars)ofthe
bacteriaChlamydiatrachomatis.
Thebacteriaarespreadbysexualcontact.Theinfectionisnot
causedbythesamebacteriathatcausegenitalchlamydia.
Chlamydiatrachomatiscauseseye(conjunctivitis,trachoma),
respiratory(pneumonia),andgenitaltract(urethritis,
lymphogranulomavenereum)infections.
DiagnosismadewiththenucleicacidtestforC.trachomatis,LGV
serovarsdiagnosedserologically.
LangeReviewofMedicalMicrobiology&Immunology14/e208

90.Whichinterleukinresponsiblefor
producingIgEfromBcells
a)IL1
b)IL3
c)IL4
d)BothB&C
CorrectAnswer-C
Answer-C,IL4
IgEisproducedbyplasmacellslocatedinlymphnodesdrainingthe
siteofantigenentryorlocally,atthesitesofallergicreactions,by
plasmacellsderivedfromgerminalcentersdevelopingwithinthe
inflamedtissue.
IgEantibodyproductionrequiresTH2cellsthatproduceinterleukin-4
(IL-4)andIL-13anditcanbeinhibitedbyTH1cellsthatproduce
interferon-(IFN-).
https://www.ncbi.nlm.nih.gov/books/NBK27117/

91.HbsAgisbasedonwhichprinciple
a)Immunochromatographyassays
b)Chemiluminescence
c)ELISA
d)Immunofluorescence
CorrectAnswer-A
Answer-A-Immunochromatographyassays
HBVchroniccarriersarethoseinwhomHBsAgpersistsformore
than6monthsinthepresenceofHBeAgoranti-HBe.HBsAgmay
persistforyearsafterthelossofHBeAg.Themostusefuldetection
methodsareenzyme-linkedImmunosorbentassayforHBV
antigensandantibodiesandPCRforviralDNA-Jawetz27/epg-
504
Serologicalassaysdetectthehostimmuneresponse(antibodiesto
HCV)oraviralantigen(HBsAg,HCVcAg).Theyarebasedonthe
immunoassayprincipleandareavailableintheformofrapid
diagnostictests(RDTs)orlaboratory-basedenzymeimmunoassays
(EIAs),chemiluminescenceimmunoassays(CLIAs)and
electrochemiluminescenceimmunoassays(ECLs).-
https://www.ncbi.nlm.nih.gov/books/NBK442281/


92.NosocomialInfectionoccurswithin?
a)A-48
b)B.72
c)C.24
d)D.50
CorrectAnswer-A
Answer-A-48hours
Ahospital-acquiredinfection(HAI),alsoknownasanosocomial
infection,isaninfectionthatisacquiredinahospitalorotherhealth
carefacility.
Nosocomialinfectionscanbedefinedasthoseoccurringwithin48
hoursofhospitaladmissionorwithin3daysofdischargeor30days
ofoperation.
Somewellknownnosocomialinfectionsinclude:
ventilator-associatedpneumonia,
Methicillin-resistantStaphylococcusaureus,
Candidaalbicans,
Acinetobacterbaumannii,
Clostridiumdifficile,
Tuberculosis,
Urinarytractinfection,Vancomycin-resistantEnterococcus,and
Legionnaires'disease.
https://www.ncbi.nlm.nih.gov/books/NBK441857/

93.Whichofthefollowingcellcomponents
producedbyNeisseriagonorrhoeaeis
responsibleforattachmenttohostcells?

a)Lipooligosaccharide
b)Pili(fimbriae)
c)IgA1protease
d)Outermembraneporinprotein
CorrectAnswer-B
Answer-B-Pili(fimbriae)
Piliarehair-likeappendagesthatextenduptoseveralmicrometers
fromthegonococcalsurface.Theyenhanceattachmenttohostcells
andresistancetophagocytosis.Theyaremadeupofstackedpilin
proteins.
Theaminoterminalofthepilinmolecule,whichcontainsahigh
percentageofhydrophobicaminoacids,isconserved.Theamino
acidsequencenearthemidportionofthemoleculealsois
conserved;thisportionofthemoleculeservesinanattachmentto
hostcellsandislessprominentintheimmuneresponse.
Theaminoacidsequencenearthecarboxyl-terminalishighly
variable;thisportionofthemoleculeismostprominentinthe
immuneresponse.ThepilinsofalmostallstrainsofN.gonorrhea
areantigenicallydifferent,andasinglestraincanmakemany
antigenicallydistinctformsofpilin
Jawetz27/e-pg-282

94.Wherewillyouputchemicalliquid
biomedicalwaste
a)White
b)Yellow
c)Blue
d)Red
CorrectAnswer-B
Ans.Byellow
Chemicalwasteiscategorizedintotheyellowcategory.the
hazardouschemicalandcytotoxicwasteisayellowcategorywitha
specialsignof"CYTOTOXIC"waste.
Anotherliquidwasteasbodysecretionsiscategorizedintothe
yellowcategoryofbiomedicalwasteguidelines,2016
Park'sPSM24thed.Pageno.831

95.Maximumworkhoursforaperson
includingovertimeunderthefactoriesact:
a)48
b)50
c)60
d)100
CorrectAnswer-C
Ans:C.60hours.
Factoriesactpermitsworkfor48hoursperweekwith2hoursof
overtimeeveryday.
makingitapproximately60hoursofmaximumworkasperthe
factoriesactofIndia
Park'sPSM24thed.Pageno.852

96.Thevaccinetobegivenafterdisaster
a)vaccinationagainsttyphoid
b)vaccinationagainstcholera
c)vaccinationagainsttyphoidandcholera
d)vaccinationagainsttetanus
CorrectAnswer-D
Ans.D.vaccinationagainsttetanus.
Themajorconcernforanyoneexposedtounsanitaryconditionsis
thattheyshouldbeuptodatewiththetetanus-containingvaccine
becauseiftheyareinjured(asiscommonindisastersettings)the
injuryislikelytobecontaminated.
Routinelyrecommendedvaccinesarerecommendedforevacuees,
justliketheyareforeveryoneelse.
Choleraandtyphoidvaccinedonothaveanyevidenceformass
vaccinationduetothelowlevelofexposureandprevention
TetanusandHepBvaccineisrequired.

97.WhatistheMONICAproject?
a)Multinationalmonitoringoftrendsanddeterminantsin
CardiovascularDisease
b)Multinationaloftrendsanddeterminantsincerebrovascular
disease
c)Multinationalmonitoringoftrendsanddeterminantsindiabetes
d)Multinationalmonitoringoftrendsanddeterminantsin
congenitalheartdisease
CorrectAnswer-A
Ans:A.Multinationalmonitoringoftrendsanddeterminantsin
CardiovascularDisease

TheWHOhascompletedaprojectknownasMONICA"
(multinationalmonitoringoftrendsanddeterminantsin
cardiovasculardiseases)"toelucidatethisissue.
Forty-onecentersin26countrieswereparticipatinginthisproject,
whichwasplannedtocontinuefora10yearperiodendingin1994.
Park'sPSM24thed.Pageno.385

98.Extendedsicknessbenefitfor
tuberculosisundertheESIActis:
a)91days
b)1-year
c)2years
d)4years
CorrectAnswer-C
Ans.C.2years
EXTENDEDSICKNESSBENEFIT:
Inadditionto91daysof
sicknessbenefitinsuredpersonssufferingfromcertainlong-term
diseasesareentitledtoExtendedSicknessBenefitasshownbelow,
foramaximumperiodoftwoyears.
ExtendedSicknessBenefitwitheffectfrom1.1.2000ispayable,in
thecasewheretheinsuredpersonhasbeenincontinuous
employmentfor2yearsTuberculosis
Park'sPSM24thed.Pageno.854

99.Apersonreports4hoursafterhavinga
cleanwoundwithoutlaceration.Hehad
takenTT10yearsbefore.thenextstepin
managementis:

a)FullcourseTetanusvaccinetobegiven
b)FulldoseTTwithTIG
c)Single-doseTT
d)Noneedforanyvaccine
CorrectAnswer-C
Ans.C.Single-doseTT
Apatientwithwoundlessthan6hoursoldifclean,non-penetrating
andwithnegligibletissuedamageifhadacompletecourseoftoxoid
oraboosterdosemorethan10yearsago(categoryC)shouldbe
treatedwithToxoid1dose.
Park'sPSM24thed.Pageno.331

100.RecentInfluenzaPandemicwasdueto:
a)H1N1
b)H5N1
c)H7N7
d)H3N2
CorrectAnswer-A
Ans.A.H1N1
H1N1?swineflu?causethemajorfluPandemic(1918and2009)
H5N1?avianinfluenza.maycausesporadicoutbreaksor
epidemics.Itisassociatedwithhighmortality.

101.MissionIndradhanushisfor:
a)Non-communicablediseases
b)Universalimmunization
c)Familyplanning
d)Safewaterandsanitation
CorrectAnswer-B
Ans.B.Universalimmunization
TheMinistryofHealth&FamilyWelfarehaslaunched"Mission
IndraDhanush",depictingsevencoloursoftherainbowinDecember
2014,tofullyimmunizemorethan89lakhchildrenwhoareeither
unvaccinatedorpartiallyvaccinated;thosethathavenotbeen
coveredduringtheroundsofroutineimmunizationforvarious
reasons.
Theywillbefullyimmunizedagainstsevenlife-threateningvaccine-
preventablediseaseswhichincludediphtheria,whoopingcough,
tetanus,polio,tuberculosis,measles,andhepatitis-B.
Inaddition,vaccinationagainstJapaneseEncephalitisand
HaemophilusinfluenzatypeBwillbeprovidedinselected
districts/statesofthecountry.
Pregnantwomenwillalsobeimmunizedagainsttetanus.
Ref.Park'sPSM24thed.Page462

102.Astudyhadanormaldistributionwith
themedianvalueas200andstandard
deviation20.68%willfallbetween

a)160-240
b)170-230
c)180-220
d)190-210
CorrectAnswer-C
Ans.C.180-220
Asthemedianvalueis200andthestandarddeviationis20,the
normaldistributionis:
68%ofthepopulationwillhavevaluesbetween?median+/-1SD=
220+/-20=180-220
95%ofthepopulationwillhavevaluesbetween?median+/-2SD=
220+/-40=160-240
Note:intheMCQ,asthedatashowsanormaldistribution,the
medianwillbeequaltomeanandthemode.
Park'sPSMEd.24thpageno.885

103.Whichofthefollowingisa
technique/methodbasedonbehavioural
sciences

a)Managementbyobjectives
b)Networkanalysis
c)Systemsanalysis
d)Decisionmaking
CorrectAnswer-A
Ans.A.Managementbyobjectives
Themethodsbasedonbehaviouralsciencesinclude
organizationaldesign
personalmanagement
Managementbyobjectives
informationsystems
communication
TheQuantitativemethodsinclude:
cost-benefitanalysis
cost-effectiveanalysis
input-outputanalysis
networkanalysisasPERTandCPM
Planningprogrammingbudgetingsystems
decisionmaking
Ref:Park's25ed,PAgeno.934

104.Asperthesustainabledevelopment
goals,ThetargetforMMRistoachieve
maternaldeathsof

a)<70/Laclivebirths
b)<100/laclivebirths
c)<7/1000livebirths
d)<10/1000livebirths
CorrectAnswer-A
Ans:A.<70/Laclivebirths
By2030,reducetheglobalmaternalmortalityratiotolessthan70
per100,000livebirths.
Park'sPSM24thed.Pageno.28Table:4

105.Thebestmethodforroutinemonitoring
ofairpollution
a)Sulphurdioxide,smoke,andparticulatematter
b)Sulphurdioxide,Hydrogensulphide,carbonmonoxide
c)Carbondioxide,hydrogensulphide,lead
d)Sulphurdioxide,Leadandparticulatematter
CorrectAnswer-D
Ans.DSulphurdioxide,Leadandparticulatematter
Airqualityindexconsistsof:
Particulatematter(lessthan2.5micrometersand10micrometer?
PM2.5andPM10)
Nitrogendioxide(NO2)
Sulphurdioxide(SO2)
Carbonmonoxide(CO)
Ozone(O3)
Ammonia(NH3)
Lead(Pb)
Reference:https://pib.gov.in/newsite/PrintRelease.aspx?
relid=110654


106.Thevariationindataiscomparedwith
anotherdatasetby:
a)Variance
b)Coefficientofvariation
c)Thestandarderrorofmean
d)Standarddeviation
CorrectAnswer-B
Ans.B.Coefficientofvariation
Variance:IsthesquareofSDwhichtellsaboutthestandard
deviation
Coefficientofvariation:Itmayhelpbycomparingthevariationsin
thedataset
Thestandarderrorofthemean:Itistocomparethemeansofthe
datasetswhichhaveadifferentsamplesize,centraltendency,and
standarddeviations
Standarddeviation:Itisthedeviationofvaluesfromthemean
Ref.FundamentalsofBiostatistics-7thEdition(Pg20,21)

107.Inwhichofthefollowingmethodsof
managementisthebenefitmeasuredin
naturalunits?

a)Programbudgetingsystem
b)Networkanalysis
c)Cost-effectiveanalysis
d)Cost-benefitanalysis
CorrectAnswer-C
Ans.C.Cost-effectiveAnalysis
Incost-effectiveanalysis(CEA),benefitsaremeasuredinnatural
unitsoftheoutcomesoftheprograms(life-yearsgained,cases
prevented,etc.)andthecostsaremeasuredinmonetaryunits.
ThemostcomprehensiveindicatorofCEAisQuality-AdjustedLife
Years(QALYs).
Park'sPSM24thed.Pageno.908


108.Thedifferencebetweentheincidencein
theexposedandnon-exposedgroupis
bestgivenby:

a)Relativerisk
b)Attributablerisk
c)Populationattributablerisk
d)Oddsratio
CorrectAnswer-B
Ans.BAttributableRisk
AttributableRisk
AttributableRiskisthedifferenceinincidenceratesorproportionsof
diseaseordeathbetweenanexposedandnon-exposedgroup.
Itisexpressedinpercentageandgivestheextenttowhichthe
diseasecanbeattributedtotheexposureinacohortstudy.
Reference:Park25thEdition,pageno:86


109.Ifonevariableisgiventhenyoucanfind
anothervariableby
a)Coefficientofvariation
b)Coefficientofcorrelation
c)Coefficientofregression
d)Coefficientofdetermination
CorrectAnswer-C
Ans.C.Coefficientofregression
Thecoefficientofcorrelationtellsaboutthestrengthofassociation
butnotaboutquantity.Ontheotherhand,thecoefficientof
regressionisusedforquantification.
Ifwewishtoknowinanindividualcasethevalueofonevariable,
knowingthevalueoftheother,wecalculatewhatisknownasthe
regressioncoefficientofonemeasurementtotheother.Itis
customarytodenotetheindependentvariablebyxandthe
dependentvariablebyy.

110.Prospectivescreeningisdoneincase
of?
a)Neonateforthyroiddiseases
b)Immigrantscreening
c)Papsmearfor45-yearfemale
d)Diabetesmellitusfor40-yearmale
CorrectAnswer-B
Ans.B.ImmigrantScreening
Screeningofimmigrantstoacountryisanexampleofprospective
screening.
Prospectivescreening:
Peoplescreenedforothersbenefit
Theessentialpurposeiscasedetection
Requestedforscreeningfordiseasecontrol;specificrequestfrom
theauthority

111.Aresearcherwantedtoprovethe
relationbetweenCOPDandsmoking.He
collectedpatientsrecordsfrom
governmenthospitalsandrecordsof
cigarettesalesfromthefinanceand
taxationdepartment.Thisisanexample
ofwhichstudydesign:

a)Cross-sectional
b)Posologicalstudy
c)Ecologicalstudy
d)Operationsresearch
CorrectAnswer-C
Ans.C.Ecologicalstudydesign
Thisisanexampleofanecologicalstudy.
AnecologicalstudyisatypeofObservationalstudywhere
informationiscollectedonagroup(orpopulation)ratherthanon
individualmembersandthenanalyzed.
Heretheassociationbetweenasummarymeasureofsaleof
cigarettes(riskfactor)andasummarymeasureofthenumberof
casesofCOPD(outcome)isstudied.

112.Astudywasdonetoassessmalnutrition
amongyoungchildren.100childrenwere
selectedeachfromruralandurban
areas.Outofthese,30amongruraland
20amongurbanwerefoundtobe
malnourished.whichofthefollowing
statisticaltestisusedtocomparethe
datasets?

a)Pairedt-test
b)Chi-square
c)Thestandarderrorofmean
d)ANOVA
CorrectAnswer-B
Ans.B.Chi-squaretest
Chi-square(x2)Testoffersanalternatemethodoftestingthe
significanceofthedifferencebetweentwoproportions.Ithasthe
advantagethatitcanalsobeusedwhenmorethantwogroupsare
tobecompared.
Park'sPSM24thed.Pageno.889

113.Theactivedisinfectantpropertyof
bleachingpowderisdueto:
a)Chlorine
b)Hypochloricacid
c)Hypochlorousacid
d)Chloramines
CorrectAnswer-C
Ans.C.Hypochlorousacid
Hypochlorousacidisthemosteffectiveformofchlorineforwater
disinfection.
Theactivecomponentofbleachingpowder,CaOCl2,is
hypochlorousacid.
Thedisinfectingactionofchlorineismainlyduetohypochlorous
acid,andtoasmallextentduetothehypochloriteions.
Park'sPSM24thed.Pageno.138

114.Motherdoesnottransmitwhatantibody
tothebaby?
a)Polio
b)Diphtheria
c)Diphtheria
d)Tetanus
CorrectAnswer-A
Ans.A.Polio
Whengivenduringpregnancy,theTdapvaccineboostsantibodies
inthemother,whicharetransferredtoherdevelopingbaby.Early
third-trimesteradministrationoptimizesneonatalantibodylevels.
Ref.https://www.ncbi.nlm.nih.gov/pmc/articles/PMC4168293/pdf/ciu327.pdf
1)


115.Voluntaryadmissioncanbedone
maximumuptohowmuchtime
accordingtoMHA2017

a)48hrs
b)7Days
c)30Days
d)90Days
CorrectAnswer-D
Ans.D.90Days
Ifadmissionrequiresmorethan30daysorreadmissionwithin7
days(section90),theyshouldbeexaminedbytwopsychiatristsand
canbeadmittedforamaximumperiodof90daysiftheysatisfy
normsasperthissectionoftheactandhavetoinformtheboardfor
permission,takingaccountofADandconsentshouldbereviewed
fortnightlyalongwithplanningforcommunity-basedtreatment.
http://www.amhonline.org/article.asp?issn=2589-
9171;year=2018;volume=19;issue=1;spage=9;epage=14;aulast=Neredumilli


116.Bloodbagsaredisposedofin
a)Yellowbag
b)Blackbag
c)Redbag
d)Whitebag
CorrectAnswer-C
Ans:C.Redbag
Theblackbagisusedforcollectingdrywastematerialwhichisnot
infectious.
Materialslikepaper,plastics,cardboardboxes,andotherdrywaste
generatedinthehospitalofficeorinthewardsaredisposedofin
thisbag.
Thisisnotbiomedicalwaste.
Theredbagisusedforthedisposalofplasticscollectedfrom
operationtheaters,ICUsandwards.
Theyellowbagisusedforhighlyinfectiousitemslikeapathological
waste,humananatomicalwastesuchasbodyparts,amputated
parts/organs,tumors,placentas,abortedordeadfetuses,blood-
soakedcottonbandages,animaltissues,organs,carcasses,etc.
BlueorWhiteopaquebagisusedforcollectingthesegregated
metalsharpssuchasneedles,blades,saws,scalpels,andglass
pieces.
RefPark23'/ep.793-794]

117.AbsolutecontraindicationforIUD(Intra
UterineContraceptiveDevice)areall
except?

a)Pregnancy
b)Undiagnosedvaginalbleeding
c)Pelvicinflammatorydisease
d)Uterinemalformation
CorrectAnswer-D
Ans.is'di.e.,Uterinemalformation
Contraindications
ABSOLUTE:
Suspectedpregnancy
Pelvicinflammatorydisease
Vaginalbleedingofundiagnosedetiology
Cancerofthecervix,uterusoradnexaandotherpelvictumours
Previousectopicpregnancy
RELATIVE:
Anaemia
Menorrhagia
HistoryofPID(PelvicInflammatoryDisease)sincelastpregnancy
Purulentcervicaldischarge
Distortionsoftheuterinecavityduetocongenitalmalformations,
fibroid
Unmotivatedperson

118.Whichofthefollowingiswater-related
disease
a)Yellowfever
b)Scabies
c)Cholera
d)Dysentery
CorrectAnswer-A
Ans.is'a'i.e.,Yellowfever
*Watercancausediseasenotonlybydrinkingbutalsobyvarious
othermodes,directlyorindirectly.
*Dependinguponthetypeoftransmission,thediseasesare
classified.
*PublicHealthClassificationofWaterbornediseases
*Waterbornediseases:Occurduetodrinkingcontaminatedwater,
transmittedbythefaeco-oralroute.Examples:Typhoid,Cholera.
Dysentery,ViralHepatitisA
*Waterwasheddiseases:Includeinfectionsoftheouterbody
surfacewhichoccurduetoinadequateuseofwaterorimproper
hygiene.Examples:Scabies,Trachoma,Typhus,Bacillary
dysentery,Amoebicdysentery
*Water-baseddiseases:Referstoinfectionstransmittedthroughan
aquaticinvertebrateanimal.Examples:Schistosomiasis.
Dracunculiasis(Guineawormdisease)
*Water-relateddiseases(Waterbreedingdiseases):Areinfections
spreadbyinsectsthatdependonwater.Examples:Malaria,
Filariasis,Dengue,Yellowfever,Onchocerciasis

119.InVision2020,thetargetforSecondary
Sevicecenterisforhowmuch
population?

a)10000
b)50000
c)1lac
d)5lac
CorrectAnswer-D
Ansis'd'i.e.5lac
Vision2020:TheRighttoSight,isaglobalinitiativelaunchedby
WHOin1999inabroadcoalitionwitha'TaskForceofInternational
Non-GovernmentalOrganisations(NGOs)'tocombatthegigantic
problemofblindnessintheworld.
Theobjectiveistoeliminateavoidableblindnessbytheyear2020
andtoreducetheglobalburdenofblindness.Thegovernmentof
Indiahasadopted'Vision2020:RighttoSight'undertheNational
ProgrammeforControlofBlindness.
BasedontherecommendationsofWHO,thereisaneedtodevelop
theinfrastructurepyramidwhichincludes
I.PrimarylevelVisionCentres
Thereisaneedtodevelop20000visioncenters,
AnEachwithoneOphthalmicAssistantorequivalent(Community-
basedMLOP),Coveringapopulationof50000.
2.ServiceCentres.
Thereisaneedtodevelop2000servicecentersatthe
secondarylevel.
Eachwithtwoophthalmologistsand8paramedics(Hospital-based

MLOP),andoneeyecaremanager,Coveringapopulationof5lacs.
3.TrainingCentres
Thereisaneedtodevelop200'TrainingCentres'forthetrainingof
Ophthalmologists,Coveringapopulationof50lacs.
4.CentreofExcellence(COE)
Thereisaneedtodevelop20COEwithwelldevelopedallsub-
specialtiesofOphthalmology,Coveringapopulationof5crores.

120.CA-125isamarkerforthescreeningof
ovariancancer.Tocharacterizethistest,
histopathologicalconfirmationofovarian
cancerwasdoneinacohortofpatients.
60/100womenwhotestedpositivefor
thistesthadovariancancerand20/100
womenwhotestednegativehadovarian
cancer.Whatisthenegativepredictive
valueofthistest?

a)20/100
b)40/100
c)60/100
d)80/100
CorrectAnswer-D
Ans:D.80/100
Negativepredictivevalue(NPV):Abilityofascreeningtesttoidentify
correctlyallthosewhodon'thavethedisease,outofallthosewho
testnegativeonascreeningtest.


NegativePredictiveValue(NPV)=Numberoftrue
negatives/(Numberoftruenegatives+numberoffalsenegatives).
Diseasepresent Diseaseabsent
TestPositive 60(a:TP)
40(b:FP)
TestNegative 20(c:FN)
80(d:TN)
Total
80
120
Truenegatives=80
Totalnegatives=100
Negativepredictivevalue=80/100=80%.

121.Screeningisnotusefulinwhich
carcinoma
a)Carcinomaprostate
b)Carcinomacolon
c)Carcinomabreast
d)Testiculartumor
CorrectAnswer-D
Ans:D.Testiculartumor
Screeningisnotusefulintesticulartumors.
ScreeningRecommendationsforAsymptomaticSubjects:
Breast
:
Self-examination.
Clinicalexamination.
Mammography.
MRI.
Cervical:
Paptest(cytology).
HPVtest
Colorectal:
Sigmoidoscopy
Fecaloccultbloodtesting(FOBT)
Colonoscopy
FecalDNAtesting
Fecalimmunochemicaltesting(FIT)
CTcolonography
Lung:
Low-dosecomputedtomography
(CT)scan

OvarianCA:
CAmarker-125
Transvaginalultrasound
Prostate:
Prostate-specificantigen(PSA)
Digitalrectalexamination(DRE)
Skin:
Completeskinexamination

122.Admissionratebiasis?
a)Reportingbias
b)Responsebias
c)Berkesonianbias
d)None
CorrectAnswer-C
Ans.is'c'i.e.,Berkesonianbias
Selectionbias
Selectionbiasisdistortionthatresultsfromtheprocedureusedto
selectsubjectsandfromfactorsthatinfluencestudyparticipation.
Groupstobecomparedaredifferentiallysusceptibletotheoutcome
evenbeforetheexperimentalmaneuverisperformed.
Selectionbiasmaybeofthefollowingtypes.
1.Surveillance/detectionbias
Apotentialartifactinepidemiologicdatacausedbytheuseofa
particulardiagnostictechniqueortypeofequipment.
Forexample,cancerratesmayvaryindifferentregionsorperiods,
notbecauseofanactualdifferenceintheincidenceofdiseasebut
becauseofdifferentdiagnostictechnologies.
Ifthediagnostictestisbeingusedinoneregionismoresensitive
thanotherregions,thecancerrateswillbehighinthatregioneven
withouttheexistenceofanactualdifference.
2.Neymansurvivalbias(Incidence-Prevalencebias)
Thistypeofbiasisduetomissingfatalcases,mildcasesorcases
ofshorterdurationfromthestudy.
Forexample,inastudyofbreastcancer,wecanchoosetwo
differenttypesofcases:?
IncidentcasesAllbreastcancerpatientsnewlydiagnosedduring

agiventime.
PrevalentcasesAllbreastcancerpatientswhoarealiveduringa
giventimeframe.
Thesecanleadtodifferentresultsbecausetheprobabilityoffinding
acaseinagiventimeframeisrelatedtomortalityrisk.Those
patientswhohayamildformofthediseaseandsurvivefora
relativelylongtimehaveagoodchanceofbeingaroundonthedate
ofdatacollection.Thosepatientswhodiequicklyareunlikelytobe
aroundonthatdate.
3.Referralbiasorvolunteerbias
Volunteerorreferralbiasoccursbecausepeoplewhovolunteerto
participateinastudy(orwhoarereferredtoit)areoftendifferent
thannon-volunteers/non-referral.
Thisbiasusuallyfavoursthetreatmentgroup,asvolunteerstendto
bemoremotivatedandconcernedabouttherehealth.
4.Responsebias
Thisoccurswhenthosewhorespondtoasurveydifferinimportant
waysfromthosewhodonotrespond.
Thisbiascanworkineitherdirection,i.e.,ifbiasoccurs,whenthose
whodonotrespondtoasurveydifferinimportantwaysfromthose
whorespond,itiscallednonrespondentbias.
5.Berkesonianbias
Berkesonianbiasresultsfromthegreaterprobabilityofhospital
admissionforpeoplewithtwoormorediseasesthanforpeoplewith
onedisease.So,itisalsoknownasadmissionratebias.
Forexample,Ifbreastcanceristheexposureofinterest(diseases
areoftentreatedasexposuresinhospital-basedcase-control
studies)andmeningiomaisthecasediseasethenpeoplewithboth
breastcancerandmeningiomacouldbehospitalizedforeither
breastcancerormeningiomaorboth.
However,peoplewithonlybreastcancerorwiththeonly
meningiomacouldbehospitalizedbecauseofoneofthese
diseases.
Therefore,agreaterproportionofpeopleinthecommunitywithboth
breastcancerandmeningiomawouldbeadmittedtothehospital
thanpeoplewithmeningiomaonly.
So,Berkesonianbiasoccurswhenbothexposure(e.g.,breast

cancerintheaboveexample)anddisease(e.g.,meningiomain
aboveexample)affecttheselection

123.Stimulationoftheexternalauditorycanal
leadstocoughduetowhichnerve
a)AuricularbranchVagus
b)Greaterauricularnerve
c)Auriculotemporalnerve
d)FacialNerve
CorrectAnswer-A
Ans:A.AuricularbranchVagusnerve.
>AuricularbranchofVagus(Arnold'snerve)
andFacialnerve
continuesinwardstosupplytheposteriorwallandflooroftheEAC.
>Thecoughresponsecausedwhilestimulatingtheearcanalis
mediatedbythevaguswhichalsosuppliesthelarynx.
(Re/.Shambaugh,6thed.,page45)

124.

wideningofthecartilaginouspartoftheextra
auditorycanalcalled.

a)Otoplasty
b)Myringoplasty
c)Tympanoplasty
d)Meatoplasty
CorrectAnswer-D
Ans.(d)Meatoplasty.
Meatoplasty
isanoperativetechniquetowidenthelateral
cartilaginouspartoftheexternalauditorycanal.
Otoplasty-isaproceduretochangetheshape,positionorsizeof
theears.
Myringoplasty-closureoftheperforationofparstensaofthe
tympanicmembrane
Tympanoplasty-surgicaltechniquetorepairadefectinthe
tympanicmembranewiththeplacementofagraft,eithermedialor
lateraltothetympanicmembraneannulus
ref:https://vula.uct.ac.za/access/content/group/ba5fb1bd-be95-
48e5-81be-586fbaeba29d/Meatoplasty.pdf


125.TubercularOtitismediaischaracterized
byallexcept
a)Painfulotorrhea
b)Multipleperforations
c)Palegranulations
d)Foul-smellingeardischarge
CorrectAnswer-A
Ans.A.Painfulotorrhea.
>Tuberculosisofmiddleearisacomparativelyrareentityusually
seeninassociationwithorsecondarytopulmonarytuberculosis,the
infectionreachesthemiddleearthroughtheeustachiantube
>Itischaracterizedbypainlessotorrhoeawhichfailstorespondto
theusualantimicrobialtreatment
>patientwithevidenceoftubercleinfectionelsewherefollowedby
multipletympanicmembraneperforations,abundantgranulation
tissue,andbonenecrosis,preauricularlymphnodeenlargement
https://www.ncbi.nlm.nih.gov/pubmed/21522113

126.Partialandfullclosuredonein
a)Atrophicrhinitis
b)Allergicrhinitis
c)Vasomotorrhinitis
d)Occupationalrhinitis
CorrectAnswer-A
Ans.A.Atrophicrhinitis
ATROPHICRHINITIS
>Itisaformofchronicrhinitisassociatedwithatrophyof
nasalmucosa,mucousglands,nerves,andvessels.
>Itcanbeprimaryorsecondary.
>Thesurgicaloptionsaimtoreducethesizeoftheroomy
cavitiestopreventtheexposuretodryingeffectsofair
andcrustsformationandthushelpregenerationofnasalmucosa.
Fullclosure-Young'soperation
Partialclosure-Modifiedyoung'soperation.
(Ref.Cummings,6thed.,695)

127.OccipitomentalviewofPNSX-raycalled
as
a)Caldwellview
b)Waterview
c)Townview
d)Pineview
CorrectAnswer-B
Answer:b.Waterview
OccipitomentalviewofPNSX-raycalledasWaterview

128.Whatcausesshiftingfluid?
a)ExudativeRetinaldetachment
b)TractionalRetinalDetachment
c)Rhegmatogenousretinaldetachment
d)Retinodialysis
CorrectAnswer-A
Ans.a.ExudativeRetinaldetachment.
>Inexudativeretinaldetachment,thesubretinalfluidmaybe
confinedtoalocalizedarea,usuallytheposteriorpole,ormay
extendtotheperiphery,evenformingbullousretinaldetachment.
>Thecharacteristicfeatureofasignificantexudativeretinal
detachmentisthepresenceofshiftingsubretinalfluid.

>Thefluidshiftstothemostdependentlocationwhenpatients
changebodyposition.
https://clinicalgate.com/nonrhegmatogenous-retinal-detachment/

129.Conjunctivalinjection,pharyngeal
injection,polymorphicrash,cervical
lymphadenopathycanbeseenin

a)Kawasakisyndrome
b)Measles
c)Thrombocytopenia
d)Mumps
CorrectAnswer-A
Ans:a.Kawasakisyndrome.
>Kawasakidiseaseisageneralizedvasculitisthataffectsmedium-
sizedarteries.
>Itischaracterizedbysystemicinflammationthatmanifestsas
persistentfever,erythemaofthemucousmembranes,bilateral
nonexudativeconjunctivitis,rash,swellingandrednessofthehands
andfeet,andcervicallymphadenopathy
Ref:https://www.ncbi.nlm.nih.gov/pmc/articles/PMC3034467/

130.IncongruousHomonymoushemianopia
withWernicke'shemianopiapupilisseen
withthelesionof

a)Opticradiation
b)Lateralgeniculatebody
c)Optictract
d)Anterioroccipitalcortex
CorrectAnswer-C
Ans:c.Optictract
>AnterioroptictractlesionproducesIncongruousHomonymous
hemianopia,decreasedvisualacuity,afferentpupil
defect(Wernicke'shemianopiapupil)andatrophyofopticdiscswith
characteristicBow-tieatrophyarecontralateral.
>AcompleteHomonymoushemianopiaresultsfromposterioroptic
tractlesions.
Ref:https://books.google.co.in/books?
id=bDpI9n4q3e0C&pg=PA376&lpg=PA376&dq=Incongruous+Homonymous+hemianopia+with+Wernicke%27s+hemianopic+pupil&source=bl&ots=36lTWDpqEI&sig=ACfU3U1mJNezirKF_11Ku5xpqqmWX6mdQw&hl=en&sa=X&ved=2ahUKEwio2-
fCi_bmAhUoxzgGHf56An8Q6AEwFXoECAoQAQ#v=onepage&q=Incongruous%20Homonymous%20hemianopia%20with%20Wernicke's%20hemianopic%20pupil&f=false


131.Thepatientcamewithproptosis,
restrictionofeyemovements,andwas
Euthyroid.Whatcouldthisbefrom?

a)OrbitalCellulitis
b)OrbitalLymphoma
c)Orbitalpseudotumor
d)Thyroidophthalmopathy
CorrectAnswer-C
Ans:C.Orbitalpseudotumor
>Proptosisandrestrictedeyemovementismostcommonly
associatedwiththyroidophthalmopathy
>ButinquestionaskedisEuthyroid.
>So,thebestoptiontoselectisOrbitalpseudotumor(Idiopathic
orbitalinflammatorysyndrome)
Ref.ComprehensiveOphthalmologybyAKKhurana-6th
EditionPageno335


132.Prerequisiteforsympathetic
ophthalmitis
a)Penetratinginjurytotheeye
b)Bluntoculartumor
c)Chemicalinjury
d)Urinarytractinfection
CorrectAnswer-A
Ans:a.Penetratinginjurytotheeye.
>Sympatheticophthalmiaisararetypeofuveitisthatcausessmall
abnormalclumpsofcells(granulomas)toform.
>Thisdisorderoccursintheuninjuredeyeafterapenetratinginjury
(suchaswhenapencil,pen,orstickpuncturestheeye)orsurgery
totheother(injured)eye.
Ref:https://www.msdmanuals.com/en-in/home/eye-disorders/uveitis-
and-related-disorders/sympathetic-ophthalmia.

133.Photostresstesttodifferentiate
a)Lensandcornea
b)ThemaculaandOpticnervediseases
c)Cataractandglaucoma
d)Retinalandvitreousdiseases
CorrectAnswer-B
Ans:b.ThemaculaandOpticnervediseases.
>Todistinguishopticnerveconductiondefectsfromthe
maculardiseaseinpatients
withotherwiseunexplainedlossof
centralvisionwefirstdeterminedthebestvisualacuitywith
correctionatdistanceinunilateraldefects.
>Thenormaleyewastestedfirstandphotostressedforten
seconds
bylookingatanordinarypenlightheld2to3cmfromthe
eye.
>ThetimerequiredtoreadthreelettersonthreeSnellentestlines
justlargerthanthebestacuitywasusedastheendpoint.
>Recoverytimewillbeprolongedwithmaculopathy.
>Prolongedrecoverytimewillnotbeobservedinpatientswho
haveopticnervedisease.

https://www.ncbi.nlm.nih.gov/pubmed/836667

134.Whichlayerofcorneahelpsinthe
hydrationofstromaofcornea
a)Endothelium
b)Epithelium
c)Descemetmembrane
d)Stroma
CorrectAnswer-A
Ans:a.Endothelium
>Thecornealendotheliumisresponsibleformaintainingthe
hydrationofthecornea.
>Thisisthrougha"Pump-Leak"mechanismwheretheactive
transportpropertiesoftheendotheliumrepresentthe"Pump"and
thestromalswellingpressurerepresentsthe"Leak"
Ref:https://www.researchgate.net/publication/51238307_Molecular_Mechanisms_Underlying_the_Corneal_Endothelial_Pump

135.A65oldmalewithahistoryof
hypertensionanddiabetes,presentsto
theOPDwithcomplaintsofdiplopiaand
squint.Onexamination,thesecondary
deviationismorethantheprimary
deviation.Whichofthefollowingismost
likelydiagnosis

a)concomitantsquint
b)paralyticsquint
c)Restrictivesquint
d)Pseudosquint
CorrectAnswer-B
Ans:b.paralyticsquint.
>Paralyticorincomitantsquintoccurswhenthereisanacquired
defectofthemovementofaneye.
>Thesquint(anddoublevision)ismaximallydemonstratedby
lookinginthedirectionofactionoftheweakenedmuscle.
>ParalyticsquintsoccurduetodiseaseoftheIII,IVandVIcranial
nerves.
Ref:https://www.gponline.com/basics-
strabismus/ophthalmology/article/1055827

136.Esotropiaiscommonin
a)Myopia
b)Hypermetropia
c)Emmetropia
d)Astigmatism
CorrectAnswer-B
And:B.Hypermetropia
>Patientswithrefractiveesotropiaaretypicallyfarsighted
(hyperopic).
>Thismeansthattheeyesmustworkhardertoseeclearly,
particularlywhentheobjectofregardisupclose.
https://aapos.org/glossary/accommodative-esotropia

137.Anextrarowofciliaposteriortothegrey
line
a)Distichiasis
b)Tylosis
c)Madarosis
d)Trichiasis
CorrectAnswer-A
And:a.Distichiasis
>Distichiasisisararedisorderdefinedastheabnormalgrowthof
lashesfromtheorificesofthemeibomianglandsontheposterior
lamellaofthetarsalplate.
Ref:https://emedicine.medscape.com/article/1212908-overview

138.Thepatientcamewithunilateral
ProptosisandbilateralAbducentnerve
palsy.Thiscouldbefrom

a)Cavernoussinus
b)Orbitalcellulitis
c)Orbitalpseudotumor
d)Orbitallymphoma
CorrectAnswer-A
Ans:a.Cavernoussinus
>Proptosisisinitiallyseenunilaterallyultimatelybecomesbilateral.
>6thcranialnerve(Abducent)passesthroughthecavernous
sinus.soincavernoussinusthrombosisbilateralabducenspalsyis
seen.
Parson'sDiseasesoftheEye-Ed.22Pg497

139.Thecharacteristicfindingoffungal
ulcers?
a)Satellitelesions
b)Dendriticulcer
c)Ringabscess
d)Whitehypopyon
CorrectAnswer-A
And.a.Satellitelesions
>Fungalkeratitisorkeratomycosisreferstoaninfectiveprocessof
thecorneacausedbyanyofthemultiplepathologicfungicapableof
invadingtheocularsurface.
>whentheepithelialintegrityisbrokeneitherduetotraumaorocular
surfacediseaseandtheorganismgainsaccessintothetissueand
proliferates.
>Multifocalstromalinfiltratesor"satellitelesions"havebeen
consideredacharacteristicfeatureoffungalkeratitis.
Ref:https://eyewiki.aao.org/Fungal_Keratitis

140.Adrugusedinapatientwithincreased
IOPandopticdiscchanges,ciliary
congestionfordecreaseIOPactsby
increasinguveoscleraloutflowis

a)Latanoprost
b)Pilocarpine
c)Dorzolamide
d)Timolol
CorrectAnswer-A
Ans:a.Latanoprost
>LatanoprostselectivelystimulatestheprostaglandinF2alpha
receptorandthisresultsinadecreasedintraocularpressure(IOP)
viatheincreasedoutflowofaqueoushumor,whichisoften
implicatedincasesofelevatedintraocularpressure.
Ref:https://www.google.com/url?
sa=t&source=web&rct=j&url=https://www.drugbank.ca/drugs/DB00654&ved=2ahUKEwicmYHW5PjmAhU94HMBHZG6ADcQFjANegQIAxAB&usg=AOvVaw34z-
Nz08_cOFzeYemAzeWf&cshid=1578651404060

141.ApatientdiagnosedwithRheumatoid
arthritiswasonmedications.After2
years,hedevelopedablurringvisionand
wasfoundtohavecornealopacity.
Whichdrugismostlikelytocausethis?

a)Sulfasalazine
b)Chloroquine
c)Methotrexate
d)Leflunomide
CorrectAnswer-B
Ans:B.Chloroquine
Inlong-termchloroquinetherapyOcularcomplicationswere
observed
>Thistherapyisusuallyusedinpatientswithrheumatoidarthritis,
lupuserythematosus,sarcoidosis,discoidlupus,andotherchronic
"collagendisease".
>Retinalchanges,cornealopacifications,blurringofvisionaresome
complicationsseeninlongtermuseofChloroquine.
Ref:https://www.ncbi.nlm.nih.gov/pubmed/1427503

142.Nutcrackeresophagus,thecorrect
statementis
a)Thereisextremelyforcefulperistalticactivityleadingto
episodesofchestpainanddysphagia
b)Thereisnomedicalt/tavailable
c)TypeofoesophagealMalignancy
d)None
CorrectAnswer-A
Ans:A.Thereisextremelyforcefulperistalticactivityleadingto
episodesofchestpainanddysphagia
NutCrackerOesophagusisahypermotilitydisorderwithhigh
amplitudeperistalsis.Itisaconditioninwhichextremelyforceful
peristalticactivityleadstoepisodicchestpainanddysphagia.
Treatmentiswithnitratesornifedipine.
Ref.davidsonsprinciplesandpractice23rdeditionpageno.795

143.Whichofthefollowingcriteriaisusedto
assesstheprognosisoftheliver
conditionasrepresentedinthepicture
below?

a)Childpughscore
b)Milanscore
c)Meldscore
d)Alvaradoscore
CorrectAnswer-A
Ans:A.Childpughscore
theliverconditionasrepresentedinthepictureabovedemonstrates
livercirrhosis.
Child-Pughscore(ortheChild?Turcotte?PughscoreorChild
Criteria)isusedtoassesstheprognosisofchronicliverdisease,
mainlycirrhosis.
Itisnowusedtodeterminetheprognosis,aswellastherequired
strengthoftreatmentandthenecessityoflivertransplantation.
Factor
1
2points
3points
point
Totalbilirubin
<34 34-50
>50
(mol/L)
Serumalbumin >35 28-35
<28
(g/L)
PTINR
<1.7 1.71-2.30
>2.30
Ascites
None Mild
Moderateto
Severe
Hepatic
None GradeI-II(orsuppressed
GradeIII-IV(or

Hepatic
None GradeI-II(orsuppressed
GradeIII-IV(or
encephalopathy
withmedication)
refractory)
Interpretation:
Points Class Oneyearsurvival Two-yearsurvival
5?6
A
100%
85%
7?9
B
81%
57%
10?15 C
45%
35%

144.Suddenonsetheadachewithneck
rigidity?
a)Intraparenchymalhemorrhage
b)Sah
c)Meningitis
d)Noneoftheabove
CorrectAnswer-B
Ans:b.Sah
suddenonsetofasevereheadache(oftendescribedas"theworst
headacheofmylife")
nauseaandvomiting
stiffneck
sensitivitytolight(photophobia)
blurredordoublevision
lossofconsciousness
seizures

145.Vegetationinmitralvalveseeninwhich
condition
a)Libmansacks
b)Infectiveendocarditis
c)NBTE
d)Rheumaticfever
CorrectAnswer-B
Ans:B.Infectiveendocarditis
Infectiveendocarditisischaracterizedbylesions,knownas
vegetations,whichisamassofplatelets,fibrin,microcoloniesof
microorganisms,andscantinflammatorycells.Inthesubacuteform
ofinfectiveendocarditis,thevegetationmayalsoincludeacenterof
granulomatoustissue,whichmaybefibrosedorcalcified.

146.Waterhammerpulseisseenin
a)Aorticregurgitation
b)Mitralstenosis
c)Aorticstenosis
d)Leftventricularfailure
CorrectAnswer-A
Ans:A.Aorticregurgitation
Waterhammerpulse

Itisalargeboundingpulse,associatedwithanincreasedstroke
volumeoftheleftventricleandadecreaseintheperipheral
resistance,leadingtowidepulsepressure.
Thepulsestrikesthepalpatingfingerwitharapid,forcedjerkand
quicklydisappears.
Itisbestfeltintheradialarterywiththepatient'sarmelevated.
Itisseeninaorticregurgitation.


147.Whatwillyoudowhen3yrsoldchild
parentscometophcwithfever,cough
since5dayswithchestindrawing
presentundermnciclassification

a)Giveantipyreticsonly
b)Giveantibioticsandfollowup
c)Referurgentlytotertiarycare
d)Giveantibioticsandrefertotertiarycare
CorrectAnswer-D
Ans:D.Giveantibioticsandrefertotertiarycare


148.Whatpoisonwillyoudetectinskeleton
evenafteremaciation
a)Lead
b)Arsenic
c)Mercury
d)Cadmium
CorrectAnswer-B
Ans:B.Arsenic
Arsenic
canbedetectedeveninconflagratedhumanbones.Hence,
itispossibletodetectpoisoningbyarsenicevenwhenthebodyhas
beenburntforalongtime.
FMT2ndeditionPageno.184

149.Ph7.2,HCO3-(10or12),pco2-35,
metabolicacidosisdueto
a)K+excretionbythekidney
b)Co2expirationbylungs
c)H+excretionbythekidney
d)Hco3lossbykidney
CorrectAnswer-D
Ans:D.Hco3lossbykidney
Inthequestion,pHisdecreased(acidosis)&pCO2isdecreased
(Normal=40-45mmHg).
AdecreasedpCO2willtrytoincreasepH,henceitmustbea
secondarycompensatingmechanism.
So,theprimarymechanismcausingtheacid-baseimbalancemust
beadecreaseinserumbicarbonateconcentrationi.e.metabolic
acidosis.

150.ImpairedfunctionofAquaporinresultsin
a)Liddelsyndrome
b)NephrogenicDI
c)Cysticfibrosis
d)Bartersyndrome
CorrectAnswer-B
Ans:B.NephrogenicDI
Aquaporins,alsocalledwaterchannels,areintegralmembrane
proteinsfromalargerfamilyofmajorintrinsicproteinsthat
formporesinthemembraneofbiologicalcells,mainlyfacilitatingthe
transportofwaterbetweencells.
Geneticdefectsinvolvingaquaporingeneshavebeenassociated
withseveralhumandiseasesincludingnephrogenicdiabetes
insipidusandneuromyelitisoptica.

151.Mostseriouscomplicationofmeaslesis:
September2008
a)Croup
b)Meningo-encephalitis
c)Otitismedia
d)Pneumonia
CorrectAnswer-B
Ans.B:Meningo-encephalitis
Measlesisahighlycommunicableacutedisease.Itisalsoknownas
rubeolaandismarkedbyprodromalfever,cough,coryza,
conjunctivitis,andpathognomonicenanthem(i.e.,Koplikspots),
followedbyanerythematousmaculopapularrashonthethirdto
seventhday.
Infectionconferslife-longimmunity.
Ageneralizedimmunosuppressionthatfollowsacutemeasles
frequentlypredisposespatientstobacterialotitismediaand
bronchopneumonia.
Inapproximately0.1%ofcases,measlescausesacutemeningo-
encephalitis,whichisthemostseriouscomplication.Subacute
sclerosingpanencephalitis(SSPE)isararechronicdegenerative
diseasethatoccursseveralyearsaftermeaslesinfection.

152.ThemostcommoncauseofdeathinSLE
inchildren
a)Lupusnephritis
b)Lupuscerebritis
c)Libmansacksendocarditis
d)Anemiaandinfections
CorrectAnswer-A
Ans:A.Lupusnephritis
MajorcausesofdeathinpediatricSLEinclude:

1. Renaldisease(lupusnephritis)
2. Severediseaseflare
3. Infections

153.ApatienthavingmultipleGallstonesand
shows8mmdilationand4stonesin
CBD,besttreatmentmodalitiesare?

a)Cholecystectomywithcholedocholithotomyatthesamesetting
b)ESWL
c)CholecystectomyandwaitforERCP
d)All
CorrectAnswer-A
Ans:A.Cholecystectomywithcholedocholithotomydoneatthe
samesitting
ManagementofsuspectedorprovenCBDstonesassociatedwith
gallbladderstones
Forgallstones-laparoscopiccholecystectomyistheprocedureof
choice.
ForCBDstonestwothingscanbedone:
1)Ifthesurgeonisexperiencedinlaparoscopictechniquesof
CBDstoneremovalthenbothcholecystectomyand
choledocholithotomyaredoneinthesamesitting.
CBDstonesarefirstconfirmedbyanintraoperativecholangiogram
thenthestonesareremovedlaparoscopicallyviathecysticductor
bycholedochotomy.
2)Ifthesurgeonisnotexperiencedwithlaparoscopicmethodsof
CBDstoneremoval,preoperativeendoscopicsphincterotomywith
stoneremovalandlaterlaparoscopiccholecystectomyisdone.
Laparoscopiccholecystectomywithcholedocholithotomyinthe
samesittingisthepreferredtechnique(providedthesurgeonis
experienced)

Butonemustkeepinmindherethat
"forelderly,poor-riskpatientswithgallstonesandCBDstones,some
haverecommendedERCPandsphincterotomyasthesole
treatment,leavinggallbladderandstonesinsitu".-Maingot's10/e
Usuallythegallstonesinthesepatientsremainasymptomaticandif
theneedarisescanbemanagedbycholecystectomy

154.Pulmonaryplethoraisseenwithall
except:
a)TGS
b)Ebsteinanomalies
c)Hypoplasticleftheartsyndrome
d)Doubleoutletrightventricle
CorrectAnswer-B
Ans:B.Ebsteinanomalies
Pulmonaryplethoraisseenin
TOF
TA
Ebstein'sanomaly
Pulmonaryatresia

155.20yearsoldmanpresentedwiththe
complaintofswellingofthewristforthe
lasttwoyears.histopathological
examinationshowedspindle-shaped
cellsandverocaybodieswhatisthe
diagnosis?

a)Lipoma
b)Dermoidcyst
c)Neurofibroma
d)Schwannoma
CorrectAnswer-D
Ans:D.Schwannoma
spindlecellsandverocaybodiesareseeninschwannoma
isatumorofthetissuethatcoversnerves,calledthenervesheath.
ThesetumorsdevelopfromatypeofcellcalledaSchwanncell,
whichgivesthemtheirname.Schwannomasareoftennot
cancerous(benign).
Themostcommontypeofschwannomaisvestibularschwannoma.
Itaffectsthenerveresponsibleforbalance(alsocalledthevestibular
nerve).Itcancauseinnereardeafnessbecausethehearingand
balancenerveruntogether,andasthetumorgrowsitdamagesthe
hearingnerve(cochlearnerve).Becauseofthisdoctorusedtocall
themacousticneuromas.
Microscopically,thetumoriscomposedoffibrocellularbundles
formingthewhorledpattern.
Thereareareasofdenseandcompactcellularity(AntoniA

pattern)alternatingwithlooseacellularareas(AntoniB
pattern).AreasofAntoniApatternshowpalisadednuclei
calledVerocaybodies.
Nervefibersareusuallyfoundstretchedoverthecapsulebutnot
withinthetumor.
Areasofdegenerationcontainhaemosiderinandlipid-laden
macrophages.
SchwanncellscharacteristicallyexpressS-100protein.
Aschwannomararelyeverbecomesmalignant.


156.Hemodynamicallyunstablepatientwith
SVT
a)IVIBUTILIDE
b)IVDILTIAZEM
c)CARDIOVERSION
d)Ivbeta-blockers
CorrectAnswer-C
Ans:C.CARDIOVERSION


157.AFemalepatientwasonlithiumfor
bipolardisorderfor6months.shefasted
forsomedaysduetoreligiouscondition
andlaterpresentedwith
seizures,tremors,confusionand
weakness.Whatinvestigationshaveto
bedonetodiagnosehercondition

a)serumelectrolytes
b)serumlithium
c)ECG
d)MRI
CorrectAnswer-B
Ans:B.serumlithium
Becauseofitseffectsonmultiplebodysystems,includingbutnot
limitedtorenal,hematologic,andthyroidsystems,appropriate
baselinestudiesmustprecedethestartoflithiumtherapy.
Theminimalteststobedoneincludeserumcreatinine,electrolytes,
thyroidfunctiontests,andacompletebloodcountwithdifferential.
Additionally,becauseofitscardiaceffect,anECGisindicated.Any
womanthoughttobepregnantshouldhaveapregnancytest.
Ref:NurcombeB.(2008).Chapter12.DiagnosticFormulation,
TreatmentPlanning,andModesofTreatmentinChildrenand
Adolescents.InM.H.Ebert,P.T.Loosen,B.Nurcombe,J.F.
Leckman(Eds),CURRENTDiagnosis&Treatment:Psychiatry,2e.

158.Lithiumcauses:
a)Hypokalemia
b)Hyperkalemia
c)Hypocalcemia
d)Hypercalcemia
CorrectAnswer-A
Ans:A.Hypokalemia
Sideeffectsoflithium

1. Neurological:-Tremoristhecommonestsideeffectoflithium.Other
CNSsideeffectsaregiddiness,ataxia,motorincoordination,
hyperreflexia,mentalconfusion,nystagmus.
2. Renal:-Nephrogenicdiabetesinsipiduswithpolyuria&polydipsia.
AmilorideistheDOCforLithiuminducednephrogenicDI.
3. Cardiovascular:-Effectsaresimilartohypokalemia.Themost
commonECGchangeisTwavedepression.
4. Endocrine:-Goitre,hypothyroidism
5. GIT:-Nausea,vomiting,diarrhea,metallictest,abdominalpain.
6. Dermatological:-Acneiformeruptions,papulareruption,
exacerbationofpsoriasis.
7. Teratogenicity:-Ebstein'sanomalyinthefetus.

159.A50-year-oldmanpresentswith
paresthesia.HB-6.8g/dl.Peripheral
smearshowsmacrocytosisand
neutrophilswithhypersegmented
nuclei.endoscopyrevealsatrophic
gastritis.amostprobablediagnosisis

a)Folatedeficiency
b)VitaminB12deficiency
c)Riboflavindeficiency
d)Irondeficiency
CorrectAnswer-B
Ans:B.VitaminB12deficiency
Macrocytosisandhypersegmentedneutrophilsonperipheralsmear,
poorabsorption(Atrophicgastritis)suggestthediagnosisofB12
deficiency.
Thepresenceofanemia(Hb=6gm%)Macrocytosis(MCV=104)and
decreasedvitaminB12levels(B12=60pg/ml)suggestsadiagnosis
ofmegaloblasticanemiaduetovitaminB12deficiency.
Thepatientinquestionthushasmacrocyticanemiaduetovitamin
B12deficiency.

160.Alltransretinoicacidisusedinthe
treatmentoftumourassociatedwith
a)BCR-ABL
b)PML-RARA
c)CMYC
d)CEBPA
CorrectAnswer-B
Ans:B.PML-RARA
All-transretinoicacid(ATRA)isanactivemetaboliteofvitaminA
underthefamilyretinoid.
Retinoids,throughtheircognatenuclearreceptors,exertpotent
effectsoncellgrowth,differentiationandapoptosis,andhave
significantpromiseforcancertherapyandchemoprevention.
DifferentiationtherapywithATRAhasmarkedamajoradvanceand
becomethefirstchoicedruginthetreatmentofacutepromyelocytic
leukemia(APL).
All-transretinoicAcidisusedinthetreatmentforAcute
PromyelocyticLeukemia(PML)

161.RenaltubularacidosiswithABGvalue
pH=7.24P02=80;PaCO2=36Na=131;
HCO3=14Cl=90;BE=-13Glucose=
135aboveABGpicturesuggest?

a)Respiratoryacidosis
b)Respiratoryalkalosis
c)Metabolicacidosis
d)Metabolicalkalosis
CorrectAnswer-C
Ans:C.Metabolicacidosis
ThegivenvalueshavelowpH,andlowHCo3Indicatemetabolic
acidosis
PCO2inlowernormalrange(normalvalue35-45mm/hg)

162.Patientwithpulmonaryfibrosis.Which
antiarrhythmicdrugshouldnotbegiven
a)Amiodarone
b)Flecainide
c)Ivibutilide
d)lidocaine
CorrectAnswer-A
Ans:A.Amiodarone
Amiodaroneanditsmetabolitescanproducelungdamagedirectly
byacytotoxiceffectandindirectlybyanimmunologicalreaction.
ThelatterissupportedbythefindingofcytotoxicTcellsin
bronchoalveolarlavage(BAL)fluidfrompatientswithdiagnosed
APT.AmiodaronemayinducetheproductionoftoxicO2radicals,
whichcandirectlydamagecells

163.A42-year-oldpatientwithobstructive
jaundice.Alp,Ggt,haptoglobinall
increased.Themostlikelycauseis:

a)Alcohol
b)Lead
c)Chronicrf
d)Noneoftheabove
CorrectAnswer-A
Ans:A.Alcohol
Anindicatorthatshouldmaketheclinicianhighlysuspiciousof
alcohol-relatedliverinjuryisAST:ALTratioof2:1ormore.
Gamma-glutamyltransferase(GGT)isanothersensitivebutnon-
specificmarkerforthehepaticinjurywhichcannotbeusedsolelyto
diagnosealcohol-relatedhepaticinsult.
LevelsofGGTgreaterthantwicethenormalvaluesinadditionto
AST:ALTratio>2stronglyindicatealcohol-inducedliverinjuryas
well.

164.ThedifferentiatingfeaturebetweenIBS
andorganicGIdiseaseis:
a)Diarrhea
b)Stoolcalprotectin
c)Painabdomen
d)Mucusinstools
CorrectAnswer-B
Ans:B.Stoolcalprotectin
BothorganicIBDandnon-organicfunctionaldisorderslikeIBS
exhibitverysimilarsymptomsresearchershaveidentifiedseveral
stool-basedbiomarkerstodifferentiatebetweenIBDandIBS.These
becomeespeciallycrucialwhentakingthenextsteptowards
decidinghowtomanagethedisease(e.g.therapeuticintervention,
etc.).Thosebiomarkersincludethefollowing:
Calprotectin
Alpha-1Antitrypsin
Lysozyme
SecretoryIgA
Albumin

165.Atienthasfatigue.Butnotgaining
weight.Bodywaswarm.Investigation
willshow:

a)LowTSHwithmoret3ort4
b)HighTSHwithnormalt3ort4
c)HighTSHwitheuthyroid
d)Increaseduptakeoft3,butdecreaset4
CorrectAnswer-A
Ans:A.LowTSHwithmoret3ort4

166.WarminginFrostfrostbiteshouldbe
doneatwhattemperature:
a)37degree
b)42degree
c)44degree
d)46degree
CorrectAnswer-A
Ans:A.37degree
Attemperaturebelowfreezing(dry-coldcondition)frostbiteoccurs
thetissuefreeze&icecrystalsforminbetweenthecells
Affectedpartshouldbewarmedusingwaterat44?isunder
Criticism,Warmingshouldlastabout20minutesatatime,Intakeof
hotfluidspromotesgeneralrewarming

167.Whichofthefollowingisnotseenin
Pituitaryapoplexy
a)Headache
b)Hypertension
c)Hypotension
d)Vomiting
CorrectAnswer-C
Ans:C.Hypotension
FollowingareseeninPituitaryapoplexy
Severehypoglycemia
Severeheadache(usuallyretro-orbital)
Impairedconsciousness
Fever
Visualdisturbances(visualfielddefect,visualacuity)
Ophthalmoplegia(ocularparesis)Causingdiplopia
Hypotension&shock
Nausea/vomiting
Meningealsign

168.Lossofpain/temperaturesensationon
ipsilateralface&C/Lbodydueto
thrombosisin

a)PICA
b)Posteriorcerebellarartery
c)Superiorcerebellarartery
d)Noneoftheabove
CorrectAnswer-A
Ans:A.PICA
.Mostcasesresultfromipsilateralvertebralarteryocclusion;
occlusionoftheposteriorinferiorcerebellararteryisresponsiblefor
it.
VesselocclusionthatresultsinLateralMedullarysyndrome:
Vertebral(mostcommon)
Posteriorinferiorcerebellar(2ndmostcommon)
Superior,middleorInferiorlateralmedullaryarteries
Lateralmedullarysyndrome(Wallenbergsyndrome):
Vertigo
Numbnessofipsilateralfaceandcontralaterallimbs
Diplopia
Dysphagia
Dysarthria
Ataxia
Hoarseness

169.AllfoundinLVFexcept
a)Lungoligemia
b)Kerleyblines
c)Rales
d)Pedaledema
CorrectAnswer-A
Ans:A.Lungoligemia
ChestX-rayFeaturesofLeftventricularfailure:
Cardiomegaly
Kerleylines
Prominentupperlobeveins
Bat'swingsshadow
Pleuraleffusion
KerleyBlineisduetointerstitialedema.
Prominentrightdescendingpulmonaryarteryisseeninacute
pulmonaryembolism.

170.Allofthefollowingshowlowglucosein
pleuralfluid,EXCEPT-
a)Empyema
b)Malignantpleuraleffusion
c)Rheumatoidarthritis
d)Dressler'ssyndrome
CorrectAnswer-D
AnswerisD(Dressler'sSyndrome):
Dresslersyndromeisasecondaryformofpericarditisthatoccurs
inthesettingofinjurytotheheartorthepericardium(theouterlining
oftheheart).Itconsistsoffever,pleuriticpain,pericarditisand/or
pericardialeffusion.
Thediseaseconsistsofpersistentlow-gradefever,chest
pain(usuallypleuritic),pericarditisand/orpericardialeffusion.The
symptomstendtooccur2?3weeksaftermyocardialinfarctionbut
canalsobedelayedafewmonths.Ittendstosubsideinafewdays,
andveryrarelyleadstopericardialtamponade.ElevatedESRisan
objectivebutnonspecificlaboratoryfinding.
Dressler'sSyndromeisnotassociatedwithlowglucoseinpleural
fluid.
RheumatoidArthritis,MalignancyandEmpyema(Bacterial
infections)areallestablishedcausesofpleuraleffusionwithlow
glucose.
Pleuraleffusionwithlowglucose(<60mg/c11)
1. Malignancy
2. Bacterialinfections
3. Rheumatoidpleuritis

171.A25yearsoldladywithahistoryoffever
for1monthpresentswithheadacheand
ataxia.Brainimagingshowsdilated
ventriclesandsignificantbasal
exudates.Whichofthefollowingwillbe
themostlikelyCSFfinding:

a)Lymphocytosis,LowGlucose,Highprotein
b)Lymphocytosis,NormalGlucose,Highprotein
c)Lymphocytosis,LowGlucose,Normalprotein
d)Neutrophilia,Lowglucose,LowProtein
CorrectAnswer-A
Ans:A.(Lymphocytosis,LowGlucose,Highprotein)
Presenceofsignificantbasalexudates,togetherwithdilated
ventricles(hydrocephalus)inayoungfemalewith
aprolongedhistoryoffeverandheadachesuggestsadiagnosisof
TubercularMeningitis.
TubercularMeningitisischaracterizedbyLymphocyticPleocytosis,
LowGlucoseandHighProteinwithintheCSF.
ThepathologicalhallmarkofTubercularMeningitisisthe
predominantinvolvementofbasalcisternsthatareobservedbythe
presenceofbasalinflammatorytissueexudate.

172.Whichofthefollowingdrugcanbegiven
inpatientsofprimarypulmonary
hypertension?

a)Icatibant
b)Bosentan
c)Labetolol
d)Sodiumnitroprusside
CorrectAnswer-B
Answer-B.Bosentan
TreatmentforPrimarypulmonaryhypertension:
CALCIUMCHANNELBLOCKERS:
Patientswhorespondtoshort-actingvasodilatorsatthetimeof
cardiaccatheterizationshouldbetreatedwithcalciumchannel
blockers.
Theendothelinreceptorantagonistsbosentanandambrisentanare
approvedtreatmentsofPAH
Bosentaniscontraindicatedinpatientswhoareoncyclosporineor
glyburideconcurrently.

173.ApatientarrivedinERfollowinganRTA
withhypotension,respiratorydistress
andsubcutaneousemphysemawithno
entryofairononeside.Whatwillbethe
bestmanagement?

a)Needledecompressionin5thintercostalspaceinthe
midaxillaryline
b)ContinuePPV
c)ShifttoICUandincubate
d)SecureIVlineandstartfluidresuscitationafterinsertionofthe
wide-boreIVline
CorrectAnswer-A
Ans-A.Needledecompressionin5thintercostalspaceinthe
midaxillaryline
Atensionpneumothoraxdevelopswhena'one-wayvalve'airleak
occurseitherfromthelungorthroughthechestwall.
Airissuckedintothethoraciccavitywithoutanymeansofescape,
completelycollapsingthencompressingtheaffectedlung.
Etiology-
Themostcommoncausesarepenetratingchesttrauma,bluntchest
traumawithparenchymallunginjuryandairleakthatdidnot
spontaneouslyclose,iatrogeniclunginjury(e.g.duetocentral
venepuncture)andmechanicalpositivepressureventilation.
C/F-
Thepatientisincreasinglyrestlesswithtachypnoea,dyspnoeaand
distendedneckveins

Treatment-
Treatmentconsistsofimmediatedecompression,initiallybyrapid
insertionofalarge-borecannulaintothesecondintercostalspacein
themidclavicularlineoftheaffectedside,thenfollowedbyinsertion
ofachesttubethroughthefifthintercostalspaceintheanterior
axillaryline.
Ref-BaileyandLove,Shortpracticeofsurgery,27thedition
publishedin2018Pg367,920


174.TreatmentofRenalcellcarcinomaof
lessthan4cmwillbe-
a)Partialnephrectomy
b)Radicalnephrectomy
c)Radicalnephrectomy+postoperativeradiotherapy
d)Radicalnephrectomy+chemotherapy
CorrectAnswer-A
Ans:A.Partialnephrectomy
Partialnephrectomyisnowbeingusedasprimarysurgicaltherapy
forpatientswithatumorlessthan4cminsize.EarlierRadical
nephrectomywasthetreatmentofchoicefortumorsofanysize.
Classicradicalnephrectomyconsistsofremovalofthekidney,
perirenalfat,adrenalgland,andregionallymphnodes.MoststageI
andstageIItumors.
Partialnephrectomy-isindicatedforpatientswithaT1tumor
(accordingtotheUICCTNMstagingsystem)andanormal
contralateralkidney.
Interventionshouldbeconsideredforgrowthto>3-4cmorby
>O.5cmperyear.


StageT1aN0M0
Partialnephrectomyrecommended.Thiscanbedonevia
open/laparoscopic/roboticprocedures
StageT1bN0M0
PN(open/laparoscopic/robotic)incaseswheretechnicallyfeasible
LaparoscopicRNshouldbeofferedifaPNisnotfeasible
OpenRNiflaparoscopicsurgeryisnotpossible.
StageT2N0M0
RN?open/laparoscopic/robotic
PN?open/laparoscopic/robotic
StageT3
RN?open,laparoscopicorrobotic-assisted
Resectionofvascularthrombosiswhenapplicable(usuallyopen)
Resectionofallgrossdiseaseincludinghilarorretroperitoneal
extension
PNmaybeattemptedinhighlyselectedcasesbyexperienced
surgeons
Ref-BaileyandLove,Shortpracticeofsurgery,27thedition
publishedin2018Pg1420
https://www.researchgate.net/publication/263933944_Surgical_management_of_renal_cell_carcinoma_Canadian_Kidney_Cancer_Forum_Consensus


175.WhichisnotseeninAsepsisscore-
a)Erythema
b)Induration
c)Serousdischarge
d)Purulentexudate
CorrectAnswer-B
Answer-B.Induration
Criterion
Points
Additionaltreatment
0
Antibioticsforwoundinfection
10
Drainageofpusunderlocalanaesthesia
5
Debridementofwoundundergeneral
10
anaesthesia
Serousdischarge
Daily0-5
Erythema
Daily0-5
Purulentexudate
Daily0-10
Separationofdeeptissues
Daily0-10
Isolationofbacteriafromwound
10
Stayasinpatientprolongedover14days
5
asresultofoundinfection
Ref-BaileyandLove,Shortpracticeofsurgery,27thedition
publishedin2018Pg48


176.Esophagealmanometrywasperformed-
itrevealedpanesophagealpressurization
withdistalcontractileintegrityas
>450mmHgpressureinthebody.What
willbethediagnosis?

a)Type1achalasia
b)Type2achalasia
c)Type3achalasia
d)Jackhammeresophagus
CorrectAnswer-C
Ans:C.Type3achalasia
TypeIachalasia(classicachalasia)-
ElevatedmedianIRP(>15mmHg),100%failedperistalsis(DCl
<100mmHg.s.cm)
PrematurecontractionswithDCIvalues<450mmHg.s.cmsatisfy
criteriaforfailedperistalsis
TypeIIachalasia(withesophagealcompression)-
ElevatedmedianIRP(>15mmHg),100%failedperistalsis,
panesophageal
pressurisationwith>20%ofswallows.
Contractionsmaybemaskedbyoesophagealpressurizationand
DCIshouldnotbecalculated.
Typelllachalasia(spasticachalasia)
ElevatedmedianIRP(>15mmHg),nonormalperistalsis,premature
(spastic)
contractionswithDCI>450mmHg.s.cmwith>20%ofswallows

Maybemixedwithpanesophagealpressurization
'Diffuseesophagealspasm'andjackhammer(nutcracker)
esophagus
-
Spasticpressuresonmanometryof400-500mmHgmarked
hypertrophyofthecircularmuscleandacorkscrewesophaguson
bariumswallow.
Ref-BaileyandLove,Shortpracticeofsurgery,27thedition
publishedin2018Pg1096,1099


177.Whatwillbetheappropriate
managementforAbdominalaortic
aneurysm-

a)Monitortillsizereaches55mm
b)Immediatesurgery
c)USGmonitoringtill>70mmasymptomatic
d)Notreatment
CorrectAnswer-A
Answer-A.Monitortillsizereaches55mm
Anasymptomaticabdominalaorticaneurysm
inanotherwisefit
patientshouldbeconsideredforrepairif>55mmindiameter
(measuredbyultrasonography).
Regularultrasonographicassessmentisindicatedforasymptomatic
aneurysms<55mmindiameter.
75%ofaneurysmsaresuitableforendovascular(minimallyinvasive)
repair,usuallyviathefemoralarteriesinthegroin.
Rupturedabdominalaorticaneurysmisasurgicalemergency.
SymptomaticAAA-
Anoperationisusuallyindicatedinpatientswhoareotherwise
reasonablyfit.Painmaybeawarningsignofstretchingofthe
aneurysmsacandimminentrupture;surgeryshouldbeperformed
assoonaspossible(usuallyonthenextavailableoperatinglist).
Ref-BaileyandLove,Shortpracticeofsurgery,27thedition
publishedin2018Pg961



178.WhatistheTstageofa2.5cmlung
carcinoma,notinvolvingthepleura?
a)T1a
b)T2
c)T1b
d)T1c
CorrectAnswer-D
Ans:D.T1c
Ref-BaileyandLove,Shortpracticeofsurgery,27thedition

publishedin2018Pg927

179.In,theprimarysurveywhichisnot
included-
a)CECTtolookforbleeding
b)Exposureofthewholebody
c)ABC
d)RecordingBP
CorrectAnswer-A
Answer?A.CECTtolookforbleeding
Theprimarysurveyaimstoidentifyandmanagethemost
immediatelylife-threateningpathologiesfirstandfollowscABCDE.
1.c-Exsanguinatingexternalhemorrhage
2.A-airwaywithcervicalspinecontrol
3.B-Breathing&ventilation
4.C-Circulation&hemorrhage
5.D-Disability
6.E-Exposure
Anassessmentofthehemodynamicstatusshouldbemadeto
identifyshockedpatients:theskinmaybepale,coolandsweaty,the
pulserateraisedtoover100perminuteandthebloodpressure
low.
Ref-BaileyandLove,Shortpracticeofsurgery,27thedition
publishedin2018Pg323


180.MClocationofgastrinomainMEN-1
syndrome?
a)Duodenum
b)Jejunum
c)Pancreas
d)Ileum
CorrectAnswer-A
Answer-A.Duodenum
PETsoccurinaround50-60VoofMEN1patients.
Themostcommonfunctionaltumourisgastrinomainduodenum
followedbyinsulinoma.
Ref-BaileyandLove,Shortpracticeofsurgery,27thedition
publishedin2018Pg856


181.Patientshaveprecancerouslesionswith
abdominalswellingandinguinalnodes
areseen.Onexamination,
lymphadenopathywasfound.Themost
probablecarcinomarelatedtothis
conditionwillbe?

a)capenis
b)CaTestis
c)caprostate
d)cabladder
CorrectAnswer-A
Answer-A.capenis
Cancerpenisspreadstoinguinalnodes.
Amass,pruritusordischargeiscommon.Advancedtumoursmay
causefecalincontinencebytheinvasionofthesphincters.
Analcanaltumoursarepalpableandirregularinduratedtender
ulceration.Sphincterinvolvementmaybeevident.
Ref-BaileyandLove,Shortpracticeofsurgery,27thedition
publishedin2018Pg1371


182.Amanunderalcoholintoxicationhad
fallenintoamanholeandhadaperineal
injurywithswollenscrotumandupper
thighalongwithbloodatmeatus.The
patientishavingdifficultypassingurine
aswell.Whatwillbetheinjury
associatedduetothistrauma?

a)Bladderrupture
b)Penilefracture
c)Bulbarurethra
d)Membranousurethra
CorrectAnswer-C
Answer-C.Bulbarurethra
Thereisahistoryofablowtotheperineum,usuallyduetoafall
astrideinjury.Thebulbarurethraiscrushedupwardsontothepubic
bone,typicallywithsignificantbruising.
Cyclingaccidents,loosemanholecoversandgymnasiumaccidents
astridethebeamaccountforanumberofcases.C/F
Thesignsofarupturedbulbarurethraareperinealbruisingand
hematoma,typicallywithabutterflydistribution.Thereisusually
bleedingfromtheurethralmeatusandretentionofurineisalso
typicallypresent.
Ref-BaileyandLove,Shortpracticeofsurgery,27thedition
publishedin2018Pg1479,1480


183.Patientwithahistoryofcarcinoma
bladderpresentingwithdyspnoeawith
clinicalsignsofDVTandtachycardia.
Theriskforthepatienttodevelop
Pulmonaryembolismaccordingto
WELL'sscore

a)High
b)Medium
c)Low
d)Cannotcommentwithoutd-dimervalues
CorrectAnswer-B
Ans.B.Medium
Wellscriteria
Score
High
>6.0
Moderate
2.0to6.0
Low
<2.0
ModifiedWellscriteria Score
PElikely
>4.0
PEunlikely
<or=4.0
ModifiedWellsCriteriaforDVT
Variable
Score
Lowerlimbtraumaorsurgeryorimmobilisationin
1
plastercast

Bedriddenfor>3daysorsurgeryinthelast4weeks
1
Tendernessalongthelineoffemoralorpoplitealveins
1
Entirelimbswollen
1
Calf>3cmlargercircumferencethantheotherside
1
10cmbelowthetibialtuberosity
1
Pittingedema
1
Dilatedcollateralsuperficialveins(notvaricoseveins)
1
PreviousDVT
1
Malignancy(includingtreatmentupto6monthsago)
1
Intravenousdrugabuse
3
AlternativediagnosismorelikelythanDVT
-2
Ref-BaileyandLove,Shortpracticeofsurgery,27thedition
publishedin2018Pg988


184.After4monthsofrenaltransplantation,a
patientcanlikelytodevelopwhich
infection-

a)EBV
b)CMV
c)Candida
d)Histoplasma
CorrectAnswer-B
Answer-B.CMV
Causesofallograftdysfunction

1.Early
Primarynon-function(irreversibleischaemicdamage)
Delayedfunction(reversibleischaemicinjury)
Hyperacuteandacuterejection
Arterialorvenousthrombosisofthegraftvessels
Drugtoxicity(e.g.calcineurininhibitortoxicity)
Infection(e.9.cytomegalovirusdiseaseingraft
Mechanicalobstruction(ureter/commonbileduct)
2.Late
Chronicrejection
Arterialstenosis
Recurrenceoforiginaldiseaseinthegraft(glomerulonephritis,
hepatitisC)
Mechanicalobstruction(ureter,commonbileduct)
Ref-BaileyandLove,Shortpracticeofsurgery,27thedition
publishedin2018Pg1551


185.Thepatientispresentwithfecal
peritonitisandduringlaparotomy,a
diverticularperforationisseen.Which
stageisclassifiedaccordingto
Hinchey'sstage?

a)1
b)2
c)3
d)4
CorrectAnswer-D
Answer-D.4
Hincheyclassificationofcomplicateddiverticulitis.

1. GradeIMesentericorpericolicabscess
2. GradellPelvicabscess
3. GradelllPurulentperitonitis
4. GradelVFaecalperitonitis
Ref-BaileyandLove,Shortpracticeofsurgery,27thedition
publishedin2018Pg1274


186.Ayoungmanmetwithamotorbike
accidentandhadinjuriestoileumand
jejunum.Thereforetheentireileumand
partialjejunumwereresected.Whichof
thefollowingwouldthepatientsuffer
from

a)VitaminB12deficiency
b)Atrophicgastritis
c)Constipation
d)None
CorrectAnswer-A
Answer-A.VitaminB12deficiency
TheileumistheonlysiteofabsorptionofvitaminB12andbilesalts.
Bilesaltsareessentialfortheabsorptionoffatsandfat-soluble
vitamins.
Followingresectionoftheileum,thelossofbilesaltsincreasesand
isnotmetbyanincreaseinsynthesis.
Thejejunumistheprincipalsitefordigestionandabsorptionoffluid,
electrolytes,iron,folate,fat,proteinandcarbohydrate,butthe
absorptionofbilesaltsandvitaminB12onlyoccursintheterminal
ileum,wheretherearespecifictransporters.
Ref-BaileyandLove,Shortpracticeofsurgery,27thedition
publishedin2018Pg283,1241


187.Apatientafteraheavymealcomeswith
epigastricpain.Onexamination
tendernessandrigidityintheupper
abdomen.X-rayshowing
pneumomediastinum.Whatcanbethe
cause-

a)Spontaneousesophagealrupture
b)Penetratingforeignbodyinjurytoesophagus
c)Perforatedpepticulcer
d)Ruptureofemphysematousbulla
CorrectAnswer-A
Answer-A.Spontaneousesophagealrupture
Thisisaclinicalcaseofspontaneousesophagealrupture/
Boerhaave'ssyndrome
Theclinicalhistoryisusuallyofseverepaininthechestorupper
abdomenafteramealoraboutofdrinking.
Mackler'striad,whichconsistsof(1)vomitingfollowedby(2)chest
painand(3)subcutaneousemphysemaduetoanesophageal
rupture.
Achestradiographisoftenconfirmedwithairinthemediastinum,
pleuraorperitoneum.
InvestigationsChestX-ray--showspneumomdiastinum(`V'signof
Naclerio).
MRI/CTchest.
Totalcount.
Ref-BaileyandLove,Shortpracticeofsurgery,27thedition

publishedin2018Pg1073
SriramBhatM,SRB'sManualofSurgery,4theditionpublished
in2013,Pg856


188.Transplantationbetweenidenticaltwins-
a)Isograft
b)Allograft
c)Autograft
d)Xenograft
CorrectAnswer-A
Answer-A.Isograft
Graft:Itisthetransferoftissuefromoneareatoanotherwithoutits
bloodsupplyornervesupply.
Autograft:Itistissuetransferredfromonelocationtoanotheronthe
samepatient.
Isograft:Itistissuetransferbetweentwogeneticallyidentical
individuals,i.e.betweentwoidenticaltwins.
Allograft:Itistissuetransferbetweentwogeneticallydifferent
members,e.g.kidneytransplantation(Humantohuman)
(Homograft).
Xenograft:Itistissuetransferfromadonorofonespeciestoa
recipientofanotherspecies(Heterograft).


189.Thoracoscore,whatisnotthe
component
a)ASACLASSIFICATIONS
b)SURGERYPRIORITIES
c)Performancestatus
d)Expectedcomplicationspost-surgery
CorrectAnswer-B
Answer-B.SURGERYPRIORITIES
TheThoracoscoreisthemostwidelyusedmodeltoassesstherisk
ofoperativemortalityinthoracicpatients

Ref-BaileyandLove,Shortpracticeofsurgery,27thedition
publishedin2018Pg915


190.Retrosternalgoiterwhichistrue?
a)AllpatientsshouldundergoCTchest
b)Allpatientsrequireamediansternotomy
c)itreceivesbloodsupplyfromthoracicvessels
d)Majorityofretrosternalgoitresshouldbeoperatedimmediately
CorrectAnswer-A
Answer-A.AllpatientsshouldundergoCTchest
Retrosternalgoitretendstoarisefromtheslowgrowthofa
multinodularglanddownintothemediastinum.
Itgetsitsbloodsupplyfromthemediastinumitself,notfromthe
neck.
Investigations
AchestX-rayshowsasofttissueshadowunderthesternum.
Aradioactiveiodinestudyisdiagnostic.
CTscan/MRIisusefulandallpatientsshouldundergoit.
Surgicalremovalofretrosternalthyroidisdone.(TOC)
Commonlyitcanberemovedthroughanincisionintheneck.
Ref-BaileyandLove,Shortpracticeofsurgery,27thedition
publishedin2018Pg810
SriramBhatM,SRB'sManualofSurgery,4theditionpublished
in2013,Pg498


191.TrueforKing'sCriteriawithacute
fulminantliverfailureexcept-
a)Age
b)Jaundice<7days
c)Serumbilirubin>17.5mg/dl
d)INR>3.5
CorrectAnswer-B
Answer-B.Jaundice<7days
King'sCollegecriteriaforlivertransplantationinAHF.

APAP-associatedAHF-
pH?<?7.3
or
INR?>?6.5,serumcreatinine>3.4mg/dl,andgradeIII-IV
encephalopathy
AllothercausesofAHF
INR>6.5
Or
Threeofthefollowingvariables:
1.Age40years
2.ThecauseisnonA,nonBhepatitisoridiosyncraticdrugreaction
3.Durationofjaundicebeforeencephalopathy>7days
4.INR?>?3.5
5.Serumbilirubin>17.5mg/dl
APAP,acetaminophen;INR,internationalnormalizedratio.

192.Apatientwhowaspostedforelective
inguinalherniasurgeryhashistoryMIfor
whichheunderwentCABG.Whatwill
youdoinpreoperativeassessment?

a)History+c/e+routinelabs+angiographytolookforstent
patency
b)History+c/e+routinelabs
c)History+c/e+routinelabs+stresstest
d)History+c/e+routinelabs+V/Qscan
CorrectAnswer-C
Answer-C.History+c/e+routinelabs+stresstest
InthegivenscenariothepatienthadundergoneCABGnotan
angioplasty,sothefirstoptionisruledout.
Withroutinelabtests,thestresstestshouldbeincludedtocheckthe
cardiacfunctionaspreoperativeassessment

193.Historyoftraumawithastabinjuryto
therightlowerchestwithlowBPandlow
pulserate.ItcanbeimprovedwithIV
fluidsandafterresuscitationinthe
traumacenterpatient'sBPbecomes
normal.AchestX-rayshowedclearlung
fields.Whatwillbethenextstep-

a)EFAST
b)Keepimmediatechesttube
c)CECTabdomen
d)CECTchest
CorrectAnswer-A
Answer-A.EFAST
Investigationsaredrivenbythecardiovascularstatusofthepatient.
Intorsotrauma,thebestandmostsensitivemodalityisaCTscan
withintravenouscontrastforbluntinjuryifthepatientisstable.
Routineinvestigationintheemergencydepartmentofinjurytothe
chestisbasedonclinicalexamination,supplementedbychest
radiography.
Ultrasoundcanbeusedtodifferentiatebetweencontusionandthe
actualpresenceofblood.
Extendedfocusedassessmentwithsonarfortrauma(eFAST)is
becomingthemostcommoninvestigation.
Ref-BaileyandLove,Shortpracticeofsurgery,27thedition
publishedin2018Pg366,372


194.Inapatientwithparathyroidadenoma,
howdoweconfirmtheremovalofthe
correctglandaftersurgery?

a)50%reductioninPTHafter10mins
b)50%reductioninPTHafter5mins
c)25%reductioninPTHafter10mins
d)25%reductioninPTHafter5mins
CorrectAnswer-A
Answer-A.50%reductioninPTHafter10mins
Miamicriteria
TheIOPTHvaluesatbaseline(pre-incisionandpreexcision)andat
5and10minutesafterparathyroidectomywerereviewedaccording
totheMiamicriterion(>50%dropfromhighestbaselineIOPTHlevel
at10minutesafterexcision),criterion1(>50%dropfrompre-
incisionIOPTHlevelat10minutes),criterion2(>50%dropfromthe
highestbaselineIOPTHlevelat10minutesandfinalIOPTHlevel
withinthereferencerange),criterion3(>50%dropfromthehighest
baselineIOPTHlevelat10minutesandfinalIOPTHlevellessthan
thepre-incisionvalue),criterion4(>50%dropfromthehighest
baselineIOPTHlevelat5minutes),andcriterion5(>50%dropfrom
preexcisionIOPTHlevelat10minutes).
Ref-https://www.ncbi.nlm.nih.gov/pubmed/16702520

195.Flapcommonlyusedinbreast
reconstructionis?
a)DIEPbasedondeepinferiorepigastricvessels
b)TRAMbasedonsuperiorglutealvessels
c)Glutealflapbasedonthoracodorsalartery
d)Latissimusdorsiflapbasedontheinferiorepigastricartery
CorrectAnswer-A
Answer-A.DIEPbasedondeepinferiorepigastricvessels
DIEPflap(basedondeepinferiorepigastricvessels)
TRAMflapbasedonItiseithersuperiorpediclebasedonthe
superiorepigastricvesselsorinferiorpediclebasedontheinferior
epigastricvessels.
Latissimusdorsimuscle/myocutaneousflapItisbasedonthe
thoracodorsalartery,abranchofthesubscapularartery
Superiorglutealflapbasedonsuperiorglutealvessels.
Ref-BaileyandLove,Shortpracticeofsurgery,27thedition
publishedin2018Pg879
SriramBhatM,SRB'sManualofSurgery,4theditionpublished
in2013,Pg350,594


196.A30-year-oldmanispresentedwith
crampingglutealpainafterwalking
500m.Whichisthevesselinvolved
duringthis?

a)Arterialdiseasewithaortoiliacinvolvement
b)Arterialdiseasewithfemoralarteryinvolvement
c)Femoralvenousinsufficiency
d)None
CorrectAnswer-A
Answer-A.Arterialdiseasewithaortoiliacinvolvement
Themusclegroupaffectedbyclaudicationisclassicallyone
anatomicallevelbelowthelevelofarterialdiseaseandisusuallyfelt
inthecalfbecausethesuperficialfemoralarteryisthemost
commonlyaffectedartery(70%ofcases).
Aorto-iliacdisease(30%ofcases)maycausethighorbuttock
claudication.Buttockclaudicationinassociationwithsexual
impotenceresultingfromarterialinsufficiencyiseponymouslycalled
Leriche'ssyndrome.Itisveryrare.
Ref-BaileyandLove,Shortpracticeofsurgery,27thedition
publishedin2018Pg943


197.Whichisthemostcommonpancreatic
endocrineneoplasm?
a)Insulinoma
b)Gastrinoma
c)VIPoma
d)Glucagonoma
CorrectAnswer-A
Answer-A.Insulinoma
Insulinoma(70-80%)>Nonfunctionaltumours(30-35%)>
gastrinoma(20-25%)

198.Anelderlymanwithalong-standing
moleoverthefacewhichisincreasingin
sizeandshowingirregularborders.What
willbethediagnosis?

a)Superficialspreadingmelanoma
b)Lentigomaligna
c)Acralmelanoma
d)Nodularmelanoma
CorrectAnswer-A
Answer-A.Superficialspreadingmelanoma
Superficialspreadingmelanoma

Thisisthemostcommonpresentation(70%)usuallyarisinginapre-
existentnaevusafterseveralyearsofslowchange,followedby
rapidgrowthintheprecedingmonthsbeforepresentation.
Nodularmelanoma(NM)Nodularmelanomaaccountsforl5%olall
MM.
Typicallyappearasblue/blackpapules,l-2cmindiameter,and
becausetheylackthehorizontalgrowthphase.
Lentigomalignamelanoma-
Presentsasaslow-growing,variegatedbrownmaculeontheface,
neckorhandsoftheelderly.
Acrallentiginousmelanoma(ALM)ALMaffectsthesolesoffeet
andpalmsofhands.
Itusuallypresentsasaflat,irregularmaculeinlaterlife.
Ref-BaileyandLove,Shortpracticeofsurgery,27thedition
publishedin2018Pg610


199.Whichparameterconclusivelyrulesout
malnutrition?
a)Edema
b)Leanbodymass
c)Skinfoldthickness
d)NormalECFvolume
CorrectAnswer-B
Ans.B.Leanbodymass
Measuringweightandheightisthemostcommonwayofassessing
malnutritioninagivenpopulation.Suchuseofmeasurementsof
dimensionsofthehumanbodyisknownasanthropometry.
Anthropometryisawidelyused,inexpensiveandnoninvasive
measureofthegeneralnutritionalstatusofanindividualora
populationgroup.Thethreecommonlyusedanthropometricindices
are:
-Weight-for-Age{WFA).
-Length-for-AgeorHeight-For-Age(HFA).
-Weight-for-LengthorWeight-for-Height{WFH).
PEMreducesgrowthinchildren
Energyexpenditureinexcessofconsumptionleadstometabolizing
nutritionreservesintheformofstoredbodyfat.
Leanbodymassintheformofmuscleandevenorgantissuewill
alsobeconsumedifPEMpersists.itservesasareliableindicator
forPEM
Weightlossaccompaniestheinitialstagesofinadequateenergy
intakebut,ifprolonged,isfollowedbywasting,calledinitssevere
clinicalform,marasmus.
Inchildren,PEMdelaysorpermanentlystuntsgrowthandincreases

morbidityandmortality.
Ref.ParkPSMed.24thPageno.677

200.A2yearsbabywith6.7kg,Hb%-6mg/dl
totalprotein3mg/dL,lowalbuminwith
distendedstomachbutnoproteinuria.
Whatwillbethediagnosis?

a)Marasmus
b)Kwashiorkor
c)Indianchildhoodcirrhosis
d)None
CorrectAnswer-B
Ans.B.Kwashiorkor
Kwashiorkorisaformofsevereacutemalnutrition(SAM)
characterizedbyedemaandapathy.
Edemaismostlikelytoappearfirstonthefeetandtheninthelower
legs.Itcandevelopintogeneralizededemaaffectingthehands,
arms,andface.
Moonface
Skinchangesincludedepigmentationofskin,dermatoses,dark,
crackedpeelingpatches(flakypaintdermatosis)withpaleskin
underneaththatiseasilyinfected.
Hairissparse,easilypulledout,andmayturnreddish.
Flagsign:alternatingbandsofhypopigmentedandnormal
pigmentedareasonthehairstrand
Theliverisoftenenlargedwithfat(fattyLiver).
Thechildrenaremiserableandapatheticandoftenrefusetoeat.
Musclewastingandgrowthfailureareseen.
Villousatrophyofsmallintestineanddiarrhea.


NelsonTextbookofPediatrics20thEditionPage:301

201.InRDSinachild,whichcellsarefound
defective?
a)Type1pneumocytes
b)Type2pneumocytes
c)Bronchialepithelium
d)None
CorrectAnswer-B
Ans.B.Type2pneumocytes
TypeIcellsformthealveolarwallwhiletheTypeIIcellssynthesize
andsecretesurfactantDPCC.
ThedefectinthebiosynthesisofDipalmitoylphosphatidylcholine
(DPPC),alsoknownasdipalmitoyllecithinleadstoRespiratory
distresssyndrome.
Ref.https://www.ncbi.nlm.nih.gov/pmc/articles/PMC2880575/

202.Causeofgreenish-blackstoolina
neonate-
a)Meconium
b)Biliverdin
c)Bilirubin
d)Urochrome
CorrectAnswer-B
Ans.B.Biliverdin
Fetalbowelcontentsconsistofvariousproductsofsecretion,such
asglycerophospholipidsfromthelung,desquamatedfetalcells,
lanugo,scalphair,andvernix.Italsocontainsundigesteddebris
fromtheswallowedamnionicfluid.Thedarkgreenish-blackis
causedbypigments,especiallybiliverdin.
Ref.WilliamsObstetricsEd.24thPageno.141

203.Atermbabyonbreastfeedingwith
bilirubin14mg/dLwhichofthefollowing
istrue?

a)Exchangetransfusion
b)Continuetobreastfeed
c)Phototherapy
d)None
CorrectAnswer-B
Ans.B.Continuetobreastfeed
Comparedwithformula-fednewborns,breastfedinfantsarethreeto
sixtimesmorelikelytoexperiencemoderatejaundice(totalserum
bilirubinlevelabove12mgperdL)orseverejaundice(totalserum
bilirubinlevelabove15mgperdL.
Inabreastfednewbornwithearly-onsethyperbilirubinemia,the
frequencyoffeedingsneedstobeincreasedtomorethan10per
day.Iftheinfanthasadeclineinweightgain,delayedstooling,and
continuedpoorcaloricintake,formulasupplementationmaybe
necessary,butbreastfeedingshouldbecontinuedtomaintainbreast
milkproduction.
Phototherapyusuallynotrecommendedwhenthetotalserum
bilirubinlevelisbelow15mgperdL.
Ref:https://www.aafp.org/afp/2002/0215/p599.html

204.Truehermaphroditismkaryotype:
a)45X0streakedgonads
b)46XXOvotestis
c)47XY+9
d)47XX
CorrectAnswer-B
Ans.B.46XXOvotestis
Truehermaphroditeorovotesticulardisorderofsexualdifferentiation
(OVO-DSD)isoneoftherarestvarietyofallintersexanomalies.
Inabout90%ofcases,patientshave46XXkaryotype.
Rarely,46XY/46XXmosaicismmayoccur.

205.APGARscore3at1minuteindicates:
a)Mildlydepressed
b)Furtherresuscitationnotneeded
c)Severelydepressed
d)Normal
CorrectAnswer-C
Ans.C.Severelydepressed
APGARCRITERIA:

Component
Scoreof
Scoreof1
Scoreof2
of
0
backronym
blueat
nocyanosis
blueor
extremities
bodyand
Skincolor
paleall
Appearance
bodypink
extremities
over
(acrocyanosis) pink
<100beatsper >100beats
Pulserate
absent
Pulse
minute
perminute
no
grimaceon
Reflex
response suctionor
cryon
irritability
Grimace
to
aggressive
stimulation
grimace
stimulation stimulation
flexedarms
andlegsthat
Activity
none
someflexion
Activity
resist
extension
Respiratory
weak,irregular, strong,robust
absent
Respiration
effort
gasping
cry

INTERPRETATION:
Thetestisgenerallydoneat1and5minutesafterbirthandmaybe
repeatedlaterifthescoreisandremainslow.
Scores7andabovearegenerallynormal
Score4to6,fairlylow
Score:3andbelowaregenerallyregardedascriticallylowand
causeforimmediateresuscitativeefforts.

206.Awomandevelopedpainandcrawling
sensationonherlegsatnight.Clinical
historyofrestlesslegsyndrome.Drugof
choice?.

a)Pramipexole
b)Gabapentin
c)VitaminB12
d)Irontablets
CorrectAnswer-A
Ans.apramipexole
Restlesslegsyndrome:
Subjectivesensation"creepy-crawly"feelinginthelimbs,and
irresistibleurgetomovethelegswhenatrestorwhiletryingtofall
asleep.
Whentheindividualislyinginbedandrelaxing,heorsheis
disturbedbythesesensations.
Thenheorshemovesthelegsandagaintriestofallasleep.
Thiscyclesometimescontinuesforhoursandresultsinprofound
insomnia.
Treatment:
Thedopaminergicagonists'pramipexoleandropiniroleareFDA
approvedandrepresentthetreatmentsofchoice.
Ref.KaplanandSadock'sSynopsisofPsychiatry-11thEdition
(Pageno559)


207.HistoryofArthritisinvolving1stMCP
joint,otherPIP&DIPjoints,spareswrist
andankle.Whatcouldbethediagnosis

a)Osteoarthritis
b)Rheumatoidarthritis
c)Psoriaticarthritis
d)Gout
CorrectAnswer-A
Ans:a.Osteoarthritis
>Thedistalinterphalangeal(DIP)jointisactuallythemostcommon
locationonthebodyforosteoarthritis(OA).
>Thefrequencyofhandarthritis,OAattheDIPjointoccursin
approximately58%ofindividuals.
>ThesymptomsofOAattheDIPjointcommonlyincludepainand
changestothesizeandshapeofthejoint.
https://www.3pointproducts.com/blog/health-arthritis-finger-
and-toe-conditions/dip-joint-osteoarthritis-how-to-treat-this-
common-form-of-arthritis


208.12yroldChildadmittedtoICUwithblunt
traumaandfemurfracture-Pao260%
despite100%o2andrebreathermask,
CXRshowslungfieldsclearbutthe
patientremainsconfused.Whatismost
likelythediagnosis-

a)Pulmonarycontusion.
b)Fatembolismsyndrome.
c)Hypovolaemicshock.
d)Pulmonaryembolism.
CorrectAnswer-B
Ans:b.Fatembolism.
Inthispatient,basedonthecasepresentationthereisRespiratory
Systeminvolvedaspao2is60,andCNSinvolvementasheisina
confusedstate.
>So,thediagnosishereisFatembolismsyndrome.
>Embolizedfatwithincapillarybedscausedirecttissuedamageas
wellasinduceasystemicinflammatoryresponseresultingin
pulmonary,cutaneous,neurological,andretinalsymptoms.
>Gurd'sclinicaldescriptionoftheFESrenewedinterestinstudying
thissyndrome
https://www.ncbi.nlm.nih.gov/pmc/articles/PMC3665122/

209.A4yroldchildwhileplayingsuddenly
spunaroundhiselbowfromherservant
maid'shandandnowcontinuously
cryingnotallowinganyonetotouchhis
elbow.Heiskeepinghiselbowextended
.whatismostlikelythediagnosis-

a)Radialheadfracture
b)Pulledelbow
c)supracondylarfracture
d)Elbowdislocation
CorrectAnswer-B
Ans:b.Pulledelbow
>Pulledelbow,alsocallednursemaid'selbow,isaradialhead
subluxationcausedbyaxialtractionorasuddenpulloftheextended
pronatedarm,anditisaverycommonphenomenon.
>Inthiscaseofpulledelbow,thechildusuallyavoidsmovingthe
affectedarm,holdingitclosetohisorherbody,withoutconsiderable
pain,andnoobviousswellingordeformitycanbeseen.
>Whileafractureshouldbeexcluded,thepulledelbowcanusually
beidentifiedbasedonthispresentation.
https://www.ncbi.nlm.nih.gov/pmc/articles/PMC5999240/

210.Asexuallyactivefemalewiththeprofuse
frothyfoul-smellingdischargewith
intenseitching.Strawberrycervix
revealedonexamination.Whatwillbe
thediagnosis?

a)Trichomonasvaginalis
b)Bacterialvaginosis
c)Candidiasis
d)None
CorrectAnswer-A
Answer:a.Trichomonasvaginalis
Trichomonasvaginitis:
ClinicalFeatures:
Thereissuddenprofuseandoffensivevaginaldischargeoften
datingfromthelastmenstruation.
Irritationanditchingofvaryingdegreeswithinandaroundthe
introitusarecommon.
Thereisthepresenceofurinarysymptomssuchasdysuriaand
frequencyofmicturition.
Theremaybehistoryofprevioussimilarattacks
OnExamination:
Thereisthin,greenish-yellowandfrothyoffensivedischargeper
vaginum.
Thevulvaisinflamedwithevidencesofpruritus.
Vaginalexaminationmaybepainful.Thevaginalwallsbecomered
andinflamedwithmultiplepunctatehemorrhagicspots.Similarspots

arealsofoundoverthemucosaoftheportiovaginalispartofthe
cervixonspeculumexaminationgivingtheappearanceof
`strawberry'
Ref:DuttaGynaecology6theditionPageno.163-164

211.33yroldfemalewithheavymenstrual
bleedingfor6monthscomestothe
gynaecologyOPD.Onexamination,no
abnormalitywasseen.USGalso
appearednormal.Thepatientwastried
tobemanagedonnon-hormonal
treatmentbutitfailed.Whatwillbethe
nextmanagementstep?

a)Hormonaltherapy
b)Endometrialsampling
c)Hysterectomy
d)Hysterectomy
CorrectAnswer-B
Ans.B.Endometrialsampling
BeforestartingHormonaltherapyendometrialsamplingisdoneasif
hormonaltherapyprecedessamplingthesamplingresultswillbe
altered.
AsalreadymentionedinquestiontheUSGandclinicalexamination
shownoabnormalityhencethediagnosiscan'tbemadeonthatso
fordiagnosisweneedtodoendometrialsamplingandseethe
hormonalpattern.Thenwecanproceedwithhormonaltherapy.
Ref.DuttaGynaecologyed.6thpageno.120

212.Postmenopausalwomen1stlineofdrug
forosteoporosisis-
a)OCP
b)Bisphosphonates
c)Raloxifene
d)Strontium
CorrectAnswer-B
Ans.is.B.Bisphosphonates
Firstlineofmanagementofosteoporosis?Bisphosphonates
females.
Alendronate,etidronate,pamidronate,andIbandronateare
bisphosphonateswhichinhibitboneresorption,andarevery
effectiveforbothosteoporosispreventionandtreatment.
Uses:
Firstlinedrugsfortreatingpostmenopausalosteoporosis
Paget'sdisease
Osteolyticbonemetastasis.
Raloxifene:Itisaselectiveestrogenreceptormodulatorwhichis
alsousefulinmanagementofosteoporosisbutitisahormonal
preparation.

213.Prolactinsecretedmaximumat-
a)24hrsafterdelivery
b)REM
c)2hrsrunning
d)24hourafterOvulation
CorrectAnswer-A
Ans.A.24hrsafterdelivery
Prolactinismaximum24hrsafterdelivery.
Prolactinsecretionalsoincreasesduringstrenuousexerciseand
sleep(NREM).
24hrsafterovulationestrogenhasanegativefeedbackeffecton
prolactinwhichdecreasesthelevelofprolactin.

214.Allareusedforpostcoitalcontraception
except-
a)CuT
b)Ru486
c)Highdoseestrogen
d)Danazol
CorrectAnswer-D
Ans.D.Danazol
EMERGENCYCONTRACEPTIVES:

Drug
Dose
PREGNANCY
RATE

Levonorgestrel
0.75mgstatand
0?1
after12hours
Ethinylestradiol50?g+
2tabstatand2after 0?2
norgestrel0.25mg
12hours
Ethinylestradiol
2.5mgBd?5days
0?0.6
Mifepristone
100mgsingledose
0?0.6
CopperIuds(Goldstandard)
Insertionwithin5
0?0.1
days
(RU486Mifepristone)
30mgPO
0?1
(ulipristalacetate)
Ref:DUTTAGYNAECOLOGYED.6THPAGENO.492

215.Apatientdeliveredathomewitha
completeperinealtearcametothe
hospitalafter2weeks.What
managementwillyouprefer?

a)Immediaterepair
b)Repair3weekspost-delivery
c)Repair6weekspost-delivery
d)Repair3monthspost-delivery
CorrectAnswer-D
Ans.D.Repair3monthspost-delivery
Therecenttearsshouldberepairedimmediatelyfollowingthe
deliveryoftheplacenta.Thisreducesthechanceofinfectionand
minimizesbloodloss.
Incasesofdelaybeyond24hours,therepairistobewithheld.
Antibioticsshouldbestartedtopreventinfection.
Thecompletetearshouldberepairedafter3monthsifdelayed
beyond24hours.
Incaseofanydoubttothegrade3rd-degreetear,itisadvisableto
classifytoahigherdegreeratherthanalowerdegree.
Ref.ObsDutta24theditionPageno.490

216.InModernobstetrics,forsensitizedRh-
negativemotherwhatshouldbedoneto
evaluatetheconditionofthemother?

a)MCAdopplerpeaksystolicvolume
b)Fetalblood
c)Amniocentesis
d)Biophysicalprofile
CorrectAnswer-A
Ans.A.MCAdopplerpeaksystolicvolume
Dopplerultrasound:SerialDopplerstudyofthemiddlecerebral
artery(MCA)-peaksystolicvelocity(PSV)isthemainstaytoassess
fetalanemia.Avalue>1.5multiplesofthemedian(MOMs)forthe
correspondinggestationalage,predictsmoderatetoseverefetal
anemia.
Thisvalue(between24weeksand35weeksofgestation),isan
indicationforcordocentesisandfetaltransfusion
MostcentershavereplacedserialamniocentesiswithserialMCA
Dopplerstudies.
Ref.OBstetricsDuttaed.8thpg.391

217.A7weekspregnantladyhas1accidental
exposuretox-ray.Whichofthefollowing
shouldbedone?

a)Continuepregnancy
b)Terminatepregnancy
c)Chromosomeanalysis
d)Preinvasivediagnostictesting
CorrectAnswer-A
Ans.A.Continuepregnancy
ExposuretoX-rayhasaverylowrangeofradsoingivencase
scenarioPregnancywillbecontinued.
Exposure>15radduringthesecondandthirdtrimesteror>5radin
thefirsttrimesterneedspatientcounseling.Electiveterminationof
pregnancymaybeconsidered.
Ref.OBSDutta8theditionPageno.740
William'sObs24theditionPageno.931Tableno46-5


218.A32-year-oldwomancomplainsof
amenorrheasincethedeliveryofababy
15monthspreviously,despitethefact
thatshedidnotbreastfeedherbaby.The
deliverywascomplicatedbyexcessive
hemorrhagethatrequiredatransfusion
of2.5litersofblood.Shehasalsobeen
fatiguedandhasgainedanadditional
4.5Kgsincethebabywasborn.
Laboratorydatashowthefollowing:
SerumLH<1IU/L(normal,4-24IU/L)
Serumestradiol5pg/mL(normal,20-
100pg/mL)
SerumTSH0.1mU/L(normal,0.5-5
mU/L)
SerumGH3ng/mL(normal,<5ng/mL)
SerumACTH28pg/mL(normal,10-50
pg/mL)
Serumprolactin2ng/mL(normal,
Injectionof500gofTRHfailedto
producetheexpectedriseinbothserum
TSHandprolactin.Whichofthe
followingdiagnosesmostlikelyexplains
thefindingsinthispatient?

a)Hashimoto'sthyroiditis
b)Isolatedgonadotropindeficiency

b)Isolatedgonadotropindeficiency
c)Primaryamenorrhea
d)Sheehan'ssyndrome
CorrectAnswer-D
Ans.D.Sheehan'ssyndrome
Sheehan'ssyndrome
Itishypopituitarismduetoischemicdamagetothepituitaryresulting
fromexcessivehemorrhageduringparturition.
Thepituitaryisenlargedduringpregnancy;itismoremetabolically
active,andmoresusceptibletohypoxemia.
Furthermore,thebloodvesselsinthepituitarymaybemore
susceptibletovasospasmbecauseofthehighestrogen.
Inabout30%ofwomenwhohemorrhageexcessivelyduring
parturition,somedegreeofhypopituitarismeventuallybecomes
manifest.
Thesymptomsdependonhowmuchofthepituitaryisdamagedand
whatcelltypesaredestroyed.Thepatientdescribedaboveexhibited
persistentamenorrheaafterthedeliveryofherinfant.
Thisisduetothedestructionofpituitarygonadotrophsand
diminishedsecretionofgonadotropins(LH).
Therealsoappearstohavebeensignificantdestructionof
lactotrophssinceTRHinjectionfailedtoinduceanincreasein
prolactin.
Hadthewomenattemptedtobreastfeedherinfant,afailureto
lactatemostlylikelywouldhaveoccurred.
Thiscaseisalsocharacterizedbysecondaryhypothyroidism.
ThelowTSHandfailuretorespondtoTRHinjectionare
confirmatory.
CorticotrophsappeartohavebeensparedsinceplasmaACTHis
normal.
Itisnotclearwhethersomatotrophsweredamaged.
FurthertestingwouldbeneededtoseeifGHreserveisdiminished.

Ref.WilliamObs.ed.24thpageno.798,1163
DuttaGynaecology6thed.Pageno.465


219.Meiosisoccursin
a)Adultovary
b)Prepubertaltestis
c)Atbirthinovary
d)All
CorrectAnswer-A
Ans.is.A.Adultovary
Meiosis1iscompletedatpubertyhenceanswerisadultovary
Totalnumberofoocytesat20weeksofintrauterinelifeisabout6?7
million.Atbirth,thetotalnumberofprimordialfolliclesisestimatedto
beabout2million.Theprimaryoocytesdonotfinishthefirstmeiotic
divisionuntilpubertyisreached.
Spermatogenesisoccursatpubertysoitcannothappenin
prepubertaltestis.

220.DCDAtwins,38wks,firsttwinbreech
motherhasBP140/96,1+proteinuria
what'sthemanagement

a)ImmediateLSCS
b)Inductionat40weeks
c)Immediateinductionanddelivery
d)Inductionifsignsofpreeclampsia
CorrectAnswer-A
ANS.A.ImmediateLSCS
CASEDISCUSSION:
ThepatienthasmildpreeclampsiawithBP140/96,soTOPis37
weeks.
Thispatientis38weekssoimmediateterminationofpregnancyis
required.
ThefirsttwinisbreechsothemodeofdeliveryisLSCShencethe
answerisA.
INDICATIONSOFCESAREANSECTION:
Theindicationsarebroadlydividedinto:
Obstetric

Placentaprevia
Preeclampsia
Previouscesareansection
Cordprolapseofthefirstbaby
Abnormaluterinecontractions
Contractedpelvis.
Fortwins:
Boththefetusesoreventhefirstfetuswithnoncephalic(breechor

transverse)presentation
Twinswithcomplications:IUGR,conjoinedtwins
Monoamniotictwins
MonochorionictwinswithTTTS
Thecollisionofboththeheadsatbrimpreventingengagementof
eitherhead.
Ref.OBSDutta8thedPageno.244

221.A36-weekoldpregnantladywith
previoustwindelivery.Whatisthe
Ovarianscore?

a)G2P1
b)G2P2
c)G3P2
d)G3P3
CorrectAnswer-A
ANS.A.G2P1
Gravidaincludesallconfirmedpregnancies.Eachpregnancyisonly
countedonetime,evenifthepregnancywasmultiplegestations,
suchastwinsortriplets
Parityreflectsthetotalnumberofbirthsafter20weeks,notthetotal
numberofinfantsborn.
Ref.OBSDutta8thedPageno.107

222.Misoprostolusedintheinductionof
labourisananalogueofwhichofthe
followingtypeofprostaglandin?

a)PGE1
b)PGE2
c)PGI2
d)PGF2alpha
CorrectAnswer-A
CorrectAns:A.PGE1
MisoprostolisamethylesterofPGE1.
IndicationsofMisoprostol:

Itisusedforcervicalripening.
Transvaginallyitisusedforinductionoflabour.
UsesofProstaglandinsinObstetrics:
Inductionofabortion
Terminationofmolarpregnancy
Inductionoflabour
Cervicalripeningpriortoinductionoflabour
Accelerationoflabour
Managementofatonicpostpartumhemorrhage
Medicalmanagementoftubalectopicpregnancy

223.Inearlypregnancyclinicalsignsof
feelingthecervixandthebodyofbulky
uterusseparatedbecauseofsoftened
isthmusat6-8weeksofgestation:

a)Goodell'ssign
b)Chadwick'ssign
c)Piskacek'ssign
d)Hegar'ssign
CorrectAnswer-D
CorrectAns:D.Hegar'ssign
At6to8weeksmenstrualage,thefirmcervixcontrastswiththenow
softerfundusandthecompressibleinterposedsoftenedisthmus
--Hegarsign

224.13yroldchildvisitgynaecologyOPD
withacomplaintofnotattaining
menarchewithkaryotype46XX.On
examination,clitoromegalyisseen.
Whichenzymeismostlikelytobe
deficientintheabovecondition?

a)21alpha-hydroxylase
b)11beta-hydroxylase
c)17alpha-hydroxylase
d)3beta-hydroxysteroiddehydrogenase
CorrectAnswer-A
Ans.A.21alpha-hydroxylase
TheconditiondescribedaboverepresentstheCongenitaladrenal
hyperplasia.
"Morethan90%ofCAHcasesarecausedby21-hydroxylase
deficiency"
Congenitaladrenalhyperplasia:
Itisduetoaninbornerrorofadrenalsteroidmetabolism,commonly
dueto21-hydroxylase(95%)andrarelydueto11-hydroxylaseor3
hydroxysteroiddehydrogenasedeficiency.
Clinicalpresentation
Anambiguityofsexatbirth
Hirsutismandamenorrheamaybethepresentingfeaturesaround
pubertyinamilderform.
Thekaryotypeis46,XX.
Ref.DuttaGynaecologyed.6thpageno.440


225.Amotherbroughther16-year-old
daughtertoGynaecologyOPDwitha
complaintofnotattendingmenarche.
ShegivesH/Ocyclicabdominalpain.On
furtherexaminationmidline,abdominal
swellingseen.Perrectalexamination
revealsabulgingmassinthevagina.
Whichofthefollowingcanbemost
commonlyseen?

a)Imperforatehymen
b)Transvaginalseptum
c)Vaginalagenesis
d)MRKH
CorrectAnswer-A
Ans.A.Imperforatehymen
Accordingtotheclinicalcase,patientssufferfromprimary
amenorrheaandcryptomenorrhea(cyclicabdominalpain).
Thegirlisagedabout14?16years.
Thechiefcomplaintsareperiodiclowerabdominalpain,whichmay
becontinuous,primaryamenorrheaandurinarysymptoms,suchas
frequency,dysuriaorevenretentionofurine.
Infact,insignificantcases,thepresentingfeaturemaybethe
retentionofurine.Thecauseofretentionisduetotheelongationof
theurethra.
Anabdominalexaminationrevealsasuprapubicswelling,whichmay

beuterineorfullbladder.Priorcatheterizationrevealsthetruestate.
Vulvalinspectionrevealsatensebulgingmembraneofbluish
coloration
Ref.DuttaGynaecologyed.6thpageno.42

226.Whichofthefollowingisnotan
estrogen-dependentpubertalchange?
a)Hairgrowth
b)Menstruation
c)VaginalCornification
d)Cervicalmucus
CorrectAnswer-B
Ans.B.Menstruation
Pubarcheordevelopmentofaxillaryandpubichairisdueto
testosterone(inboththesexes).
Pubertyinfemales:
Involvesthebeginningofmenstrualcycles(menarche),breast
development(thelarche),andanincreaseinadrenalandrogen
secretion(adrenarche).
Estradiolinducesthedevelopmentofsecondarysexcharacteristics,
includingthebreastsandreproductivetract,andincreasedfatinthe
hips.
Estrogensalsoregulatethegrowthspurtatpuberty,vaginal
cornification,andcervicalmucusproduction,induceclosureofthe
epiphyses,haveapositiveeffectinmaintainingboneformation,and
canantagonizethedegradingactionsofparathyroidhormoneon
bone.
Reference:RhoadesandTanner'sMedicalPhysiology,2ndedition
(Pageno:680)

227.A22-year-oldprimigravidavisitsANC
OPDwith20weeksPOG.Onexamination
uterineheightrevealsa16-week
size.USGshowsreducedliquor.What
willbethediagnosis?

a)Renalagenesis
b)Fetalanemia
c)Barter'ssyndrome
d)Liddlesyndrome
CorrectAnswer-A
Ans.A.Renalagenesis
Thequestionstatesreducedliquorthatmeansoligohydramniosis
seen.
Oligohydramniosisdefinedasanamnioticfluidindexof5cmor
less.
Itisalmostalwayspresentwhenthereiseitherobstructionofthe
fetalurinarytractorrenalagenesis.
WilliamsObstetricsed24thpage237

228.MostcommonsiteforFertilizationis-
a)Ampulla
b)Isthmus
c)Intramural
d)Fimbriae
CorrectAnswer-A
Ans.A.Ampulla
Fertilizationistheprocessoffusionofthespermatozoonwiththe
matureovum.
Itbeginswithspermeggcollisionandendswiththeproductionofa
mononucleatedsinglecellcalledthezygote.
Itsobjectivesare:
Toinitiatetheembryonicdevelopmentoftheeggand
Torestorethechromosomenumberofthespecies.
Almostalways,fertilizationoccursintheampullarypartoftheuterine
tube.
Reference:DuttaObstetricsed.8thPageno23

229.Distensionmediausedforhysteroscopy
withbipolarcautery?
a)Glycine
b)NS
c)Co2
d)Dextran70
CorrectAnswer-B
Ans.B.NS
Thedistendingmediacommonlyusedinhysteroscopyisnormal
saline
--Theuterinecavityisdistendedwithamediatoseparatethe
uterinewallsandtohaveapanoramicview.Themediausedcould
beeitheragasoraliquid.
Carbondioxide(CO2)--iscommonlyusedfordiagnosticpurposes.
Itissolubleinbloodandissafe.
Hysteroflatorprovidesagasflowrateofamaximumof100mLper
minuteandamaximumpressureof100mmHg.
Liquidmediaisusedforoperativeprocedures.
Normalsalinecanbeusedissuitableforbipolarcauterybutnot
suitableformonopolarelectrosurgery.Constantflowistobe
maintainedtoflushtheoperativearea.
Ref.DuttaGynaecologyed.6thpageno.620,624

230.Besttreatmentoptionforseptateuterus-
a)TompkinsMetroplasty
b)Jonesmetroplasty
c)Strassmannmetroplasty
d)Transcervicalhysteroscopicresectionoftheseptum
CorrectAnswer-D
Ans.D.Transcervicalhysteroscopicresectionoftheseptum
Hysteroscopicmetroplastyismorecommonlydone.
Resectionoftheseptumcanbedoneeitherbyaresectoscope
orbylaser.
Advantagesare:

1. Highsuccessrate(80?89%),
2. Shorthospitalstay
3. Reducedpostoperativemorbidity(infectionoradhesions)
4. Subsequentchanceofvaginaldeliveryishighcomparedto
abdominalmetroplastywherethecesareansectionismandatory.
Othermethods:
Abdominalmetroplastycouldbedoneeitherbyexcisingtheseptum
(Strassman,Jones,andJones)orbyincisingtheseptum
(Tompkins).
Ref.DuttaGynaecologyed.6thpageno.47

231.ApregnantfemalehadMeconium
stainedliquorandunderwentemergency
LSCS.Afewdayslaterhercondition
deteriorated.USGshowededematous
bowels.What'sthecause?

a)Meconiumperitonitis
b)Paralyticileus
c)Adhesiveintestinalobstruction
d)Intra-abdominalabscess
CorrectAnswer-B
Ans.B.Paralyticileus
POSTOPERATIVECOMPLICATIONSOFLSCS:
Intestinalobstruction:Theobstructionmaybemechanicaldueto
adhesionsorbands,orparalyticileusfollowingperitonitis.
Paralyticileusisanadynamicobstructioninwhichthereisafailure
oftransmissionofperistalticwaves
Clinicalfeatures:
Theresultantstasisleadstothefollowing:
Accumulationoffluidandgasinthebowel
Withassociateddistension:markedandtympanitic
Vomiting(effortless)
Absenceofbowelsounds
Absoluteconstipation
Duttaobstetricsed.8thPageno.678
Bailey&Love'sShortPracticeofSurgery-27thEdition(Page
no1297)


232.18-year-oldgirlpresentswith6months
ofamenorrheawithh/olow-gradefever,
weightloss,painabdomen,generalized
weaknesses.OnPRexamination,
palpableleft-sidedpelvicmassfelt...
Diagnosisis

a)Fibroidwithdegeneration
b)TBpelviswithTuboovarianmass
c)Ectopicpregnancy
d)Granulosacelltumour
CorrectAnswer-B
Ans.B.TBpelviswithTuboovarianmass
Thetuberclesbursttopourthecaseousmaterialinsidethelumen
producingtubercularpyosalpinx,whichmayadheretotheovaries
andthesurroundingstructures.
Oftentheinfectionspreadsoutwardsproducingperisalpingitiswith
exudation,causingdenseadhesionswiththesurroundingstructures
tuberculartubo-ovarianmass.
Clinicaldiagnosticfeatures:
Weakness,low-gradefever,anorexia,anemiaornightsweatsmay
bepresent.
Infertility:Itmaybeprimaryorsecondary
Chronicpelvicpain
Vaginaldischarge--postcoitalbleedingorablood-stained
discharge.
Constitutionalsymptomssuchaslossofweight,malaise,anorexia,

pyrexia,andanemiaarepresentintheacutephaseofthedisease.
Menstrualabnormality:Inabout50percent,themenstrualfunctionis
normal.
PresenceofpelvicmasswithnodulesinthepouchofDouglas
palpable
Ref.GynaecologyDuttaed.6thPageno.139-142

233.AChildisborn,coveredwithathick
membranouscoat,whatcouldbethe
possiblediagnosis?

a)Lamellarichthyosis
b)X-linkedichthyosis
c)IchthyosisVulgaris
d)Ichthyosisacquista
CorrectAnswer-A
Answer-a.Lamellarichthyosis
Explanation-Ichthyosisisoneofthemostcommon
genodermatoses.Itischaracterizedbydry(xerotic)scalyskinall
overthebody.Scalesaredull-brown-black.
Thebasicdefectisanimpairmentinthebarrierfunctionofskinand
inabilitytomaintainmoisture.Collodionbabyisthetermusedto
denotethenewbornbabywithichthyosis;thenewbornbabyis
encasedinathick,shinymembranecoatcalledcollodionandis
seeninLamellarichthyosis
Ref-ArvindAroraskin,6thedition,pageno203

234.Mouthtomouthrespirationprovides
whatpercentageofoxygen?
a)10%
b)16%
c)21%
d)100%
CorrectAnswer-B
Ans.B.16%
Mouthtomouthbreathingprovides0.8to1.2litersofexhaledairper
breathand16%ofoxygenwhichisenoughtosustainlife.
TheuseofAmbubagandroomairprovides21%O2.
TheAmericanHeartAssociationrecommendstidalvolumesof700
to1000mLduringmouth-to-mouthventilation,butsmallertidal
volumesof500mLmaybeofadvantagetodecreasethelikelihood
ofstomachinflation,asmouth-to-mouthventilationgascontainsonly
17%oxygen,but4%carbondioxide.

235.Cancerpatientundergoesradiotherapy,
pickthetruestatementfor
radiosensitivityoftissues?

a)Rapidlydividingcellsareresistanttoradiation
b)GImucosaisoneofthemostradioresistanttissuesinthebody
c)Theintensityofradiationisinverselyproportionaltothesquare
ofdistancefromthesource
d)Smallbloodvesselsareleastresistanttoradiation
CorrectAnswer-C
Answer-C-Theintensityofradiationisinverselyproportional
tothesquareofdistancefromthesource
Theinversesquarelawdescribestheprincipleofdosereduction
asthedistancefromthesourceincreases.
Thedoseisproportionaltotheinverseofthesquareoftheradius.
Thusifyoudoublethedistanceyoureducethedosebyafactorof
four.
Ref-1.ShafieiSA,HasanzadehH,ShafieiSA.Asimple
calculationmethodfordeterminationofequivalentsquarefield.
JMedPhys.2012;37(2):107-11.DOI:10.4103/0971-
6203.94746-FreetextatPubMed-Pubmedcitation


236.MostcommonDRUGcausing
dependence?
a)Cannabis
b)Cocaine
c)Heroin
d)Amphetamine
CorrectAnswer-A
ANS-A.Cannabis
Cannabis-9millionusers,192millionusers(Acc.ToWHODrug
report2018)
Heroin(opioidderivative)-2.5lakhusers
Itisestimatedthat275millionpeopleusedillicitdrugs,suchas
cannabis,amphetamines,opioids,andcocaine,in2016which
translatesintoanannualprevalenceofillicitdruguseof5.6%.
Cannabisismostusedwith192millionusers.Some31million
peoplewhousedrugssufferfromdrugusedisorders.-(WHODrug
report2018)
Ref:ARVINDARORAreviewofpsychiatryp:64,WHODrug
report(2018)


237.MBBSstudentshewaschokingwith
dyspnea,chesttightness,anxietyandan
impendingsenseofdoomon
examinationallsystemicconditionswere
foundnormal,thenshewentto
psychiatry.Whatistheprobable
diagnosisofthecondition?

a)Panicdisorder
b)Depression
c)Epilepsy
d)Asthma
CorrectAnswer-A
ANS-A.Panicdisorder
Panicdisorder-severeanxietywithchoking,chestpainand
palpitations.
Depression-lowmoodandlostinterestinactivities
Epilepsy-neurologicaldisorderwithwhole-bodyfatigueandmuscle
spasms
Anasthma-respiratorydisorderassociatedwithsymptomsofcough,
wheezing,etc
Ref:ARVINDARORAreviewofpsychiatryp:131

238.Apatientwithdepressionwasgiven
Imipraminefor2weeks.Relatives
noticedincreasedexcitement,colourful
clothes,increasedtalking.Whatisthe
nextstepinmanagement?

a)AntipsychoticwithImipraminecontinued
b)DiscontinueImipramineandstartValproate
c)ContinueImipraminealone
d)ManagewithValproatealone
CorrectAnswer-A
ANS-A.AntipsychoticwithImipraminecontinued
Theconditionfromwhichthepatientaboveissufferingisabipolar
disorderwhichincludesbothdepressionandmania.
Soinordertotreatbothantidepressanti.eimipramineiscontinued
alongwithanantipsychoticsuchasphenothiazines,thioxanthenes,
etc.
Antipsychoticorvalproatealonecannotreducethedepressionand
Imipraminealonecannotreducethemania.
Ref:ARVINDARORAreviewofpsychiatryp.no:111

239.APatientfallsdownoftenwith
behavioralchangeandenuresis.Whatis
theconditionassociatedwithhim?

a)Frontotemporaldementia
b)Normalpressurehydrocephalus
c)Parkinson'sdisease
d)Alzheimer'sdisease
CorrectAnswer-B
Ans-B.Normalpressurehydrocephalus
Frontotemporaldementia-Associatedwithpoorbehavioralcontrol,
decisionmaking,andlanguage.
Normal-pressurehydrocephalus-Associatedwithgaitdisturbance,
behaviouralchange,enuresis,anddementia.
Parkinson'sdisease-Associatedwithtremorandmuscularstiffness
Alzheimer'sdisease-Associatedmainlywithmemorylossand
confusion
Ref:NormalPressureHydrocephalusByMichaelJ.Fritsch,
UweKehler,UllrichMeierP.16;Oxfordtextbookofneurological
surgerybyAdelHelmyP.94


240.APatientwithdepressivesymptomsfor
6monthsandassociatedwith
auditoryhallucinationsfor2weeks.What
istheprobablediagnosisofthe
condition?

a)Psychoticdepression
b)Schizoaffectivedisorder
c)Maniadepressiveillness
d)Schizophrenia
CorrectAnswer-A
Ans-A.Psychoticdepression
Psychoticdepression-Depressionalongwithpsychoticsymptoms
suchashallucinations,delusions,etc.
Schizoaffectivedisorder-Symptomsofschizophreniaandmood
disturbancesoccurtogether.
Maniadepressiveillness-Maniadepressionalongwithpsychotic
symptoms.
Schizophreniaincludesallthesymptomsofschizophrenia
Ref:ArvindArorareviewofPsychiatrypage.103


241.Identifythetypeofjointinthegiven
picture
a)Syndesmosis
b)Synarthrosis
c)Synovialjoint
d)Symphysis
CorrectAnswer-C
Ans:Csynovialjoint
Themarkedstructureisajointbetweenthetransverseprocess
ofvertebraandribs-acostotransversejoint.
Interchondral,Sternocostal,Costovertebral,andCostotransverse
areallplanetypesofsynovialjoints.
BDC,7thedition,volume1,pg234.


242.Identifythemuscle.Whatisitsnerve
supply
a)Dorsalscapularnerve
b)Thoracodorsalnerve
c)Spinalaccessorynerve
d)Suprascapularnerve
CorrectAnswer-A
Ans:A.Dorsalscapularnerve
Markedmuscleislevatorscapulae.ItissuppliedbyC3,C4,andC5
(dorsalscapularnerve)


243.Identifythecartilage
a)Hyalinecartilage
b)Elasticcartilage
c)Articularcartilage
d)Fibrocartilage
CorrectAnswer-D
Ans:D.Fibrocartilage
Fibrocartilage
Fibrocartilageischaracterizedbyalargeamountofirregularand
densebundlesofcollagenfibersinthematrix.Itconsistsof
alternatinglayersofcartilagematrixandthickdenselayersoftypeI
collagenfibers.
Ref.InderbirSingh'sTextbookofHumanHistology-8thEdition
(Pagenos56-59)



244.Identifytheboundariesoftheanatomical
structureintheimage
a)Extensorpollicislongus
b)Abductorpollicislongus
c)Styloidprocessoftheradius
d)Alloftheabove
CorrectAnswer-D
Ans:D.Alloftheabove
Themarkedstructureistheanatomicalsnuffbox.
Boundariesare:
Themedialborder(ulnarside)ofthesnuffboxisthetendonofthe
extensorpollicislongus.
Thelateralborder(radialside)isapairofparallelandintimate
tendonsoftheextensorpollicisbrevisandtheabductorpollicis
longus.
Theproximalborderisformedbythestyloidprocessoftheradius
Thedistalborderisformedbytheapproximateapexofthe
schematicsnuffboxisoscelestriangle.

Thefloorofthesnuffboxvariesdependingonthepositionofthe
wrist,butboththetrapeziumandprimarilythescaphoidcanbe
palpated.


245.Identifythecelltypemarkedinthe
cerebellum?
a)Basketcells
b)Granulecells
c)Golgicells
d)Purkinjecells
CorrectAnswer-D
Ans:D.Purkinjecells
Explanation:



ThePurkinjecellspresentthemoststrikinghistologicalfeatureof
thecerebellum.
Elaboratedendritesextendintothemolecularlayerfromasingle
subjacentlayerofthesegiantnervecellbodies(calledthePurkinje
layer).
Onceinthemolecularlayer,thePurkinjecelldendritesbranch
extensivelyinaplaneatrightanglestothetrajectoryoftheparallel
fibers.
Inthisway,eachPurkinjecellisinapositiontoreceiveinputfroma
largenumberofparallelfibers,andeachparallelfibercancontacta
verylargenumberofPurkinjecells(ontheorderoftensof
thousands).
Ref:https://www.ncbi.nlm.nih.gov/books/NBK10865/


246.Identifythesleepwavemarkedinthe
EEGduringthesleep-wakecycle?
a)Stage1sleep
b)REMsleep
c)Stage2sleep
d)Stage3sleep
CorrectAnswer-B
Ans:B.REMsleep
Explanation:
REM/ParadoxicalSleep:
Duetothe"presenceofrapideyemovement"recordedon
electrooculogram.
Hasparadoxicalelevationofbrainactivity,metabolism&
physiologicalactivity.
"High-amplitudeslowwaves"replacedby"rapid,lowvoltageactivity
&betawave"-
Duringstage3&4REMsleep.
FeaturesofREMsleep:

1.InEEGrecording:
Beta-wave.
Reappearanceofalphawave.
Saw-toothwave(lowvoltagefastactivity).
Ponto-genital-occipitalspikes.
Ref:ArvindArorareviewbookofphysiology(p.204-205)


247.Casal'snecklaceisseenindeficiencyof:
a)VitaminAdeficiency
b)Niacin
c)Irondeficiencyanemia
d)VitaminB12deficiency
CorrectAnswer-B
Ans:B.Niacin
Thedeficiencyofniacinleadstotheclinicalconditioncalled
pellagra.PellagraisanItalianword,meaning"roughskin".Pellagra
iscausedbythedeficiencyofTryptophanaswellasNiacin.Pellagra
isseenmoreinwomen;thismaybebecausetryptophanmetabolism
isinhibitedbyestrogenmetabolites.
Thesymptomsofpellagraare:
Dermatitis:Intheearlystages,brightrederythemaoccurs,
especiallyinthefeet,ankles,andface.Increasedpigmentation
aroundtheneckisknownasCasal'snecklace.Thedermatitisis

precipitatedbyexposuretosunlight
Diarrhea:diarrheamaybemildorseverewithbloodandmucus.
Dementia:Itisfrequentlyseeninchroniccases.
Deliriumiscommoninacutepellagra.Irritability,inabilityto
concentrateandpoormemoryaremorecommoninmildcases.
Ataxiaandspasticityarealsoseen.
Ref-DMVasudevan-Textbookofbiochemistryformedicalstudents,
6thedn,WaterSolubleVitamins,pg395.


248.AnHivpositivepatientwithaCD4
COUNTOF300/Cummpresentswith
mucosallesionsinthemouthasshown
inthefigure.onmicroscopybudding
yeastsandpseudohyphaeareseen.A
mostprobablediagnosisis?

a)Candidiasis
b)Hairyleukoplakia
c)Lichenplanus
d)Diphtheria
CorrectAnswer-A
CorrectAns:A.Candidiasis
MostcommonopportunisticinfectioninHIViscandidiasis
CandidiasisisafungalinfectionduetoanytypeofCandida.
Whenitaffectsthemouth,itiscommonlycalledthrush.
Signsandsymptomsincludewhitepatchesonthetongueorother

Signsandsymptomsincludewhitepatchesonthetongueorother
areasofthemouthandthroat.Othersymptomsmayinclude
sorenessandproblemsswallowing.
Whenitaffectsthevagina,itiscommonlycalledayeastinfection.
Signsandsymptomsincludegenitalitching,burning,andsometimes
awhite"cottagecheese-like"dischargefromthevagina.
Robbin'sbasicsofpathology502,503


249.A25-Year-oldmalepresentedwitha2cm
thyroidnodule.Athyroidectomywas
done.Thehistologypictureisgiven
below.whatcouldbethediagnosis

a)Papillarycarcinomathyroid
b)Follicularadenoma
c)Gravesdisease
d)Adenomatousgoitre
CorrectAnswer-A
Ans:A.Papillarycarcinomathyroid
Papillarycarcinomasmaymanifestassolitaryormultifocallesions
withinthethyroid.Insomecases,theymaybewell-circumscribed
andevenencapsulated;inotherinstances,theyinfiltratethe
adjacentparenchymawithill-definedmargins.
Thelesionsmaycontainareasoffibrosisandcalcificationandoften
arecystic.Oncutsurface,theymayappeargranularandsometimes
containgrosslydiscerniblepapillaryfoci.
Thedefinitivediagnosisofpapillarycarcinomacanbemadeonly

aftermicroscopicexamination.
Thenucleiofpapillarycarcinomacellscontainveryfinelydispersed
chromatin,whichimpartsanopticallyclearappearance,givingrise
tothedesignationgroundglassor"OrphanAnnieeye"nuclei.in
cross-sections.
UnlikehyperplasticpapillarylesionsseeninGravesdisease,the
neoplasticpapillaehavedensefibrovascularcores.Concentrically
calcifiedstructurestermedpsammomabodiesoftenarepresent
withinthepapillae.
Focioflymphaticpermeationbytumorcellsarepresent
Robbin'sbasicsofpathology9theditionpageno.736


250.A5-year-oldchildpresentedwitha
historyofbloodinthestools.On
examination,therewasapolypoidmass
intherectum,abiopsyofwhichshowed
asbelow.Amostprobablediagnosisis?

a)Villousadenoma
b)Juvenilepolyp
c)Vascularmalformation
d)Serratedadenoma
CorrectAnswer-B
Ans:B.Juvenilepolyp
Thehistologicalpictureofjuvenilepolyps.Itconsistsof
pedunculated,smooth-surfaced,reddishlesionsthatarelessthan3
cmindiameteranddisplaycharacteristiccysticspacesoncut
sections.
Microscopicexaminationshowsthespacestobedilatedglands

filledwithmucinandinflammatorydebris
Robbin'sbasicsofpathology9theditionpageno.592


251.25-year-oldmanpresentsforaroutine
physicalexamination.Thepatientistall
andonexamination,hewasfoundto
haveanearlydiastolicmurmur.His
familypedigreeisgivenbelow.Whichof
thefollowingisthemodeofinheritance
bywhichthediseaseislikelytobe
transmitted?

a)AD
b)AR
c)XLR
d)XLD
CorrectAnswer-A
Ans:A.AD
AutosomalDominant(AD)InheritanceDiseases
?Mutatedgenescanexpressthemselvesinaheterozygousstate.
?Usually,itcausesadefectinthesynthesisofstructuralQornon-
enzymeproteins.

enzymeproteins.
?Thesehaveavariableonset.
?Thesearecharacterizedbyreducedpenetrance(individualsinherit
thegenebutcanbephenotypicallynormal)andvariable
expressibilityQ(thetraitisseenintheindividualscarryingthe
mutantgenebutisexpresseddifferentlyamongindividuals,e.g.
patientsofneurofibromatosishavevariantfrombrownishskinspots
tomultipleskintumoursindifferentpatients).


252.A51-year-oldpersoncamewitha
complaintofhematuria.Onexamination,
hewasnormotensiveandhadpedal
edema.Investigationsrevealedthe
patienthadnoglucosuriaandhada
creatininevalueof9mg%.Renalbiopsy
isasshownbelowwhichofthefollowing
investigationsoneshoulddotoidentify
theetiologyofthedisease?

a)ANA
b)ANTIGBMantibodies
c)HIVRNA
d)Urineimmunoelectrophoresis
CorrectAnswer-B
Ans:B.ANTIGBMantibodies
Kidneybiopsy:Immunofluorescencedemonstratedlinearstaining

withIgG,characteristicofanti-glomerularbasementmembrane
disease
,alsoknownasGoodpasturesyndrome.
Anti?glomerularbasementmembrane(anti-GBM)antibodiesare
seeninGoodpasturesyndrome.Theycauseendothelialcellinjury
resultinginanephriticsyndromewithhematuria.
Robbins&CotranPathologicBasisofDisease,9edPagenos
912-914



253.A35-year-oldheterosexualpatient
diagnosedwithHIVhadahistoryof
chronicwaterydiarrhea.Acolonoscopic
biopsyisshownbelow.Amostprobable
diagnosisis?

a)Giardia
b)CMV
c)Microspora
d)Cryptosporidium
CorrectAnswer-D
Ans:D.Cryptosporidium
DiseasecausedbyinfectionwithCryptosporidiumspecies,a
protozoalparasite
Traditionallyconsideredacoccidianparasite,butrecentevidence
suggestsitmaybeagregarineparasite
Infectionismostcommonintheterminalileumandproximalcolon
Thediseasealsooccursintheproximalsmallintestine,distalcolon,
gallbladder,bileducts,andpancreas

gallbladder,bileducts,andpancreas
Inimmunosuppressedpatients,chronicdiarrheaoccursthatisoften
debilitating,acalculouscholecystitis,sclerosingcholangitis,
pancreatitis,biliarystrictures,andrespiratorydiseasemayoccur
Intissuebiopsies,2-5mbasophilicroundbodiesareseen
protrudingfromtheapexofenterocytes("bluebeads")withinthecell
membrane;highlightwithGiemsastain
Villousatrophy,crypthyperplasia,cryptitisandincreasedmixed
inflammatorycellswithinthelaminapropriamaybeseen
ref:Bennett:Mandell,Douglas,andBennett'sPrinciplesand
PracticeofInfectiousDiseases,8thEdition
,CDC
-Cryptosporidium



254.An11-yearboypresentedwithcoughfor
15daysonexaminationhewasfoundto
havecervicallymphadenopathy.lymph
nodeexaminationshowedbelowfinding
whatcouldbethediagnosis

a)Leprosy
b)Sarcoidosis
c)TB
d)Syphilis
CorrectAnswer-C
Ans:C.TB
Histologicalpictureshownaboveisoftuberculosis
Tuberculosisisacommunicablechronicgranulomatousdisease
causedbyMycobacteriumtuberculosis.Itusuallyinvolvesthelungs
butmayaffectanyorganortissueinthebody.
Typically,thecentersoftuberculousgranulomasundergocaseous
necrosis.histologicxamination,sitesofactiveinvolvementare

markedbyacharacteristicgranulomatousinflammatoryreactionthat
formsbothcaseatingandnoncaseatinggranulomaswhichconsistof
epithelioidhistiocytesandmultinucleatedgiantcells.
Robbinsbasicsofpathology9theditionpageno.493


255.A40yearoldpersonpresentedwith10*8
swellinginaretroperitoneal,biopsyfrom
thelesionisasshownbelow.Molecular
analysisdemonstratedt(12,16).Amost
probablediagnosisis

a)Myxoidliposarcoma
b)Lipoma
c)Synovialsarcoma
d)Pleomorphicsarcoma
CorrectAnswer-A
Ans:A.Myxoidliposarcoma
Myxoidliposarcomaisthemostcommonhistologictype.
Itiscomposedofmonomorphic,fusiformorstellatecells
representingprimitivemesenchymalcells,lyingdispersedin
mucopolysaccharide-richgroundsubstance.
Occasionaltumourgiantcellsmaybepresent.Theprominent

meshworkofcapillariesformingchicken-wirepatternisa
conspicuousfeature
RefRobbin'sbasicsofpathologypageno.792.


256.47.35-year-oldwomanwithalong
historyofdyspnea,chroniccough,
sputumproduction,andwheezingdies
ofrespiratoryfailurefollowingaboutof
lobarpneumonia.Shewasnotasmoker
oranalcoholic.Thelungautopsyis
shownintheimage.Whichofthe
followingunderlyingconditionwasmost
likelyassociatedwiththepathologic
changesshownhere?

a)Mutationindyneinarms
b)Cysticfibrosis
c)Alpha1antitrypsindeficiency
d)Antibodiesagainsttype4collagen
CorrectAnswer-C

CorrectAnswer-C
Ans:C.Alpha1antitrypsindeficiency
Thephotomicrographaboveisfromapatientwithhereditaryalpha-
1-antitrypsindeficiency.
Alpha-1-antitrypsinisnormallypresentin
serum,tissuefluidsandmacrophages.It'sabig-timeinhibitorof
proteases(whicharedestructiveenzymessecretedbyneutrophils
duringinflammation).Goodthingtohavearound.
Patientswhoaredeficientinalpha-1-antitrypsinarelessableto
countertheeffectsofproteases,andtheelastictissueinthelungis
eventuallydestroyed,producingemphysema(irreversible
enlargementoftheairspacesdistaltotheterminalbronchiole).
Inadditiontoemphysema,patientswithalpha-1-antitrypsin
deficiencymaydevelophepatitisandevencirrhosisfrom
accumulationofabnormally-foldedalpha-1-antitrypsinwithin
hepatocytes.
Theseproteinsarevisibleasbrightlyeosinophilicinclusionswith
regularhematoxylinandeosinstaining.Thediseasevariesinits
presentation;somecasesaresevereatbirth,andothersare
asymptomaticuntillaterlife.Inpatientswithsevereliverdisease,
livertransplantationisthetreatmentofchoice.


257.GraphshowingthreedrugsA,B&C.
Whichofthefollowingdrugsshownin
thegraphbelowhasthehighest
potency?

a)DrugA
b)DrugB
c)DrugC
d)BothDrugA&B
CorrectAnswer-A
Ans:A.DrugA
Thelineontheleftismorepotent.
DrugAismorepotent.
DrugPotency:
ThepositionofDRConthedoseaxisistheindexofdrugpotency
whichreferstotheamountofdrugneededtoproduceacertain

response.
ADRCpositionedrightwardindicateslowerpotency.
Relativepotencyisoftenmoremeaningfulthanabsolutepotency
andisgenerallydefinedbycomparingthedose(concentration)of
thetwoagonistsatwhichtheyelicitahalf-maximalresponse(EC50
).
Drugpotencyisclearlyafactorinchoosingthedoseofadrug.
Ref:K.D.Tripathi7thEdition.Page.54?55


258.InthefollowingX-rayofthewrist,whatis
theexactage-
a)2yrs
b)8yrs
c)6yrs
d)9yrs
CorrectAnswer-C
Answer-C-6yrs
Assessmentsofskeletalmaturityinpre-pubertalchildrenare
primarilybasedontheepiphysealsizeofthephalangesasthey
relatetotheadjacentmetaphyses.
Duringthisstageofdevelopment,theossificationcentersforthe
epiphysisincreaseinwidthandthickness,andeventuallyassumea
transversediameteraswideasthemetaphysis
Moreweightisgiventothesizeoftheepiphysisinthedistal

phalangesthantothatinthemiddlephalanges,andevenlessto
thatintheproximalphalanges.
http://www.chospab.es/biblioteca/DOCUMENTOS/Atlas_of_Hand_Bone_Age.pdf


259.Identifythephenomena
a)ElectricBurn
b)CrocodileBurn
c)Scalds
d)Putrefaction
CorrectAnswer-D
Answer-D-Putrefaction
Putrefactionisthedecompositionofthebodycarriedoutbythe
microbialaction.Aftercessationofhemostasis,thenaturalfloraof
thebodymigratesfromtheguttothebloodvesselsandspreadsall
overthebody.
Externalmicro-organismsenterthebodythroughthealimentary
canal,respiratorytract,andopenwounds.Intheabsenceofbody
defenses/immunemechanisms,themicrobeskeepgrowing,asthey
feedupontheproteinsandcarbohydratesofthebloodandbody
parts.
Theprincipalbacterialagentcausingputrefactionisthegram-
positive,anaerobic,androd-shapedClostridiumwelchii.
Itreleaseslecithinase,whichcauseshydrolysisoflecithinpresentin
thebloodcells,causingtheirlysis.Putrefactionbeginswithinan

hourofdeath,butthepeakactivityofthemicrobesoccursaround
the24-hourtimeframe.
https://www.ncbi.nlm.nih.gov/books/NBK539741/


260.IdentifythefollowingImage
a)Neisseriameningitidis
b)Neisseriacinerea
c)Neisseriagonorrhoeae
d)Neisseriapolysaccharea
CorrectAnswer-A
Answer-A-Neisseriameningitides
Neisseriameningitidesoftenreferredtoasmeningococcus,isa
Gram-negativebacteriumthatcancausemeningitisandotherforms
ofmeningococcaldiseasesuchasmeningococcemia,life-
threateningsepsis.
N.meningitidisisagram-negativeproteobacteriumandmember
ofthebacterialfamilyofNeisseriaceae.
N.meningitidisisafastidiousbacteria,dyingwithinhourson
inanimatesurfaces,andiseitheranencapsulatedor
unencapsulated,aerobicdiplococcuswitha"kidney"or"coffee-
bean"shape.
Jawetz27/e-pg-287;


261.IdentifytheImage
a)Trichuristrichiura
b)Ancylostomaduodenale
c)Paragonimus
d)Strongyloides
CorrectAnswer-A
Answer-A-Trichuriasistrichiura
Itisbrownincolourbeingbile-stained.
Ithasatripleshell,theoutermostlayerofwhichisstainedbrown.
Itisbarrel-shapedandabout50umlongand25umwideinthe
middle,withaprojectingmucusplugateachpolecontainingan
unsegmentedovum
Theplugsarecolourless.Theeggfloatsinthesaturatedsalt
solution
Jawetz27/e-pg-724


262.Identifytheorganismcausingan
infectionontheupperarm
a)Tineacorporis
b)Tineacapitis
c)Tineacruris
d)Tineamanus
CorrectAnswer-A
Answer-A-TineaCorporis.
Dermatophytosisoftheglabrousskincommonlygivesrisetothe
annularlesionsofringworm,withaclearing,scalycentersurrounded
byaredadvancingborderthatmaybedryorvesicular.
Thedermatophytegrowsonlywithindead,keratinizedtissue,but
fungalmetabolites,enzymes,andantigensdiffusethroughtheviable
layersoftheepidermistocauseerythema,vesicleformation,and
pruritus.
Thelesionsexpandcentrifugallyandactivehyphalgrowthisatthe
periphery,whichisthemostlikelyregionfromwhichtoobtain
materialfordiagnosis.

Whentheinfectionoccursinthegroinarea,itiscalledTinea
cruris,
orjockitch.
Tineamanusreferstotheringwormofthehandsorfingers.
Tineacapitisisdermatophytosisorringwormofthescalpandhair
Jawetz27/e-pg-668


263.ImmunoglobulinImageshownbelowis
of-
a)IgA
b)IgG
c)IgM
d)IgE
CorrectAnswer-A
Ans:A.IgA
IgAisthemajorimmunoglobulinresponsibleformucosalimmunity.
ThelevelsofIgAintheserumarelow,consistingofonly10?15%of
totalserumimmunoglobulinspresent.Incontrast,IgAisthe
predominantclassofimmunoglobulinfoundinextravascular
secretions.
Inserum,IgAissecretedasamonomerresemblingIgG.Inmucous
secretions,IgAisadimerandisreferredtoassecretoryIgA.
ThissecretoryIgAconsistsoftwomonomersthatcontaintwo
additionalpolypeptides:theJchainthatstabilizesthemoleculeand
asecretorycomponentthatisincorporatedintothesecretoryIgA
whenitistransportedthroughanepithelialcell

Jawetz27/e-pg-138


264.A15yearsoldboypresentedwithfever
andchillsfor3days.Onexamination,he
wasfoundtohavedelayedskinpinch
timeanddryoralmucosa.Aperipheral
bloodsmearrevealedthefollowing
picture.Identifythepathogeninvolved?

a)Babesia
b)Plasmodiumvivax
c)Plasmodiumfalciparum
d)Salmonellatyphi
CorrectAnswer-B
Answer-B-Plasmodiumfalciparum
Givenclinicalpresentationsuggestiveofmalaria.
Doubleringsinerythrocytes&banana-shapedgametocytesona
peripheralbloodsmear-TypicalP.falciparuminfection.

peripheralbloodsmear-TypicalP.falciparuminfection.
Theearlyringformintheerythrocyteisverydelicateandtiny,
measuringonlyone-sixthoftheredcelldiameter.Ringsareoften
seenattachedalongthemarginoftheredcell,theso-calledform
appliqueoraccole.
Binucleaterings(doublechromatin)arecommonresemblingstereo
headphonesinappearance.Severalringsmaybeseenwithina
singleerythrocyte.
Paniker'sTextbookofMedicalParasitology-8thedition,pg-73;
Jawetz27/e-medicalparasitology-pg-721



265.Apatientcomplainsaboutpainful
blistersaroundtheangleofmouth
identifythepathogen

a)HerpesLabialis
b)Herpangina
c)Herpeszoster
d)Epstein-Barr
CorrectAnswer-A
Answer-A-HerpesLabialis
Herpeslabialis
,commonlyknownascoldsores,isatypeof
infectionbytheherpessimplexvirusthataffectsprimarilythelip.
Symptomstypicallyincludeaburningpainfollowedbysmallblisters
orsores.Thefirstattackmayalsobeaccompaniedbyfever,sore
throat,andenlargedlymphnodes.
Relapsinginfectionsarelimitedtothemucosaofthehardpalateor,
inolderchildrenandadults,thelips.Thenumberofrelapses
decreasesaftertheageof35years.
https://www.ncbi.nlm.nih.gov/pmc/articles/PMC2602638/



266.Apatientwiththefollowingfeature
shownintheimage.Thepatientreports
havinganother3-year-oldsiblingat
home,whoisfullyimmunizedasperthe
immunizationschedule.whatisthebest
measuretopreventdiphtheriainthe
siblingofthediphtheriacasechild.

a)Givediphtheriatoxoidbooster
b)GiveafullcourseofDPTvaccine
c)Giveprophylacticerythromycin
d)Nothingisrequiredtobedone
CorrectAnswer-D
Ans.D.Nothingisrequiredtobedone
Asperthenationalimmunizationprotocol,eachchildreceivesDPT
boosterat16-24
months,andthenat5years.nowasthechildintheMCQis3years
oldandisimmunizedtodate,thechildmusthavereceivedtheDPT

boosterinjectionjustwithinthelast2years.hencethechildis
protectedandprobablynoaddedimmune-preventionorvaccination
isrecommended.
Page174,25ed.PARK


267.Themostcommonsiteoforiginofthe
diverticulumofthepharynxseeninthe
bariumswallowgivenbelowis

a)Betweenstylopharyngeusandpalatopharyngeus
b)Betweenmiddleandinferiorconstrictor
c)Betweeninferiorconstrictorandesophagus
d)Betweenthyropharyngeusandcricopharyngeus
CorrectAnswer-D
Ans:(d)Betweenthyropharyngeusandcricopharyngeus
ThegivenpictureisthebariumswallowshowingZenker's
diverticulum.
>Zenkerdiverticulum,apulsiondiverticulumofthehypopharynx,isa
rarelesionthatoccursinelderlypopulations.>Theconditionresults
inaclassicpresentationofsymptoms,withcomplicationsthat
includeaspirationandpneumonia,andismanagedbyendoscopic
andsurgicalrepair.
Ref.ScottBroTon,Vol3;746

Ref.ScottBroTon,Vol3;746
Ref:https://emedicine.medscape.com/article/836858-overview.


268.BattlesignimageBluishPurplecolour
behindmastoid?
a)Battlesign
b)Bezoldabscess
c)BothAandB
d)Noneofthese
CorrectAnswer-A
Ans.(a)Battlesign.
>BattleSign(alsocalledBattle'sSign)isdefinedasbruisingover
themastoidprocess.Itisretroauricularormastoidecchymosisthat
istypicallytheresultofheadtrauma.
ref:https://www.statpearls.com/kb/viewarticle/18169?
utm_source=pubmed


269.Thepatientpresentswith,fever,
dysphagia.Imageshowingpushing
tonsil.whatisthediagnosis?

a)Parapharyngealabscess
b)Retropharyngealabscess
c)Peritonsillarabscess
d)Ludwig'sangina
CorrectAnswer-C
Ans.C.Peritonsillarabscess
Peritonsillarabscess,alsoknownasquinsy,isthelocalized
collectionofpusinperitonsillarspacebetweenthetonsillarcapsule
andsuperiorconstrictormuscle.
>Thetonsilisfoundpusheddownwardandmedially.
>Theuvulaisswollenandedematousandpushedtotheopposite
side.
>Thereisabulgeonthesoftpalateandanteriortonsillarpillar.
>Mucousmaybeseenoverlyingthetonsillarregion.

ref:https://www.ncbi.nlm.nih.gov/books/NBK519520/


270.Thispatientgivesahistoryoftoothache
foroneweek.Whatisthediagnosis:
a)Acuteparotitis
b)Angioneuroticedema
c)Ludwig'sangina
d)Parapharyngealabscess
CorrectAnswer-C
Ans.C.Ludwig'sangina
>Thisisaninfectionofthesubmandibularspace,i.e.floorofthe
mouth.
>ThemostcommoncauseofLudwig'sanginaisdentalCaries.
>Thesubmandibularspaceisdividedinto2spacesbythe
mylohyoidmuscle(alsoknownastheoraldiaphragm).
>Thespaceabovethemylohyoidisknownassublingualspace.
Heretheinfectionspreadsfrompremolartooth.
>Thespacebelowthemylohyoidisknownassubmaxillaryspace
laterallyandsubmentalspaceinthecenter.
>Infectiontothisspacefollowscariousmolars.

(Ref.ScottBrown,8thed.,VoI3;628)


271.Themovementislostin:
a)Thirdnervepalsy
b)Trochlearpalsy
c)Sixthnervepalsy
d)Facialnervepalsy
CorrectAnswer-B
Ans:b.Trochlearpalsy
>Thetrochlearnerveisthefourthcranialnerve(CNIV)andoneof
theocularmotornervesthatcontroleyemovement.
>Thetrochlearnerve,whilethesmallestofthecranialnerves,has
thelongestintracranialcourseasitistheonlynervetohaveadorsal
exitfromthebrainstem.
>Itoriginatesinthemidbrainandextendslaterallyandanteriorlyto
thesuperiorobliquemuscle.
Ref:https://www.ncbi.nlm.nih.gov/books/NBK537244/


272.Identifytheconditiongivenbelowinthe
image
a)Peutzjeghers
b)Juvenilepolyp
c)Villousadenoma
d)Hyperplasticpolyp
CorrectAnswer-B
Answer-B.Juvenilepolyp
TheimageshownhereisJuvenilepolyp.
Thisisabrightred,glisteningpedunculatedsphere(cherrytumour),
whichisfoundininfantsandchildren.
Itcancausebleeding,orpainifitprolapsesduringdefecation.
Ref-BaileyandLove,Shortpracticeofsurgery,27thedition
publishedin2018Pg1327



273.BariumSwallowexaminationisshown.
Whatcanbethemostprobable
diagnosis?

a)EsophagealCa
b)EsophagealRing
c)EsophagealTear
d)AchalasiaCardia
CorrectAnswer-A
Answer-A.EsophagealCa
TheimageisEsophagealcarcinoma
Squamouscellcancerandadenocarcinomaarethemostcommon
types.
Theclassicappearanceofamidoesophagealproliferative
squamouscellcarcinoma.
Squamouscellcarcinomaoftheesophagusproducinganirregular
stricturewithshoulderedmargins.
Ref-BaileyandLove,Shortpracticeofsurgery,27thedition

publishedin2018Pg1086


274.Identifytheconditiongivenintheimage
below?
a)Chroniclymphedema
b)Cushingsyndrome
c)Osteoporosis
d)None
CorrectAnswer-A
Answer-A.Chroniclymphedema
Theimageshowsbuffalohumpappearanceofthefootseenin
chroniclymphedema.
Lymphoedemacharacteristicallyinvolvesthefoot.
Thecontouroftheankleislostthroughinfillingofthesubmalleolar
depressions,a'buffalohump'formsonthedorsumofthefoot,the
toesappear'square'becauseofconfinementoffootwearandthe
skinonthedorsumofthetoescannotbepinchedbecauseof
subcutaneousfibrosis(Stemmer'ssign).

Ref-BaileyandLove,Shortpracticeofsurgery,27thedition
publishedin2018Pg998



275.Identifytheconditiongivenbelow-
a)MCUwithBulbarurethralstricture
b)MCUwithpenilestricture
c)RGUwithmembranousstricture
d)RGUwithprostaticstricture
CorrectAnswer-A
Answer-A.MCUwithBulbarurethralstricture
Thecommoncausesofurethralstricturesare:
Inflammatory

1. Secondarytourethritis
2. Secondarytobalanitisxeroticaobliterans.(BXO)
Traumatic
1. Bulbarurethralinjury
2. Pelvicfractureurethraldisruptioninjury
Iatrogenic
1. Secondarytourethralinstrumentationincludingcatheterizationand


transurethralprostatectomy
2. Secondarytoradicalprostatectomy
3. Secondarytoradiotherapyforprostatecancer
Idiopathic
Ascendingurethrogramshowingurethralstricturesofthebulbar
urethra.
Ref-BaileyandLove,Shortpracticeofsurgery,27thedition
publishedin2018Pg1482



276.Whatistheuseoftheinstrumentgiven
intheimagebelow?
a)Laparoscopicsterilisation
b)Removalofectopicpregnancy
c)Terminationofpregnancy
d)Laparoscopicprocedurestocreatepneumoperitoneum
CorrectAnswer-A
Ans.A.Laparoscopicsterilisation
Lapringapplicator:Duringatuballigation,a2cmto3cmsegmentof
thefallopiantubeisdrawninsideanarrowcone-shapedapplicator.
Thesilasticringpreviouslystretchedaroundtheapplicator,isthen
releasedontothetuballoop.Oncethesilasticringcontracts,the
fallopiantubeisblocked
Ref.https://www.surgicalinstruments.com/general-lap-bariatric-
instruments/fallopian-ring-applicator-for-tubal-ligation



277.Identifytheimagebelow:
a)Femalecondom
b)Malecondom
c)Chaaya
d)Today
CorrectAnswer-A
Ans.A.Femalecondom
Thefemalecondom(Femidom):
Itisapouchmadeofpolyurethanethatlinesthevaginaandalsothe
externalgenitalia.Itis17cminlengthwithoneflexiblepolyurethane
ringateachend.Theinnerringattheclosedendissmaller
comparedtotheouterring.
Ref.Duttagynaecology6thed.Pageno.476


278.Whatconditionofthemotheris
associatedwiththefollowingfetal
anomaly?

a)ACEinhibitor
b)GDM
c)PregestationalDM
d)Valproate
CorrectAnswer-C
Ans.C.PregestationalDM
TheimageshowsCaudalregressionsyndrome
Thisrareanomalyischaracterizedbytheabsenceofthesacral
spineandoftenportionsofthelumbarspine.Itisapproximately25
timesmorecommoninpregnancieswithpregestationaldiabetes
Sonographicfindingsincludeaspinethatappearsabnormallyshort,
lacksthenormallumbosacralcurvatureandterminatesabruptly
abovetheleveloftheiliacwings.
Becausethesacrumdoesnotliebetweentheiliacwings,theyare

abnormallyclosetogetherandmayappear"shield-like."
Theremaybeabnormalpositioningofthelowerextremitiesanda
lackofnormalsofttissuedevelopment.
OBSWILLIAMSed.24pageno.204


279.Interpretthepartogram.
a)CPD
b)Maternalexhaustion
c)Inadequateuterinecontractions
d)Ruptureuterus
CorrectAnswer-A
Ans.A.CPD
Patientadmittedat7AM
Cervicaldilatation4cm
Head:5/5thpalpable
After2hrs
Cervicaldilatation:5cm

Cervicaldilatation:5cm
Head:3/5thpalpable
After4hrs:11AM
Cervicaldilatation7cm
Head:2/5thpalpable
Cervicaldilatation:Reached10cm
Uterinecontractionisstrongafter4hrsbuttheDescentofthehead
isstill2/5th(constantafter4hrs)
Diagnosis:Arrestofdescent
Themostcommoncauseofthearrestofdescentiscephalopelvic
disproportion
Ref.https://hetv.org/resources/reproductive-
health/impac/Symptoms/Unsatisfactory__progress_labour_S57_S67.html
https://apps.who.int/iris/bitstream/handle/10665/58903/WHO_FHE_MSM_93.8.pdf?
sequence=1



280.60yrfemalewithahistoryofintermittent
bleedingwithUSGpicshownbelow.
Whatisthediagnosis?

a)Endometrialpolyp
b)Caendometrium
c)Submucosalfibroid
d)None
CorrectAnswer-A
AnsA.Endometrialpolyp
The3Dimagegivesabetterpictureofalargevesselpassingintoa

polyp.
Endometrialpolyps:
Endometrialpolypsarelocalizedovergrowthofthebasalstroma
withcentralbloodvesselscoveredbysurfaceendometriummostly
risingfromthefundalarea.
Itisimportanttomentionthattheliabilityofpolypstobleedfollows
theirpeculiarstructure.Eachpolyphasacentralfeedingvessel,soft
stromaandbacktobackglandularity.
Onlineref.http://www.gynaecologist4u.com/9.html


281.WhatwillbetheHysteroscopicfindingin
thegivenimage?
a)BilateralHydrosalpinx
b)Bilateralcornualblock
c)NormalHSG
d)Bicornuateuterus
CorrectAnswer-A
Ans.A.BilateralHydrosalpinx
TheimageisHysterosalpingogramshowingbilateralhydrosalpinx
(fimbrialblock).
Hysterosalpingogramshowingradio-opaqueshadowfillingthe
uterinecavity.Thetubesofbothsidesaredistendedwiththeradio-
opaquedye.Thereisnoevidenceofperitonealspillage.




NORMALHSGSHOWINGSPILLAGE
Hysterosalpingogramshowingbilateralcornualblock.
Hysterosalpingogramshowingbicornuateuterus.
Ref.DCDutta'sTextbookofGynecology6thEditionPageno.
657-659,661



282.Aladyontreatmentforinfertility
developedascites,abdominalpain&
dyspnea.
USGofthepatientwasdoneshown
below.Whatwillbethediagnosis?

a)PCOS
b)OHSS
c)Thecaluteincyst
d)Mucinouscystadenomas
CorrectAnswer-B
ANS.B.OHSS
Ovarianhyperstimulationsyndrome(OHSS):

Itisaclinicalsymptomcomplexassociatedwithovarian
enlargementresultingfromexogenousgonadotropintherapy.
Symptomsmayincludeabdominalpainanddistension,ascites,
gastrointestinalproblems,respiratorycompromise,oliguria,
hemoconcentration,andthromboembolism.
Thesesymptomsmaydevelopduringovulationinductionorinearly

pregnanciesthatwereconceivedthroughexogenousovarian
stimulation.
REF:DCDutta'sTextbookofGynecology6thEditionPAGENO.
529,530,661



283.5-yearmalechildpresenttotheclinic
withH/OrecurrentinfectionOn
examinationhewasfoundtohave
rashesshownbelowintheimage.On
routinebloodinvestigation,lowplatelet
countwasfoundwhatwillbethe
diagnosis?

a)WiskottAldrichsyndrome
b)Job'ssyndrome
c)ChediakHigashisyndrome
d)None
CorrectAnswer-A
Ans.A.WiskottAldrichsyndrome
WiskottAldrichsyndrome:Thereisamutationinthegeneencoding
Wiskott-Aldrichsyndromeprotein(WASP),whichislocatedon
Xp11.23.TheX-linkeddiseaseischaracterizedby
thrombocytopenia,eczema,andamarkedvulnerabilitytorecurrent

thrombocytopenia,eczema,andamarkedvulnerabilitytorecurrent
infection,resultinginearlydeath.
Plateletsaresmallinsize.
ThereisaprogressivelossofTlymphocytesintheperipheralblood
andintheT-cellzones(paracorticalareas)ofthelymphnodes,with
variabledefectsincellularimmunity.
Patientsdonotmakeantibodiestopolysaccharideantigens,andthe
responsetoproteinantigensispoor.TheyarepronetodevelopB-
celllymphomas.
Robbins&CotranPathologicBasisofDisease,9ed:Pageno
242



284.Thepatientispresentingwiththe
deformityofthefingerasshown.ThePIP
isinvolvedbuttheDIPisspared

a)Osteoarthritis
b)Rheumatoidarthritis
c)PsoriaticArthritis
d)Ankylosingspondylitis
CorrectAnswer-B
Ans:b.Rheumatoidarthritis
>Swan-neckdeformityofthefingerdescribeshyperextension
atthePIPjointwithflexionoftheDIPjoint

>RheumatoidArthritisClinicalPresentation
Ref:https://www.medscape.com/answers/331715-5365/what-is-
a-swan-neck-deformity-of-the-finger-in-rheumatoid-arthritis-ra



285.Whatisthemostlikelydiagnosisgiven
intheimage-
a)Popeye'ssign
b)Griesingersign
c)Risingsunsign
d)Winnersign
CorrectAnswer-A
Ansa.Popeye'ssign
>presentedtotheorthopedicclinicwithalargebulgeonhisright
upperarm.
>Examinationrevealedanobviousdeformityintheanteriormid-
upperarmthatmaybecomemoresignificantduringelbowflexion
>ThistypicalfindingonphysicalexaminationisknownasPopeye
signorPopeyedeformity,whichiscausedbybulgingofthebiceps
musclebellyafterruptureofthebicepstendon
https://academic.oup.com/qjmed/article/112/12/931/5487419


286.Thepatientispresentingwithpain
aroundthebaseofthethumb,tendons
involved-

a)APB&EBL
b)APL&EPB
c)APB&EPB
d)APL&EPL
CorrectAnswer-B
Ans:b.APL&EPB
>TheconditionexplainedinthepatientisDeQuervain's
Tenosynovitis
>DeQuervain'sTenosynovitisisapainfulinflammationoftendons
onthesideofthewristatthebaseofthethumb.
>Thesetendonsincludetheextensorpollicisbrevis(EPB)andthe
abductorpollicislongus(APL).

>Thesemusclesarelocatedonthedorsalsideoftheforearmand
gotothelateralsideofthethumbthroughafibrous-osseoustunnel
madeoftheprocessusstyloideusradiiandtheextensor
retinaculum.

retinaculum.
https://www.physio-
pedia.com/De_Quervain%27s_Tenosynovitis
Aclinicalcaseofabnormallengtharms,longfingersandtoes
Ans:Marfansyndrome(Ortho)


287.Theshownangleintheimageisknown
as
a)Cobb'sangle
b)Bohler'sangle
c)Fergusonangle
d)Baumann'sangle
CorrectAnswer-A
Ans:a.Cobb'sangle
>TheimageshowstheScoliosispatterninVertebra.
TheCobbAngleisusedasastandardmeasurementtodetermine
andtracktheprogressionofscoliosis.
>Theangleofcurvatureismeasuredbydrawinglinesparalleltothe
upperborderoftheuppervertebralbodyandthelowerborderofthe
lowestvertebraofthestructuralcurve.
>Thenerectingperpendicularsfromtheselinestocrosseachother,
theanglebetweentheseperpendicularsbeingthe`angleof
curvature'
>ACobbangleof10degreesisregardedasaminimumangulation

>ACobbangleof10degreesisregardedasaminimumangulation
todefineScoliosis.
Ref:https://www.physio-pedia.com/Cobb%27s_angle


288.Whatisthediagnosisof55oldwomen
withachroniclowbackache-
a)Osteoporosis
b)Hurler'ssyndrome
c)Paget'sdisease
d)Renalosteodystrophy
CorrectAnswer-A
Ans:a.Osteoporosis
>Osteoporosisis,forthemostpart,adiseaseofaging.
>Theolderseniorisathighriskforosteoporosis.
>Theriskforlowbonedensityincreaseswithageandsignificantly
impactsbonestrength.
https://www.ncbi.nlm.nih.gov/pmc/articles/PMC2685234/


289.Identifytheconditionshowninthe
image?
a)IchthyosisVulgaris
b)Syndromalichthyosis
c)Leprosy
d)Sarcoidosis
CorrectAnswer-A
Answer-A.IchthyosisVulgaris
Ichthyosisisoneofthemostcommongenodermatoses.Itis
characterizedbydry(xerotic)scalyskinalloverthebody.Scalesare
dull-brown-black.Thebasicdefectisanimpairmentinthebarrier
functionofskinandinabilitytomaintainmoisture.
Ichthyosiscanbedividedinto:-
1. Inheritedichthyosis(IchthyosisVulgaris,X-linkedIchthyosis,
Lamellarichthyosis,epidermolytichyperkeratosis).
2. Syndromalichthyosis
3. Acquiredichthyosis(Ichthyosisacquisita).
ThemostcommontypeofichthyosisisichthyosisVulgarisinwhich
thereisadeficiencyoffilaggrinaproteinfoundinthegranularlayer

thereisadeficiencyoffilaggrinaproteinfoundinthegranularlayer
(stratumgranulosum)
Manydifferentmetaphorshavebeenusedtodescribethe
appearanceandtextureoftheskininvarioustypesandstagesof
ichthyosis,e.g.,alligatorskin,Crocodileskin(Sauroderms),Fish
skinandporcupineskin.
Ref-ArvindAroraskin,6thedition,pageno203


290.Thepatientcamewithhistoryofbullae
involving>30"A,bodysurfacearea
alongwitherosionsofthelipsandother
mucosaeforthepast7days.Whatisthe
mostprobableunderlyingetiology?

a)Bacterialinfection
b)Viralinfection
c)Drugs
d)Malignancy
CorrectAnswer-C
Ans:C.Drugs
TypicalofSteven-Johnsonsyndrome&toxicepidermalnecrolysis.
Toxicepidermalnecrolysis(TEN):
Alsoreferredtoas"Lyell'ssyndrome".
Rare,life-threateningskincondition.
Usuallycausedbyadrugreaction.


291.Whichofthefollowingdoestheimageof
Capnographbelowdepicts?
a)Duringinspiration
b)Inspirationwithcardiacoscillations
c)Duringexpiration
d)spontaneousrespiration
CorrectAnswer-D
Ans.D.spontaneousrespiration
Theimageinthegivenquestionshowsacapnographwithclefts
duringphaseIIIwhichindicatesspontaneousbreathingeffortsby
thepatientduringcontrolledmechanicalventilation.Alsoknown
ascurarenotchorcleft.
Ref:https://www.ncbi.nlm.nih.gov/pubmed/24132805
Miller'sAnesthesia8thedition(Pageno.1554)



292.Identifythedeviceshownintheimage.
a)NasopharyngealAirway
b)EndotrachealTube(Cuffed)
c)GuedelAirway
d)LaryngealMaskAirway
CorrectAnswer-D
Ans:D.)LaryngealMaskAirway
Laryngealmaskairway
Atypeofsupraglotticairway.
ItisalsoreferredtoasBrainmaskasitwasinventedbyDr.Archies
Brain.
LMAprovidesanairwayintermediatebetweenthefacemaskand
trachealtube
Itisamedicaldevicethatkeepsapatient'sairwayopenduring
anaesthesiaorunconsciousness.
TheprincipleofLMAisthatitprovidesaneffectivegas-tightseal
aroundthelaryngealinletwithoutanythinghavingtopassthrough
thevocalcord.
LMAisveryeffectiveinmaintainingapatentairwayinthe
spontaneouslybreathingpatient.

Itiscomposedofanairwaytubethatconnectstoanellipticalmask
withacuffwhichisinsertedthroughthepatient'smouth,downthe
windpipe,andoncedeployedformsanairtightsealontoptheglottis
(unliketrachealtubeswhichpassthroughtheglottis)allowinga
secureairwaytobemanagedbyahealthcareprovider.
LMAisavailablein8differentsizes
Disadvantage:
OneofthemajordisadvantagesofLMAisthatitssealaround
laryngealinletdoesnotalwayspreventaspiration.
Therefore,LMAshouldnotbeusedwherethereisanincreasedrisk
ofaspirationofstomachcontents,e.g.'fullstomachpatient.


293.IdentifytheconditionintheX-raygiven
below-
a)TGA
b)TAPVC
c)TOF
d)Ebstein'sanomaly
CorrectAnswer-C
Answer-C-TOF
TheimageshownaboveisofTetralogyofFallotcondition.
Classicallythechestradiographdemonstratesa'boot-shaped'heart
withpoorlydevelopedlungvasculature.
Thediagnosisisconfirmedwithechocardiography.
Surgerytocorrectthetetralogyisthemainstayoftreatmentandis
usuallycarriedoutat4-6
monthsofage,whenpossible.
Ref-BaileyandLove,Shortpracticeofsurgery,27thedition
publishedin2018Pg905



294.Achestradiographobtainedamalewith
hypertension.Whatwillbethe
diagnosis?

a)TetralogyofFallot
b)Ebstein'sAnomaly
c)TAPVC
d)CoarctationofAorta
CorrectAnswer-D
Answer-D-CoarctationofAorta
Thechestradiographclassicallydemonstratesrib-notchingbecause
ofdilatedposteriorintercostalvessels.Theheartisusuallyofnormal
sizeintheolderchildandshowsaclassical'threesign"replacing
thetypicalaorticknuckle.
Ref-BaileyandLove,Shortpracticeofsurgery,27thedition
publishedin2018Pg906



295.Steeplesignisseeninwhichofthe
followingcondition?
a)Acuteepiglottitis
b)Acutelaryngotracheobronchitis
c)Laryngealpapillomatosis
d)Bilateralabductorparalysis
CorrectAnswer-B
Answer-B-Acutelaryngotracheobronchitis
TheimagebelowindicatesAcutelaryngotracheobronchitis.
Croupisusuallyofsloweronsetthanacuteepiglottitisandoccurs
mostcommonlyinchildrenunder2yearsofage.
Thechildrenhavebiphasicstridorandareoftenhoarsewitha
typicalbarkingcough.
Ref-BaileyandLove,Shortpracticeofsurgery,27thedition
publishedin2018Pg743



296.Identifytheradiologicalsigngiven
below-
a)Diverticulitis
b)Ischaemiccolitis
c)Appendicitis
d)None
CorrectAnswer-B
Answer-B-Ischaemiccolitis
Single-contrastbariumenemashowsthenarrowingofthe
descendingcolonwithascallopedappearancewithulcerationorthe
classicaloedematous'thumbprinting'signinischaemiccolitis.
Plainfilmsarefrequentlytakenandoftenrevealsplenicflexure
irregularitywithamuralthickening.
Ref-DavidSutton,TextbookofRadiologyandImaging,Volume
1,Edition7th,Pg653
Basicofradiology,Lange,2ndedition,Pg642



297.Amiddle-agedmanwithaswellingover
thenecksincechildhoodwiththe
overlyingskinnotintactwhichhadabag
orworm-likeappearancewithablack
spotinthemiddle.Whatwillbethe
diagnosis?

a)Cirsoidaneurysm
b)Varicocele
c)Plexiformneurofibroma
d)Lymphangioma
CorrectAnswer-C
Answer-C-Plexiformneurofibroma
PlexiformneurofibromasrepresentanuncommonvariantofNF-1in
whichneurofibromasarisefrommultiplenervesasbulgingand
deformingmassesinvolvingalsoconnectivetissueandskinfolds--
hencetheclinicaldescriptionoflesionsas"bagsofworms."

Skinexaminationalsorevealedmultipleneurofibromasandcaf?-au-
laitmaculesonthetrunkandarms.
Ref-
https://www.ncbi.nlm.nih.gov/pmc/articles/PMC4753888/#!po=35.7143



298.Identifytheradiologicalimagegiven
below-
a)Intussusception
b)Carcinomacolon
c)Sigmoidvolvulus
d)Sigmoidvolvulus
CorrectAnswer-A
Answer-A-Intussusception
Thisimageshowsintussusception.
Theclassicpresentationiswithepisodicabdominalpainand
screamingepisodesassociatedwiththepassageofbloodand
mucus('redcurrentjelly').
Theabdominalradiographmaydemonstrateanabsenceofbowel
gasintherightiliacfossawitharoundedsoft-tissuemassa

crescentofairattheapexofintussusception,orsmall-bowel
obstruction.
Ref-DavidSutton,TextbookofRadiologyandImaging,Volume
1,Edition7th,Pg872



299.Namethesignseeninthegivenbelow
imagerepresents-
a)Mickeymousesign
b)Stringsign
c)Tillauxsign
d)Stemmer'ssign
CorrectAnswer-A
Answer-A-Mickeymousesign
TheMickeyMousesignissaidtorepresentthenormalanatomyof
thecommonfemoralartery,commonfemoralveinandgreat
saphenousveinonultrasoundattheleveljustinferiortotheinguinal
crease.
Reference-Coleridge-SmithP,LabropoulosN,PartschH,Myers
K,NicolaidesA,CavezziA.Duplexultrasoundinvestigationof


theveinsinthechronicvenousdiseaseofthelowerlimbs--UIP
consensusdocument.PartI.Basicprinciples.Europeanjournal
ofvascularandendovascularsurgery:theofficialjournalofthe
EuropeanSocietyforVascularSurgery.



300.A35-year-oldmalepresentswith
recurrentepisodesofabdominalpain,
jaundice,andfatigueandunderwent
MRCP.Whatwillbethemostlikely
diagnosis?

a)Primarybiliarycirrhosis
b)Caroli'sdisease
c)Primarysclerosingcholangitis
d)Orientalcholangitis
CorrectAnswer-C
Answer-C-Primarysclerosingcholangitis
ThisimageshowsPSC.
MRimagingofthebiliarysystem,especiallyMRCP,hasbecome
moreimportantinbiliaryimaging,includingthedetectionofcalculiof

thegallbladderandbiliarytree.
ThediagnosisofPSCisprincipallybasedonthefindingsat
cholangiography,inwhichirregular,narrowedbileductsare
demonstratedinboththeintrahepaticandextrahepaticbiliarytree.
Ref-BaileyandLove,Shortpracticeofsurgery,27thedition
publishedin2018Pg1167
Basicofradiology,Lange,2ndedition,Pg313

invalidquestionid

This post was last modified on 30 July 2021